Está en la página 1de 134

2008

Oral and Maxillofacial Surgery Self Assessment


Tool (OMSSAT) Committee

1. Qué droga o qué clase de droga debería ser evitada si es posible en pacientes con
cardiomiopatía hipertrófica?

A. β- bloqueadores.
B. Inhibidores de la enzima convertidora de la angiotensina.
C. Bloqueadores de los canales de calcio.
D. Diuréticos.

RESPUESTA: D

Deshidratación en pacientes con esta condición actúan para incrementar gradientes de


presión en el tracto de alto flujo del corazón y generar un incremento en síntomas. Esto
puede ser exacerbado también con actividad extenuante y resultar en muerte súbita.
Deshidratación y el uso de diuréticos debería ser evitado si es posible así no alterar este
gradiente. Digitales, nitratos, vasodilatadores y agonistas β – adrenérgicos también son
evitados.

2. Cuál tipo de cardiomiopatía es más frecuente asociada con presiones de tracto de


alto flujo?

A. Restrictiva
B. Dilatada
C. Idiopática
D. Hipertrófica.

RESPUESTA: D.

Pacientes con cardiomiopatía hipertrófica son más frecuentemente asociados con tener
incremento de obstrucción del tracto de flujo ventricular izquierdo. Esto ocurre con
aproximadamente 25% de estos pacientes. Esto es usualmente relacionado a estrechamiento
del área subaórtica como secuela de la aposición de la válvula mitral en yuxtaposición al
septum interventricular alargado. Cardiomiopatía hipertrófica es una enfermedad
hereditaria autosómica dominante al menos el 50% de las veces. Estas son formas
esporádicas de la enfermedad debido a mutaciones espontáneas.

3. Que cuidado debería ser dado en la administración de anestésicos locales con


vasoconstrictor a pacientes hipertensos tratados con que medicación?

A. Inhibidores de la ECA
B. Betas Bloqueadores no selectivos.
C. Bloqueadores de los canales de calcio.
D. Bloqueadores de receptor de angiotensina.

RESPUESTA: B

Tratamiento de hipertensión con un beta bloqueador no selectivo va a afectar ambos


receptores beta-1 y beta-2. Epinefrina o levonordefrina va a normalmente causar
estimulación de ambos receptores alfa y beta adrenérgicos. Vasoconstrictor mediado por
Alfa-1 es esta sin oposición con beta bloqueados y puede resultar en hipertensión severa y
posible reflejo de bradicardia.
Inhibidores de la ECA afectan la vía de renina-angiotensina dirigiendo a una disminución
en la producción de la angiotensina II. Angiotensina II es un potente vasoconstrictor y por
tanto este reduce la vasoconstricción periférica y la postcarga y va a disminuir la presión
sanguínea. Bloqueadores de los canales de calcio va a dirigir a la disminución en la
vasoconstricción de la vasculatura periférica dirigiendo a la disminución de la presión
sanguínea.

4. El agente con el tiempo de acción más lento cuando se está tratando una crisis
hipertensiva es:

A. Esmolol .
B. Labetolol.
C. Nitroglicerina sublingual.
D. Hidralazina.

RESPUESTA: D

Esmolol es un agente beta bloqueador selectivo en contra los receptores B1. El tiempo de
inicio de acción para esmolol IV es 2-10 minutos. Labetalol tiene un inicio de 2-5min. Este
es un bloqueador B1 y B2 y también un receptor alfa. Inicio de la nitroglicerina sublingual
es de 2-5 minutos. Hidralazina es un dilatador periférico directo con prolongada duración
de acción.
5. Un masculino blanco de 56 años ha sido diagnosticado con hipertensión secundaria
asociada con hiperaldosteronismo. El hallazgo de laboratorio consistente con este
diagnóstico es:

A. Hiperkalemia
B. Hipokalemia
C. Hipercalcemia
D. Hipocalcemia

RESPUESTA: B

Un adenoma adrenal o hiperplasia de la glándula adrenal incrementa la secreción de


aldosterona de la zona glomerulosa de la glándula. Aldosteronismo primario produce
hipertensión secundaria asociada con hipocalemia y actividad de la renina suprimida.
Aldosterona causa excreción de potasio y reabsorción del sodio del túbulo distal y el ducto
colector, que dirige a hipocalemia e hipernatremia con incremento del volumen del fluido
extracelular. Niveles de magnesio y cloro pueden también estar reflexivamente alterados.
Cambios en los niveles de calcio no son típicamente observados.

6. Un masculino de 70 años de edad con historia de estenosis mitral se presenta


quejándose de fatiga de larga data, disnea y dolor ocasional en el pecho.
Clínicamente, el muestra distensión yugulovenosa, edema periférico y
hepatoesplenomegalia. Hallazgos clínicos adicionales más consistentes con esta
presentación son:

A. Presión de la arteria pulmonar disminuida por cateterización.


B. Hipertrofia ventricular izquierda por radiografía de tórax.
C. Hallazgos en ECG de desviación del eje derecho, onda P elevada en II, III y AVF.
D. Componente pulmonar ausente del segundo sonido cardíaco.

RESPUESTA: C

Signos y síntomas de Cor Pulmonar/Hipertensión pulmonar están descritas. Los hallazgos


en ECG en la respuesta C son vistos en la hipertrofia atrio-ventricular que resulta. La
presión arterial pulmonar podría estar elevada en este escenario. LVH podría verse como
una secuela, pero no es típico. El componente pulmonar del segundo sonido es acentuado
en lugar de ausentado.

7. Cuál de los siguientes tiene un incremento en el intervalo PR progresivo?

A. Bloqueo cardiaco de primer grado.


B. Bloqueo cardiaco de Mobitz I de segundo grado.
C. Bloqueo cardiaco de Mobitz II de segundo grado.
D. Bloqueo cardiaco de tercer grado.
RESPUESTA: B

El Bloqueo AV Mobitz tipo I de segundo grado es también asociado con menos de pausa
compensatoria y una duración QRS normal. Esto podría estar vista con la toxicidad de
drogas como digitales y beta bloqueadores.
Bloqueo cardíaco de primer grado, bloqueo en primer grado auriculoventricular (AV), se
define como prolongación del intervalo PR en el ECG a más de 200 mseg.
Con el bloqueo AV de primer grado, cada impulso auricular se transmite a los ventrículos,
resultando en una tasa ventricular regular. Este tipo de bloqueo AV pueden derivarse de los
retrasos en la conducción sistema en el nodo AV, el sistema de His-Purkinje, o una
combinación de ambos.
Bloqueo cardíaco de segundo grado o bloqueo de segundo grado auriculoventricular (AV)
de, se refiere a una trastorno de la conducción sistema cardiaco en el que algunos impulsos
auriculares no son conducidos a los ventrículos. Electrocardiográficamente, algunas ondas
P no son seguidas por un QRS complejo. Bloqueo de segundo grado de AV se compone de
2 tipos: Mobitz I o Wenckebach bloque, y Mobitz II.

El bloqueo AV Mobitz I de segundo grado se caracteriza por una progresiva prolongación


del intervalo PR, que se traduce en una progresiva reducción del intervalo R-R.
Últimamente, el impulso auricular falla a la conducción, un complejo QRS no se genera, y
no hay contracción ventricular. El intervalo PR es más corto en el primer tiempo en el
ciclo, mientras que el intervalo RR es más largo en el primer tiempo en el ciclo.

Bloqueo AV Mobitz II segundo grado se caracteriza por una inesperado impulso auricular
sin conducción. Así pues, los intervalos PR y RR entre latidos son bloqueos cardíacos
completos constantes, también conocidos como de tercer grado, bloqueo cardíaco, o
bloqueo de tercer grado auriculoventricular (AV) es un trastorno del sistema de
conducción cardiaca, donde no hay conducción a través del nodo AV. Por lo tanto, la
completa disociación de la actividad auricular y ventricular existe. El mecanismo de escape
ventricular puede ocurrir en cualquier lugar de la Nodo AV para el paquete-rama del
sistema de Purkinje.
8. Paciente femenino de 64 años de edad está cursando ahora su segundo día de
postoperada de toma de injerto de cresta iliaca para reconstrucción de defecto
mandibular. Ella continúa con disnea en reposo desde que salió de la anestesia. Su
prueba de Péptido natriuretico tipo B (PNB) es elevada. Esto podría indicar que el
paciente está sufriendo de:

A. Embolismo pulmonar.
B. Enfermedad pulmonar obstructiva crónica.
C. Acidosis metabólica.
D. Falla cardiaca congestiva.

RESPUESTA: D

PNB es una neurohormona que es liberada por el miocardio ventricular en respuesta al


volumen ventricular y la presión de la sobrecarga. En pacientes quienes presenta con disnea
de etiología desconocida, un PNB plasmático >100 picogramos/mililitros puede ser usado
como evidencia de falla cardiaca como una causa de disnea (exactitud diagnostica: 84%).
Esto es usado en la diferenciación de disnea debido a una falla cardiaca de causas no
cardiacas.

9. Paciente masculino de 70 años de edad es llevado a extracciones múltiples dentales.


Brevemente después de administrar anestesia local con epinefrina el se queja de
aplastamiento de tórax subesternal que se irradia a su brazo izquierdo. Cuál es la
intervención terapéutica más apropiada?

A. Nitroglicerina sublingual.
B. Oxigeno suplemental.
C. Aspirina 325 mg PO.
D. Sulfato de morfina intramuscular.

RESPUESTA: B

El paciente siendo tratado parece que sufre de daño isquémico al miocardio como describe
con los síntomas de dolor torácico subesternal. A pesar del acrónimo común MONA, el
tratamiento inicial de la enfermedad cardiaca isquémica es: Oxigeno, nitroglicerina,
aspirina y morfina, en ese orden.

10. Cuáles son los efectos de usar inhibidores de la ECA y diuréticos en combinación
para el tratamiento de falla cardiaca congestiva?

A. Efecto inotrópico y cronotrópico positivo.


B. Efecto inotrópico y cronotrópico negativo.
C. Incremento de la precarga y postcarga.
D. Disminución de la precarga y postcarga.
RESPUESTA: D

Inhibidores de la ECA y diuréticos son recomendados de rutina para el uso en el


tratamiento de la FCC. Inhibidores de la ECA bloquean el sistema renina- angiotensina
produciendo vasodilatación por limitación de la vasoconstricción inducida por la
angiotensina II. La vasodilatación es predominantemente arterial cual disminuye la
postcarga. Esto disminuye el trabajo del miocardio y disminuye los requerimientos de
energía del miocardio.

Diuréticos disminuye el fluido extracelular así disminuyendo la presión del llenado


ventricular o precarga. Este trata los síntomas de la FCC.

11. Es pregunta usando los aparatos de sonido.

12. Cuál de los siguientes es el factor de riesgo de muerte súbita cardiaca en un paciente
con estenosis aórtica?

A. 1%
B. 5%
C. 10%
D. 15%

RESPUESTA: B.

La estenosis aortica es la obstrucción del flujo sanguíneo a través de la válvula aortica.


Estenosis aortica (EA) tiene varias etiologías: unicúspide congénita y válvula bicúspide,
fiebre reumática y cambios degenerativos calcificantes de la válvula.
Fisiopatología: Cuando la válvula aórtica se vuelve estenótica, resistencia a la presión
sistólica de eyección ocurre y se desarrolla un gradiente de presión sistólica entre el
ventrículo izquierdo y la aorta. Válvulas aórticas estenótica tienen una disminución de la
apertura que lleva a un aumento progresivo de la presión sistólica ventricular izquierda.
Esto conduce a la sobrecarga de presión en el ventrículo izquierdo, que, con el tiempo,
provoca un aumento en el espesor de la pared ventricular (es decir, hipertrofia concéntrica).
En esta etapa, la cámara no es dilatada y la función ventricular se conserva, aunque
cumplimiento de la diastólica puede verse afectada.

Mortalidad / morbilidad: la muerte cardiaca súbita se produce en 3-5% de los pacientes con
EA. Adultos con EA tienen una tasa de mortalidad de 9% por año. Una vez que se
desarrollen los síntomas de la incidencia de muerte súbita aumenta a 15-20%, con un
promedio de duración de supervivencia de menos de 5 años. Los pacientes con angina de
esfuerzo o síncope sobrevivir una media de 3 años. Después del desarrollo de insuficiencia
ventricular izquierda, la esperanza de vida es ligeramente superior a 1 año.
13. En el ECG abajo muestra cuál de los siguientes ritmos?

A. Ritmo de escape ventricular.


B. Bloqueo cardiaco de primer grado
C. Bloqueo cardiaco de segundo grado
D. Bloqueo cardiaco de tercer grado

RESPUESTA: D

ECG con bloqueo cardíaco de tercer grado tiene las siguientes tres características: ondas P
presente con una tasa auricular regular más rápida que la tasa ventricular, complejos QRS
están presentes con una tasa ventricular lenta, las ondas P no guardan relación con los
complejos QRS y los intervalos PR son completamente variable porque las aurículas y
ventrículos están eléctricamente desconectado.

Bloqueo cardíaco de segundo grado se divide en Mobitz I y Mobitz II. Mobitz tipo I,
también llamado Wenckebach, resulta en el progresivo alargamiento del intervalo PR con
eventual caída de un complejo QRS. Mobitz II se caracteriza por la pérdida repentina de un
complejo QRS P-R sin alargamiento.

14. Un paciente de trauma en la unidad de cuidados intensivos quirúrgicos tienes los


siguientes resultado de gasometría:

pH 7.32 (Normal 7.36-7.44)


PaCO2 46mm Hg (Normal 36-44mm Hg)
HCO3 23 mEq/L (normal 22-26 mEq/L)

Estos hallazgos es indicativo de cuál condición?

A. Acidosis respiratoria.
B. Alcalosis Respiratoria.
C. Acidosis metabólica.
D. Alcalosis Metabólica.

RESPUESTA: A

Acidosis respiratoria se produce cuando su pH es inferior a 7.36 y PaCO2 es superior a 44


mm / Hg. Alcalosis respiratoria se produce cuando el pH es superior a 7.44 y PaCO2 está
por debajo de 36 mm / Hg. Acidosis metabólica tiene un pH inferior a 7.36 y HCO3 está
por debajo de 22 mm / Hg. Alcalosis metabólica tiene un pH superior a 7,44 y HCO3 es
superior a 26 mm / Hg.

15. Cuál de los siguientes es el factor de riesgo más importante para desarrollar una
neumonía nosocomial?

A. Malnutrición
B. Ventilación mecánica.
C. Residencia en una casa de reposo. (casa de ancianos)
D. Abuso del tabaco.

RESPUESTA: B

Si bien la malnutrición, el consumo de tabaco, y la residencia en una casa de reposo la


facilitan como factores de riesgo, la intubación endotraqueal y ventilación mecánica es por
mucho el más importante contribuyente a neumonía nosocomial. La intubación
endotraqueal proporciona una vía para la contaminación bacteriana en los pulmones.

Este está aumentado en pacientes con trauma torácico o lesión pulmonar. La aspiración de
las secreciones orales en las vías respiratorias superiores es común en la mayoría de los
casos de neumonía. Promedios de 1 billón de bacterias se encuentran en cada mililitro de
saliva.

16. Complicaciones crónicas de cuál proceso patológico incluye cirrosis del hígado,
disfunción pancreática, sinusitis e hiperreactvidad bronquial?

A. Bronquitis crónica.
B. Fibrosis quística.
C. Sarcoidosis.
D. Tuberculosis.

RESPUESTA: B

La fibrosis quística es un trastorno autosómico recesivo. En los Estados Unidos, afecta a 1


en 2500 Caucásicos, 1 de cada 17.000 afroamericanos, y 1 en 90.000 asiáticos. Más de 230
mutaciones han sido identificadas con distintos grados de severidad de la enfermedad, con
el defecto más común en el brazo largo del cromosoma 7. El defecto genético causa
defectos de transporte de cloro y el aumento de la reabsorción de sodio en las vías
respiratorias y epitelio ductal, creando secreciones espesas y viscosas anormales en tractos
respiratorias, hepatobiliar, gastrointestinal y reproductor. Las secreciones gruesas causan
obstrucción luminal y la destrucción de conductos exocrinos. La media de supervivencia es
de 40 años de edad.
Un criterio de diagnóstico es una concentración de cloruro de sudor > 60 mEq / L. Las
pruebas genéticas pueden además de completar confirmar el diagnóstico. Los hallazgos
clínicos son también importantes en diagnóstico. Presencia Pseudomonas aeruginosa moco
de en el esputo es una característica de la FQ. Radiografía de tórax con bronquiectasia
incluyen anormalidades del lóbulo superior derecho y predominio de lado derecho.
Neumotórax espontáneo y la hipertensión pulmonar pronóstico resultados son negativos.

Terapia estándar para la FQ incluye terapia física de pecho, antibióticos necesarios,


alimentación adecuada y el ejercicio físico. Los broncodilatadores pueden ser utilizados
para los pacientes con broncoespasmo o la limitación del flujo aéreo. Los trasplantes de
pulmón para los pacientes con fibrosis avanzada con se están llevando a cabo.

17. Cuál de los siguientes es comúnmente asociado con Asma?

A. Disminución de la conformidad pulmonar.


B. Sensibilidad colinérgica.
C. Enfermedad de reflujo gastroesofágico.
D. Hipocarbia.

RESPUESTA: C

Más del 80% de los asmáticos presentan enfermedad por reflujo gastroesofágico (ERGE).
Aunque enfermedades respiratorias pueden inducir reflujo gastroesofágico mediante el
aumento de los gradientes de presión abdominotorácico, hay una considerable evidencia
que sugiere que la vía más común es que para que el reflujo gastroesofágico desencadene
los síntomas respiratorios, especialmente del asma. Esto puede ocurrir de la
microaspiración directa con la consiguiente inflamación y broncoconstricción, efectos
mediados vagalmente de la estimulación de los receptores de las vías respiratorias
superiores, o de irritación esofágica aferente causando una reflejo neurogénico.

Disminución de la conformidad pulmonar se asocia con enfermedad pulmonar restrictiva


no obstructiva, y el asma es una enfermedad pulmonar obstructiva. Asmáticos comúnmente
han aumentado la sensibilidad colinérgica. La Hipercarbia secuelas de un ataque asmático.

18. Cuál de los siguientes es mejor caracterizado como enfermedad pulmonar


restrictiva?

A. Disminución de la capacidad pulmonar total


B. Disminución en FEV1/FVC (FEV1: volumen expiratorio forzada en 1sec,FVC:
capacidad vital funcional)
C. Incremento en capacidad vital
D. Incremento en resistencia de la vía aérea.

RESPUESTA: A
Trastornos restrictivos evaluados por la medición de volúmenes pulmonares. La gravedad
del defecto restrictivo se basa en el CPT (capacidad pulmonar total). En trastornos
pulmonares restrictivos el VEF1 puede disminuir, pero la relación FEV1/FVC se
mantendrá. La CPT, CRF (capacidad residual funcional) y VR (volumen residual) serán
disminuido. La resistencia de las vías respiratorias no se ve afectada en una enfermedad
pulmonar restrictiva.

19. Cuál es el diagnóstico más parecido para un paciente con los siguientes hallazgos
después de un test de función pulmonar?

FVC: normal
FEV1: disminuido
FEV1/FVC: disminuido

A. Debilidad de los músculos respiratorios


B. Enfermedad pulmonar obstructiva
C. Enfermedad pulmonar restrictiva
D. Neumotórax espontáneo

RESPUESTA: B

Enfermedad pulmonar obstructiva incluye asma, bronquitis crónica, enfisema,


bronquiectasias, fibrosis quística, y la obstrucción de las vías respiratorias superiores.
Enfermedades como el asma y la bronquitis, que obstruyen las vías respiratorias, reducir el
flujo espiratorio y, por tanto, reducir las tasas de FEV1 y FEV1/FVC. Un defecto
obstructivo es esencialmente caracterizado por una disminución desproporcionada de la
tasa de flujo de aire en relación con el volumen real exhalado (es decir, FVC) y el
Estrechamiento de las vías respiratorias y se indica la limitación de flujo durante
vencimiento. FEV1/FVC de valores que son inferiores a 70% reflejan la obstrucción leve,
los inferior al 60% sugieren obstrucción moderada, y los inferiores al 50% indican
obstrucción grave.
En la enfermedad pulmonar restrictiva y la debilidad muscular respiratoria, la capacidad
vital forzada y VEF1 están disminuidas, mientras que la tasa de FEV1/FVC es normal.
Neumotórax puede ser considero una enfermedad pulmonar restrictiva.

20. Cuál es considerado el diagnóstico más definitivo en el embolismo pulmonar?

A. Ultrasonido del sistema femoro-popliteo


B. Escaneo nuclear por ventilación-perfusión
C. Radiografía de tórax
D. TC contrastada

RESPUESTA: D
21. Cuál de los siguientes grupos farmacológicos es un factor de riesgo para el
tromboembolismo venoso?

A. Hipercolesterolémicos
B. Terapia de reemplazo de estrógenos
C. AINEs
D. Inhibidores de la ECA

RESPUESTA: B

Los estrógenos y píldoras anticonceptivas se pueden resultar en hipercoagulabilidad y son


un factor de riesgo para el tromboembolismo venoso. La embolia pulmonar por lo general
es una complicación de la trombosis venosa del sistema ilíaca poplíteo-pélvico-femoral de
la extremidad inferior. Factores que pueden contribuir a la trombosis son: estasis venosa,
hipercoagulabilidad, lesión endotelial. Paciente con factores de riesgo incluyen: la
insuficiencia cardíaca, neoplasia, inmovilización, traumatismos, la obesidad,
edad avanzada, el embarazo, cirugía extensa, y los anticonceptivos orales.
No hay datos que soporten los hipercolesterolémicos, AINEs o inhibidores de la ECA como
el factor de riesgo para el tromboembolismo venoso.
La prevención es el mejor tratamiento que incluye: dispositivo de compresión intermitente,
almacenamiento de medicamentos antiembólicos, uso selectivo de heparina, y
deambulación temprana post-operatorio.

22. Cuál es el primer curso de acción para pacientes que se sospeche fuertemente que
tiene una trombosis venosa profunda?

A. Warfarina
B. Trombectomia
C. Heparina
D. Estreptoquinasa

RESPUESTA: C

El tratamiento debería incluir la atención de cuidado para la hipoxemia e hipotensión.


Anticoagulación con heparina IV debe iniciarse lo más pronto se sospeche clínica de PE sin
tener que esperar a estudios de diagnóstico. El objetivo es de 1.5-2.5 veces el valor de
control del PTT para 5-10 días. Toda demora en llegar a rango terapéutico PTT ha
demostrado que aumenta el riesgo de trombosis recurrente progresiva y embolismo
pulmonar. Luego, la terapia con Warfarina es iniciada con un INR de 2-3 por 3-6 meses o
indefinidamente si el paciente es de alto riesgo. Si existen contraindicaciones para la
Anticoagulación o si el paciente ha reaparecido con PE, a pesar del tratamiento
anticoagulante de elección es la interrupción de la vena cava. (Greenfield filtro).
Terapia del Sistémica trombolítico y embolectomía podrán indicarse en casos muy raros.
Heparina (no fraccionada) es un glucosaminoglucano con variable tamaño molecular.
Potencia la acción de la antitrombina III así inactivando la trombina y evita la conversión
de fibrinógeno en fibrina.
Heparina de bajo peso molecular (HBPM) se ha utilizado en lugar de la no fraccionada con
resultados comparables con la heparina para el tratamiento de la TVP y no masivo PE.
HBPM es enzimáticamente desglosarse en moléculas más pequeñas más uniforme con
menos variación en contra la respuesta de coagulación. Debido a la previsibilidad de la
respuesta y la vía de administración de estos pacientes son, en algunos casos, tratados como
pacientes ambulatorios, a manera de reducir los costos, pero no cambia los resultados
clínicos. HBPM se administra por vía subcutánea con un inicio lento, pero luego la dosis de
heparina IV es menos frecuente, hay menos trombocitopenia inducida por heparina, menos
osteoporosis inducida por heparina, y un poco menos riesgo de hemorragia.

23. Un paciente de 60 años de edad reporta una historia de fumador y tos productiva
frecuente por lo menos 3 meses en los últimos 2 años. Cuál es el diagnóstico más
parecido?

A. Enfisema
B. Fibrosis quística
C. Asma
D. Bronquitis crónica

RESPUESTA: D

La bronquitis crónica se define como la presencia de una tos productiva durante al menos 3
meses en cada uno de los 2 últimos años en una persona con exceso de secreción mucosa
no debido a otra enfermedad. La causa más común es el tabaquismo. La bronquitis crónica
puede dar lugar a la retención de dióxido de carbono, hipoxemia crónica, eritrocitosis,
hipertensión pulmonar e insuficiencia cardiaca derecha. La unidad respiratoria se hace
menos sensible a la retención de dióxido de carbono y es posible estar deprimido por la
administración de oxígeno. (Síndrome Blue Bloater)

24. Cuál de las siguientes pruebas de función pulmonar es el más usado en la


evaluación del paciente asmático?

A. TLC (Capacidad pulmonar Total)


B. FEV 1 (Volumen espiratorio forzada en un segundo)
C. FRC (Capacidad residual funcional)
D. Capacidad vital

RESPUESTA: B

El VEF1 mide la cantidad de aire que pueden ser expulsados por la fuerza en 1 segundo y
refleja el grado de obstrucción de las vías aéreas grandes. Valores VEF1 son normalmente
de alrededor de 3L para hombres y 2L para mujeres, pero varían con cada individuo.
FEV1/FVC deberá ser normalmente de > 70% y una tasa de VEF1 o FEV1/FRC menor al
50% indica asma moderada a grave. TLC y FRC puede ser elevado en la enfermedad
obstructiva. Enfermedad obstructiva (por ejemplo como el asma) se caracteriza por las
dificultades de vencimiento. O bien se requiere más fuerza a expirar un determinado
volumen de aire, o el vaciado de los pulmones es más lento, o ambos. Espirometría
(Pruebas de función pulmonar), tanto las medidas de volumen pulmonar y del flujo de gas.
La función pulmonar las pruebas a menudo demostrar reversibilidad parcial (mejora en el
VEF1 de al menos 15%) después de la administración de broncodilatadores. La ausencia de
mejora en las pruebas de función pulmonar después de la administración de un
broncodilatador no constituye la prueba de la obstrucción irreversible de las vías
respiratorias.

25. Cuál de los siguientes medicamentos debería ser evitada en un paciente con historia
de poliposis y asma moderada?

A. Acetaminofen
B. Tramadol
C. Propoxifeno
D. Diclofenaco

RESPUESTA: DICLOFENACO

Asma inducida por aspirina (AIA) se caracteriza por episodios de asma grave y a veces con
riesgo para la vida debido al uso de AINEs (antiinflamatorios no esteroideos).
Los individuos con AIP sufren de asma crónica que es a menudo dependiente de esteroides
y asociados con un número significativo de personas con rinosinusitis crónica y pólipos
nasales (Sampter la tríada). AIP generalmente aparece en la 3 ª a 4 ª década de la vida.
Los pacientes con AIP pueden reaccionar a los AINEs que inhiben la enzima
ciclooxigenasa 1 (COX-1). COX-1 cataliza la formación de prostaglandinas y tromboxanos
de la membrana celular ácido araquidónico. Si la COX-1 es inhibida, puede causar un
cambio hacia una vía alternativa al metabolismo del ácido araquidónico envolviendo a la
lipoxigenasa 5. Los productos de lipoxigenasa 5 que incluyen diversos leucotrienos son
potentes mediadores de la inflamación. Estos leucotrienos pueden actuar como
bronchoconstrictores. Estos leucotrienos también puede aumentar la secreción de moco,
inducir la inflamación de la mucosa nasal, edema de las vías respiratorias promover y atraer
a los eosinófilos a las vías respiratorias. El aumento de las concentraciones de los
leucotrienos ha sido detectado en pacientes AIP en comparación con los pacientes no AIP
tanto en la línea de base y después de desafío ASA (aspirina).

26. Un paciente asmático comienza a quejarse de disnea severa y tos. Auscultación de


los pulmones revela sibilancias. Cuál de las siguientes clases de drogas refleja el
tratamiento inicial preferido?

A. Xantines
B. Adrenérgicos
C. Anticolinérgicos
D. Corticoesteroides

RESPUESTA: B

Agentes Beta adrenérgicos son los preferidos para la medicación de rescate inicial de crisis
de broncoespasmo. El agente ideal beta adrenérgicos beta 2 poseería actividad selectiva
pura sin efectos cardiacos secundarios. Las catecolaminas broncodilatadoras viejas como la
epinefrina e isoproterenol beta 2 no son específicos y tienen una corta duración de acción.
Epinefrina (1:1000 o 1mg/ml 0.3mg-SC 0,5 mg cada 20 minutos por 3 dosis) pueden ser
administrados a los pacientes incapaces de coordinar los tratamientos en aerosol o MDI.
Agentes más recientes, como el resorcinol broncodilatadores (metaproterenol, terbutalina y
fenoterol) y los saligénicos broncodilatadores (albuterol y carbuterol) tienen una mayor
especificidad beta 2 y una mayor duración de acción. Albuterol es un inhalados de acción
corta beta 2 agonista que puede ser administrado como una nebulizada
o como una solución MDI (inhalador dosificado). Albuterol inhalado podría darse en
90ug/puff con 4o-8puff cada 20 minutos hasta 4 horas para el paciente adulto. Xantinas
incluyen medicamentos como la teofilina. Ellos no se consideran un tratamiento de primera
línea para asma aguda. Xantinas son a menudo reservadas para aquellos pacientes cuyos
síntomas persisten a pesar del uso de un esteroide inhalado y antagonista beta 2. Niveles
terapéuticos son a menudo asociados a efectos secundarios gastrointestinales, cardiacos y
del sistema nervioso central. Hay un estrecho índice terapéutico, la toxicidad puede ocurrir
y ser fatal. Anticolinérgicos, tales como atropina o bromuro de ipratropio son los
broncodilatadores, pero su uso puede producir efectos secundarios tales como la
taquicardia. Los Corticoesteroides como la hidrocortisona y betametasona son
muy eficaces en el tratamiento del asma crónica por reducción de la inflamación de la de la
mucosa del lumen, y pueden restaurar algunos beta adrenérgicos de respuesta, pero la
aparición de efecto puede demorar de 4-8 horas.

27. Otra de sonidos

28. Paciente masculino de 25 años es llevado bajo anestesia general para tratamiento de
fractura de mandíbula. El anestesiólogo nota dificultad en la ventilación del paciente
y obtiene esta radiografía. Que tratamiento es apropiado para la condición de este
paciente?

A. Colocar una aguja en lado derecho


B. Tratamiento nebulizador con albuterol
C. Antibióticos
D. Colocación de tubo torácico en el lado izquierdo

RESPUESTA: A

Un neumotórax a tensión es la acumulación de aire en el espacio pleural, creando presión


positiva. Porque la tensión del neumotórax puede causar compromiso hemodinámico, esta
emergencia médica requiere inmediata descompresión. Además de la resistencia a la
la ventilación, la presión arterial disminuye y aumenta la presión venosa central. El
aumento de la presión debe ser relevado inmediatamente con aguja de toracotomía, que es
realiza mediante la inserción de una aguja de gran diámetro (calibre 16 o 18) a través del
segundo o tercer espacio intercostal anterior en el lado envuelto. Un tubo torácico es
también un tratamiento adecuado para el neumotórax a tensión, sin embargo, en este caso
sería en el lado equivocado de la paciente.
29. Un paciente femenino de 32 años de edad, presenta refiriendo debilidad al masticar.
Sus síntomas están empeorando durante las comidas de la tarde. Examen físico
muestra diplopía y ptosis palpebral. Si sus signos clínicos se mejoran con la
administración de edrofonio cuál de los siguientes usted sospecharía?

A. Mutación en el gen distrofino


B. Anticuerpos para los canales de calcio presinápticos de voltaje
C. Anticuerpos para proteínas sarcoglicanos
D. Anticuerpos en contra de receptor postsináptico de nicotina y acetilcolina

RESPUESTA: D

El paciente está demostrando hallazgos clásicos relacionados con la miastenia grave. Esta
enfermedad es el resultado de autoanticuerpos a los receptores postsinápticos nicotínicos de
la acetilcolina. Su prevalencia es de 1 en 7500 y, aunque afecta a todos los grupos de edad,
es más común en los hombres en sus cincuenta y sesenta, y las mujeres en sus años veinte y
treinta. Características cardinales incluyen la debilidad y la fatiga de los músculos que es
progresiva y puede mejorar con el descanso. Debilidad muscular craneal usualmente
involucra los párpados y los músculos extraoculares resultado en lo ptosis y diplopía.
Debilidad al masticar puede resultar después de un prolongado esfuerzo, y nasal, así como
dificultad al tragar también pueden el resultado de debilidad de músculos palatinos y
faríngeos. Los reflejos tendinosos profundos se conservan. Ocasionalmente, los músculos
respiratorios pueden desarrollar debilidad que requieren asistencia ventilatoria, lo que se
conoce como una crisis miasténica. Respuesta A está asociado con distrofia muscular de
Duchenne. B representa la fisiopatología de Síndrome Miasténico de Lambert-Eaton. La C
es la etiología de la distrofia muscular de Limb-Girdle

30. Caballero de 67 años se presenta a tu consultorio con xerostomía y


queratoconjuntivitis seca. El tiene una historia de fumador y diagnosticado
recientemente de cáncer de pulmón. Usted nota ptosis palpebral intermitente y
disminución en los reflejos tendinosos que mejoran brevemente con el ejercicio,
pero actividad sostenida empeora. Que desorden usted sospecharía?

A. Hipertiroidismo
B. Miastenia Grave
C. Distrofia óculo faríngea
D. Síndrome miasténico de lambert-Eaton

RESPUESTA: D

Síndrome miasténico de Lambert-Eaton es un trastorno de la unión neuromuscular


presináptica. Que causa los síntomas que se asemejan a los de la miastenia grave. Sin
embargo, normalmente es asociado con una malignidad más común de cáncer de pequeñas
células de pulmón. Se piensa que es causado por autoanticuerpos a los canales de calcio en
los nervios motores terminales, lo que dificulta la liberación de acetilcolina. Tratamiento de
la enfermedad supone inmunosupresión a menudo con esteroides, así como la
plasmaféresis, que a menudo se realizado preoperatoriamente, junto con la cesación de los
medicamentos de anticolinesterasa. Un diagnóstico diferencial incluye: el hipertiroidismo,
el botulismo, lesiones intracraneales, neurastenia, y oftalmoplejía externa progresiva.

31. Masculino de 5 años de edad se presenta a tu consultorio con su mamá. En la silla


dental, el muchacho presiona sus manos en contra de sus rodillas para elevar su
dorso superior en una posición derecha. Usted también nota que los músculos de su
pantorrilla son bastante más grandes en comparación con sus muslos y que
intelectualmente está afectado. Cuál es el proceso patológico más parecido que
afecta a esta joven persona?

A. Miastenia grave
B. Dermatomiosistis
C. Polimiositis
D. Distrofia muscular de Duchenne

RESPUESTA: D

El niño lleva a cabo una clásica maniobra de Gowers elevandose a sí mismo a una
posición erecta, que es la mejor vía para el diagnóstico de distrofia muscular de Duchenne.
Esta es una Miopatía hereditaria que se conoce también como la distrofia muscular
pseudohipertrófica. Se trata de un trastorno recesivo ligado al cromosoma X con una
incidencia de 30 por 100.000 hombres. Mientras que está presente al nacimiento, por lo
general se hace evidente entre las edades de 3 y 5. Los niños tienen dificultad continuar con
las actividades físicas con sus compañeros, y tienen una debilidad muscular progresiva. El
alargamiento de las pantorrillas es clásico también. También sufren de deterioro intelectual
y tienen un IQ por debajo del estándar.

32. Que hallazgos es comúnmente asociado a síndrome de Sturge-Weber?

A. Convulsiones
B. Pits labiales
C. Machas Café-au-lait
D. Hipoplasia mandibular pronunciada

RESPUESTA: A

El clásico de las manifestaciones clínicas de síndrome de Sturge-Weber son nevus


vasculares faciales (mancha en vino de Oporto), epilepsia, los déficit cognitivos, y
hemiparesia o hemiplejia, hemianopia, glaucoma. Los pacientes con síndrome de Sturge-
Weber puede tener la alargamiento asociados a labios, maxilar o mandíbula.
Anormalidades leptomeningeas pueden causar convulsiones, hemiplejia contralateral y
retraso motor.
33. Paciente de 24 años de edad con fractura panfacial aguda desarrolla dificultad al
hablar y confusión. Usted ha sido llamado a la UCI para examinarlo por una
fractura maxilar. Mientras habla con el paciente, el agudamente desarrolla dificultad
al hablar a pesar de estar consciente. Sus signos vitales permanecen estables y se
recupera por completo en algunos minutos. Cuál es la explicación más exacta para
la dificultad al habla y confusión?

A. Ataque isquémico transitorio


B. Hipoglicemia
C. Convulsiones
D. Narcolepsia

RESPUESTA: C

Convulsiones postraumáticas pueden ocurrir como resultado agudo de un trauma craneal


penetrante o romo. Convulsiones inmediatas post- traumáticas ocurren en las 24 horas del
Accidente y son más comunes en niños que en adultos.

34. Paciente femenino de 25 años de edad con esclerosis múltiple es llevada a cirugía en
la sala de operaciones bajo anestesia general. Cuál de los siguientes drogas
bloqueadoras neuromusculares está contraindicado en este individuo?

A. Vercuronio
B. Cisatracurio
C. Rocuronio
D. Succinilcolina

RESPUESTA: D

Drogas de bloqueo neuromuscular deberían ser usadas con juicio. Succinilcolina posee el
riesgo de hiperkalemia debido a la liberación intracelular de potasio. Hiperkalemia
representa un riesgo para pacientes con patología de denervación muscular y puede dirigir a
un arresto cardiaco.

35. El diagnóstico definitivo de enfermedad de Alzheimer puede solo ser hecho por
cuál de las siguientes pruebas y exámenes?

A. Examen Psiquiátrico
B. Examen neurológico
C. Estudios imagenológicos (CT, MRI y PET)
D. Examen del cerebro por autopsia

RESPUESTA: D
La enfermedad de Alzheimer es insidioso en su aparición, muy gradual en su progresión y,
en última instancia es fatal. Se manifiesta como cambios en la memoria y el diario
funcionamiento y se caracteriza por la amnesia anterógrada, deterioro cognitivo y la
demencia que conducen a la incapacidad del paciente para participar en el auto-cuidado,
alteraciones en el habla, y anormalidades en la deglución. Siete etapas de la progresión de
la enfermedad se han desarrollado para definir mejor la naturaleza de la progresión de
deterioro cognitivo. Un diagnóstico clínico presuntivo de la enfermedad de Alzheimer se
obtiene mediante la identificación de los síntomas clínicos del paciente y compararlos con
un conjunto de criterios conocidos. Los pacientes generalmente se someten a exámenes
físicos, neurológicos y psiquiátricos. Se hace un intento de excluir enfermedades tratables
como la enfermedad de Parkinson, la toxicidad de drogas, enfermedades metabólicas, la
deficiencia de la dieta, accidentes cerebrovasculares, enfermedades infecciosas y CNS (por
ejemplo, la sífilis o el VIH). Común herramientas de diagnóstico incluyen Mini Mental
State Examination, CBC, los electrolitos séricos, pruebas de función hepática, el colesterol,
los lípidos séricos, la vitamina B12 y ácido fólico, pruebas de función tiroidea, análisis de
orina, radiografía de tórax, ECG, EEG, estudios de imagen y CNS. Aunque cada una de
estas pruebas cuando se toma como un todo, puede llevar la identificación de la enfermedad
de Alzheimer a través de un proceso de eliminación, el diagnóstico sólo puede hacerse con
certeza en el momento de la muerte del paciente cuando el tejido cerebral se examina para
ver placas celulares intra-ß-amiloide, ovillos neurofibrilares ß-extracelular amiloide y
intracelulares de la proteína “tau” en la corteza entorrinal, hipocampo, áreas relacionadas
con la memoria, aprendizaje, lenguaje y comportamiento emocional.

36. Masculino de 65 años llevado bajo anestesia general para reparar una fractura
mandibular hace un día. Su familia nota debilidad en el brazo derecho y calumnia al
hablar por los últimos 30 minutos. Cuál de los siguientes estudios debería
inicialmente hacerse?

A. Punción lumbar
B. Arteriograma carotideo
C. Doppler carotideo
D. Tomografía no contrastada de cabeza.

RESPUESTA: D

Imágenes del cerebro sigue siendo un componente necesario de la evaluación de


emergencia de los pacientes con sospecha de accidente cerebrovascular. Tanto la TC y la
RM son las opciones para la obtención de imágenes del cerebro, pero para la mayoría de los
casos y en la mayoría de las instituciones, la TC sigue siendo la forma más práctica de
prueba inicial de imagen cerebral. TC de cabeza determinará si la causa de la debilidad del
brazo y trastornos del habla es un ACV y si es isquémico o hemorrágico. Los accidentes
cerebrovasculares isquémicos constituyen aproximadamente el 80% de los accidentes
cerebrovasculares. El tratamiento trombolítico está iniciado dentro de las 3 horas del evento
disminuye largo plazo la discapacidad de la persona.
37. Un paciente de 32 años de edad con artritis reumatoide se presenta para tratamiento
de pericoronitis de un 48 parcialmente erupcionado. El paciente toma metotrexato.
Qué medicación debería ser evitada en el manejo postoperatorio de este individuo?

A. Hidrocodona
B. Codeína
C. Tylenol
D. Ibuprofeno

RESPUESTA: D

El metotrexato se usa para el tratamiento de la artritis reumatoide. El mecanismo de acción


de metotrexato es que es un antimetabolito ácido fólico que inhibe la síntesis del ADN.
El metotrexato se une irreversiblemente a la reductasa dihidrofolada, lo que resulta en la
inhibición de las purinas timidilicas y la síntesis de ácidos.
El metotrexato tiene efectos secundarios que incluyen la supresión de la médula ósea,
anemia aplásica, y toxicidad GI con la administración concomitante de AINEs. Penicilinas
puede aumentar las concentraciones de metotrexato, debido a una reducción de la secreción
tubular renal. Tylenol y los opiáceos pueden ser administrados a este paciente.
Graves, a veces mortales, la toxicidad (incluyendo toxicidad hematológica y GI) se ha
producido tras la administración de un AINE (por ejemplo, indometacina, ketoprofeno)
concomitantemente con metotrexato (en particular con altas dosis de la terapia) en
pacientes con diversas neoplasias malignas, psoriasis, artritis reumatoide.
AINES deber ser evitado en pacientes que están tomando altas dosis de metotrexato.

38. Examen ocular de un paciente de trauma en el departamento de emergencias revela


una pupila esta fija y dilatada, con el mismo ojo desviado lateralmente y hacia abajo
con ptosis del párpado. Cuál de los siguientes nervios craneales está dañado?

A. Óptico (II)
B. Oculomotor (III)
C. Troclear (IV)
D. Abducen (VI)

RESPUESTA: B

El nervio Oculomotor se ha lesionado en este caso. Este nervio suministra la función


motora todos los músculos extraoculares, excepto el recto lateral (suministrado por el
nervio abducens) y el oblicuo superior (suministrado por el nervio troclear). Por lo tanto el
único nervio funcionamiento y que han causado una mirada hacia abajo y lateral en el ojo
afectado. El nervio oculomotor suministra también la función motora al elevador del
párpado superior. La pérdida de esta función ha causado de la ptosis palpebral. Fibras
parasimpáticas transportadas por el nervio craneal III del ganglio al ciliar esfínter pupilar
causando la constricción pupilar. Daño a estas fibras causas de dilatación de la pupila
afectada.
39
¿El mecanismo de acción del diazepam (valium) cuando se trata el estado epiléptico
ocurre vía una inhibición mediada por GABA de que ion?

A. Calcio
B. Potasio
C. Sodio
D. Cloro
Respuesta: D
Racional:
Durante las convulsiones, las concentraciones de potasio extracelular y calcio
intracelular aumentan y contribuyen a la excitabilidad aumentada del agregado neuronal
epiléptico. Durante la convulsión propiamente, las neuronas son tónicamente
despolarizadas y disparadas continuamente en una sostenida, de alta frecuencia descarga
(correspondiendo a la fase tónica de una convulsión). La convulsión termina cuando re
polarizaciones fásicas interrumpen el patrón de disparo continuo (la correlación celular
de la fase clónica) y gradualmente se restaura el potencial de acción de la membrana a lo
normal o a un estado temporal hiperpolarizado, (depresión postictal). Las
benzodiacepinas y los barbitúricos ejercen su efecto mediante el aumento postsináptico
inhibitorio del acido gama amino butírico a través de un efecto en los ionoforos de cloro.
La fenitoina son anticonvulsivantes efectivos porque producen un bloqueo uso
dependiente de los canales de sodio, limitando la capacidad de las neuronas para
disparar, a una alta frecuencia.
Referencia:
Goldman: Cecil Textbook of Medicine, 22nd ed.; 2004, WB Saunders, Chapter 426,
Status
Epilepticus
Drug Information Handbook for Dentistry, 12th ed. Wynn, Meiller, Crossley. Lexi-
comp,
2006, p. 455.

40 El síndrome de Horner resulta del daño a ¿cual fibra nerviosa?


A. Fibras simpáticas a la pupila
B. Fibras parasimpáticas a la pupila
C. Nervio óptico
D. Nervio oculomotor
Respuesta: A
Racional:
Una pequeña pupila unilateral es común debido a una menor actividad de las vías
simpáticas ipsilaterales. Miosis es comúnmente asociada con ptosis debido a
denervación simpático del musculo tarsal y anhidrosis facial. Esta combinación es
conocida como síndrome de Horner. Y puede deberse a una lesión del hipotálamo y tallo
cerebral, médula espinal, o fibras simpáticas de la pupila. El síndrome de Horner puede
ser el primer signo de un tumor apical pulmonar (tumor de Pancoast) o puede ocurrir en
enfermedades afectando la arteria carótida. El tumor de Pancoast causa compresión de la
médula espinal a nivel C8 y T. Las fibras simpáticas al ganglio cervical superior se
originan a nivel de T1
Referencia:
Andeoli, Carpenter, et al. Cecil Essentials of Medicine, 7th ed. Saunders Elsevier,
Philadelphia, 2007 p. 1080
Goetz: Textbook of Clinical Neurology, 2nd ed. Saunders Elsevier 2003, Chapter 21 .
Autonomic nervous system < Pupils < Horner’s syndrome, and Chapter 15 . Strength
and
Reflexes > Lower Motor Neuron Syndromes

41¿Cual de los siguientes son factores de la coagulación dependientes de vitamina K?


A. III, VII, IX, X
B. II, VII, IX, X
C. III, VIII, IX, XII
D. II, VIII, IX, XII
Respuesta: B
Racional:
Hay 5 factores producidos en el hígado: II, V, VII, IX, y X. De estos, cuatro son
dependientes de vitamina K, II, VII, IX, and X. La Vitamina K se requiere para la
gama-carboxilación de los factores. Virtualmente todos los factores son producidos en el
hígado. El Factor VIII es hecho en varios tejidos, incluyendo el hígado, las células
epiteliales del glomérulo y túbulo en el riñón. La Warfarina interfiere con la síntesis
hepática de los factores dependientes de vitamina K. El PT/INR son ambos medidas de
la vía extrínseca en la cascada de la coagulación.
Referencia:
Abubaker AO: Oral and Maxillofacial Surgery Secrets. Hanley & Belfus, Inc.,
Philadelphia, PA, 2001; Page 130
Andeoli, Carpenter, et al. Cecil Essentials of Medicine, 7th ed. Saunders Elsevier,
Philadelphia, 2007 p. 532-538
Tierney LM: 2005 Curent Medical Diagnosis and Treatment. McGraw-Hill, Inc, New
York, New York; 2005; Page 656

42¿Cuál es la etiología de la trobocitopenia asociada a enfermedad enfermedad


hepática?
A. Síntesis reducida de urea
B. Falla renal funcional
C. Esplenomegalia
D. Ascitis
Respuesta: C
Racional:
La hipertensión portal es una secuela común de falla hepática crónica. Una congestión
vascular dentro del sistema portal lleva a una esplenomegalia. Un vaso agrandado puede
secuestrar eritrocitos y plaquetas. El secuestro de las plaquetas por el bazo llevará a una
trombocitopenia.
La síntesis reducida de urea no lleva a una trombocitopenia. Una falla renal, también
conocida como síndrome hepatorenal es una complicación tardía de la cirrosis. Ocurre
en presencia de fallo hepático sin anormalidades intrínsecas renales. La Ascitis es una
acumulación de exceso de fluido en el abdomen y es causada por cirrosis en al menos
80% de los pacientes.
Las Complicaciones of ascitis incluyen peritonitis bacterial espontanea, hidrotórax y
hernias de la pared abdominal.
Reference:
Oral & Maxillofacial Surgery Knowledge Update. Vol 1, Part II, 1995, Patient
Evaluation,
p. 39.
Mercer KW, Gail Macik B, Williams ME. Hematologic disorders in critically ill
patients.
Semin Respir Crit Care Med. 2006 Jun; 27:286-96.
Goldman: Cecil Textbook of Medicine, 22nd ed, Chapter 157, Cirrhosis and its sequelae
<
Complications of Cirrhosis.

43¿Cuál de los siguientes es un componente de la clasificación modificada de Child-


Pugh?
A. Albumina
B. Lactato deshidrogenasa
C. TPT
D. Amonio
Answer: A
Rationale:
La clasificación de Child-Pugh modificada fue diseñada para indicar la severidad de la
cirrosis en pacientes con enfermedad hepática crónica. Los componentes de la
clasificación incluyen: albumina, bilirrubina, PT, INR, ascitis, y encefalopatía. El TPT
no se incluye como componente de la clasificación de Child-Pugh. TPT es usado para
medir la vía intrínseca. El Pt es usado para medir la vía extrínseca. El Amonio es
descodificado en urea por el hígado. Niveles aumentados de amonio se piensa
contribuyen a la encefalopatía hepática. Los niveles de Lactato deshidrogenasa pueden
ser elevados en hepatitis.
Sistema modificado de puntaje de Child-Pugh
Puntaje de Child-Pugh modificado *
Parámetros 1 2 3
Albumina (g/dL) >3.5 1.8.3.5 <2.8
TP
Segundos prolongado <4 4.6 >6
INR <1.7 1.7.2.3 >2.3
Bilirrubina (mg/dL). <2 2.3 >3
Ascitis Ausente, leve, moderada o tensa
Encefalopatía Ninguna, Grado I.II Grado III.IV
From Pugh RNH, Murray-Lyon IM, Dawson JL, et al: Transection of oesophagus for
bleeding of oesophageal varices. Br J Surg 60:646.649, 1973.
* Class A = 5 to 6 points, B = 7 to 9 points, and C = 10 to 15 points.
Referencia:
Martins A, Cortez-Pinto H, Marques-Vidal P, Mendes N, et al. Treatment and
prognostic
factors in patients with hepatocellular carcinoma. Liver Int. 2006 Aug; 26(6):680-7.
Miller: Miller’s Anesthesia, 6th ed, Chapter 55 . Anesthesia and the Hepatobiliary
System
< Cirrhosis as a perioperative risk factor.

44 Detección de solo antígeno de superficie de Hepatitis B (Anti-HBsAg) en el suero es


asociado con ¿cuál de los siguientes?
A. Infección crónica
B. Infección aguda
C. Inmunidad
D. Respuesta celular inicial
Respuesta: C
Racional:
Los antígenos de superficial de Hepatitis B codifican el envoltorio viral: EL Anti-
HBsAg es el anticuerpo formado contra el antígeno y su presencia confiere inmunidad
cuando la titulación es mayor a 10mlU/ml.

Referencia:
Oral and Maxillofacial Surgery Clinics of North America, Volume 18, Number 2, May
2006, p. 217-219
Shepard CW, Simard EP, Finelli L, Fiore AE, Bell BP. Hepatitis B virus infection:
epidemiology and vaccination. Epidemiol Rev. 2006; 28(1):112-25. 2006

45 La primera linea de tratamiento para una ulcera péptica puede ser:


A. Bloqueadores H2 y antiácidos
B. protectores mucosos y antibióticos.
C. inhibidor de la bomba de protones y bloqueadores H2.
D. inhibidor de la bomba de protones y antibióticos.
Respuesta: D
Racional:
Las ulceras pépticas son llamadas primarias cuando ocurren sin ningún factor
predisponente como una enfermedad aguda, trauma, uso de AINES, alcohol, o fumado.
El Helicobacter pylori es reconocido como un importante factor en la patogénesis de la
enfermedad de ulcera péptica primaria.
Los inhibidores de la secreción ácida y agentes antimicrobianos son recomendados. Esta
terapia de combinación es la primera linea de tratamiento debido a que las drogas anti
secreción proveen alivio rápido de dolor y acelerado sanado; mientras los antibióticos
erradican el Helicobacter pylori.
En los Estados Unidos, posiblemente 80% de los pacientes que tienen ulceras
duodenales están infectados por el organismo, y más del 60% de ellos tiene ulceras
gástricas.
Referencia:
Perioperative Management of the Oral and Maxillofacial Surgery Patient, Part II, Oral
and
Maxillofacial Surgery Clinics of North America, Vol. 18, No. 2, May 2006,
pp. 245-246
Dental Management of the Medically Compromised Patient, 6th Ed., 2002, Little, J.W.,
Falace, D.A., Miller, C.S., Rhodus, N.L., pp. 192-193
Feldman: Sleisenger & Fordtran’s Gastrointestinal and Liver Disease, 8th ed., Saunders,
2006, p.1094.

46 La metaplasia de Barret es una complicación asociada con ¿cuál de las siguientes?


A. Colitis crónica ulcerativa
B. Ulcera péptica
C. Reflujo gastroesofágico
D. Enfermedad de Chron
Respuesta: C
Racional:
El esófago de Barrett no es poco común, y se encuentra en aproximadamente 6% a 12%
de los pacientes que sufren de endoscopia para diagnóstico de reflujo gastroesofágico y
en 1% a 2% de pacientes no seleccionados que reciben endoscopía.
El esófago de Barrett ocurre cuando el epitelio escamoso del esófago es remplazado por
epitelio metaplásico columnar. Se desarrolla debido a una lesión crónica producida por
el reflujo crónico hacia el epitelio escamoso del esófago. Estos pacientes frecuentemente
tienen un reflujo gastroesofágico severo, usualmente con síntomas de reflujo crónico por
más de 10 años, pobre movilidad esofágica, hernias hiatales grandes, y extenso reflujo
de ácido y bilis. Los pacientes con segmentos largos de esófago de Barrett tienen un
estimado de 30 a 125 veces más riesgo de desarrollar cáncer esofágico comparado con la
población general. Datos epidemiológicos sugieren que el intervalo medio de desarrollar
esófago de Barrett a la evolución de cáncer puede ser de 20 a 30 años.
El apropiado intervalo de vigilancia para pacientes con esófago de Barrett no ha sido
estudiado prospectivamente. No obstante, recientes programas, como aquellos
propuestos por el colegio Americano de Gastroenterología se basan en el grado de
displasia. Individuos con alto grado de dysplasia pueden recibir endoscopías cada 3
meses, mientras que aquellos con bajo grado de displasia pueden ser seguidos
anualmente.
Referencia:
Current Medical Diagnosis and Treatment, 42nd Ed., 2003, Tierney, L.M., McPhee,
S.J.,
Papadakis, M.A., pp. 552-553
Feldman: Sleisenger and Fordtran’s Gastrointestinal and Liver Disease, 8th ed.,
Saunders,
2006, 921-929.

47 ¿Cuál de los siguientes diferencia la diabetes tipo I de la tipo IIs?


A. No ocurre Cetoacidosis en DM tipo II
B. Hay mayor relación familiar con la DM tipo I que con la DM tipo II
C. El uso de insulin esta sólo indicado en la DM tipo I
D. La neuropatía diabetic es una complicación primaria de la DM II
Respuesta: A
Racionale:
Ya que la producción de insulin continua en individuos con DM tipo II, la cetoacidosis
no ocurre aunque se necesite de insulin para finalmente manejar la hiperglicemia. Las
tendencias genéticas hacia desarrollar diabetes estan asociadas con ambos tipos, aunque
la relación es más fuerte con la DM tipo II donde hay una tasa de concordancia mayor al
90% entre gemelos idénticos. Aquellos con DM tipo II frecuentemente progresan hacia
la necesidad de control de la hiperglicemia con insulina exógena. La neuropatía diabetic
es una frecuente complicación de ambos tipos de DM.
La cetoacidosis diabetic es un sindrome en el cual la deficiencia de insulin y glucagon se
combina en exceso para producer un hiperglicémico, deshitrado, acidótico paciente con
un profundo desequilibrio electrolítico. La glucosa en los túbulos renales extrae agua,
sodio, potasio, magnesio, calcio, fosforo, y otros iones de la circulación hacia la orina.
Esta diuresis osmótica combinada con la probre injesta y vómito produce profunda
deshidratación y desequilibio electrolítico asociado con la cetoacidosis diabética. In
pacientes con DM tipo II, el paciente desarrolla un estado hiperosmolar no cetónico.
La disminuida acción de la insulina resulta en glicogenolsis, gluneogénesis, y menor
toma periférica de la glucosa. La hiperglicemia atrae fluido del espacio intracelular
dentro del espacio extracelular, momentáneamente manteniendo una adecuada
perfusión. Pronto de todas maneras, este fluido es perdido en una profunda dieresis
osmótica, limitada finalmente por hipotensión y una subsecuente caída en la tasa de
filtración glomerular. La orina es extremadamente hipotónica, con una concentración de
sodio en orina de entre 50 y 70 mEq/L , comparada con 140 mEq/L en fluido
extracelular. Esta diuresis hipotónica produce profunda deshidratación, llevando a
hiperglicemia, hipernatremia, e hipertonisidad asociada.
Referencias:
Baker, Maurer, Wauman; Perioperative management of diabetes mellitus, Oral and
Maxillofacial Clinics of North America: 10:3:363-371; W.B.Saunders, Philadelphia,
1998
Powers: C.323, Diabetes mellitus, Harrison’s Principles of Internal Medicine, 16th Edit.,
p.2169-2185; McGraw-Hill, New York, 2005
Marx: Rosen’s Emergency Medicine: Concepts and Clinical Practice, 6th ed., 2006
Mosby,
Inc., p. 1962-1967.

48 El sindrome de Cushing es más común debido a ¿cuál de los siguientes?


A. un adenoma pituitario funcionando
B. carcinoma adrenal
C. administración exógena de esteroides
D. hyperplasia adrenal
Respuesta: A
Racional:
El sindrome de Cushing se debe a un exceso de glucocorticoides crónico endogeno o
exogeno.
La causa más común es la de un adenoma pituitario funcionando (llamado enfermedad
de Cushing). Aproximadamente 80% de los casos dependientes de ACTH son debidos a
esta etiología. El síndrome de Cushing resultante de fuentes endógenas puede ser
dependiente o independiente de ACTH. El síndrome de ACTH ectópica cuenta por cerca
del 10% de los casos. Ejemplos de fuentes endógenas independientes de ACTH:
adenoma pituitario, carcinoma pulmonar de células pequeñas, carcinoma bronquial. Se
puede deber también a uso iatrogénico exógeno de esteroides. Signos y síntomas de
síndrome de Cushing incluyen: obesidad centrípeta, disfunción reproductiva,
abnormalidades psiquiátricas, cambios osteoporóticos que lleven a fractura patológica,
adelgazamiento de la piel, miopatía y prupuras, hipertensión, intolerancia a la glucosa
con diabetes, desequilibrio electrolítico, retención de agua, presión intraocular
aumentada y exoftalmos.
Referencias:
Andreoli T: Cecil Essentials of Medicine., Mosby, Philadelphia, 2001 pp569-570
Henderson K, Baranski T, Bickel P: The Washington Manual Endocrinology
Subspecialty
Consult., Lippincott Williams & Wilkins, Philadelphia, 2005 pp76-77
Larsen: Williams Textbook of Endocrinology, 10th ed. 2003, p. 508-525.

49 Los hallazgos al examen físico de un paciente con enfermedad de Addison sin gratar
incluyen:
A. estrias en piel.
B. hiperpigmentación de la mucosa oral y la piel
C. obesidad centrípeta.
D. acne
Respuesta: B
Racional:
La insuficiencia adrenocortical primaria es conocida como enfermedad de Addison. La
causa es más frecuente el resultado de la destrucción atuopinmune de las glandular
adrenales. La perdida de secreción de ACTH vista en la insuficiciencia adrenocortical
secundaria es debida a suppression pituitaria resultado de un uso a largo plazo de
glucocorticoides. Las estrias en piel , la obesidad centrípeta y el acné en el adulto son
vistas en pacientes diagnosticados con el síndrome de Cushing ( exceso adrenocortical).
La hiperpigmentación de la mucosa oral y la piel es vista en pacientes con enfermedad
de Adisson debido a que la ACTH continua siendo producida por la glandula pituitaria
funcionando normal: ACTH activa directamente los receptores de melanocortina
resultando en pigmentación de piel y mucosa. Otros signos y síntomas frecuentemente
vistos en pacientes con insuficiciencia adrenocortical (primaria o secundaria) incluyen;
nausea, vomito, anorexia, perdida de peso, y fatigua.
La crisis adrenocortical se caracteriza: deshidratación, hipotensión o shock,
nausea/vomito, dolor abdominal severo, hipoglicemia inexplicada y fiebre,
hiperpigmentación, anormalidades electrolíticas (hiponatremia, hipercalemia, azotemia,
hipercalcemia) e hipotiroidismo. La crisis adrenal es precipitada por estrés. La
insuficiencia adrenal aguda es una amenaza contra la vida y el tratamiento no debe ser
demorado mientras se espera por pruebas definitivas de diagnóstico. No obstante , en
adicion a la medida de electrolitos en plasma y glucosa sanguinea, muestras apropiadas
de ACTH y cortisol deben ser tomadas antes de dar cortiscosteroides. Si el paciente no
está en estado crítico, un test de estimulación aguda de ACTH puede realizarse.
Medidas de emergencia para el tratamiento de una insuficiencia adrenal aguda incluyen
el establecimiento de acceso IV, toma de sangre para electrolitos y glucosa, asi como
medidas de cortisol y ACTH en plasma. Una infusion de 0.9% de salina con o sin 5% de
dextrose se debe iniciar. Hydrocortisone 100 mg es administrada y repetida cada 6
horas.
Prevención de una crisis adrenal en una enfermedad o situacion menor de estres se
puede dar doblando o triplicando la dosis de glucocorticoide por los pocos dis de la
enfermedad. Para procedimientos menores bajo anestesia local, no se necesita
suplementar dosis. Para procedimientos moderadamente estresantes, una sóla dosis de
100mg IV de hidrocortisona justo antes del procedimiento es administrada. Para cirugía
mayor administre hidrocortisona 100mg IV justo antes de la inducción de la anestesia y
continue cada 8 horas por las primeras 24 horas. Redusca la dosis rápidamente
disminuyendo a la mitad por dia a un nivel de mantenimiento.
Referencia:
Greenspan F, Gardener D: Basic & Clinical Endocrinology., Lange Medical
Books/McGraw-Hill, New York, 2004 pp384-386
Henderson K, Baranski T, Bickel P: The Washington Manual Endocrinology
Subspecialty
Consult., Lippincott Williams & Wilkins, Philadelphia, 2005 pp61-66
Larsen: Williams Textbook of Endocrinology, 10th ed. 2003, p. 525-527.

50 En la tabla abajo , ¿Cuál de los siguientes valores de laboratorio sería mas


comumente notado en el paciente hipertiroideo primario?
Respuesta: B
Racional:
El paciente con hipertiroidismo primario tiene elevaciones de ambas T3 y T4 desde la
glándula tiroides hiperplásica, nodulo, o adenoma/adenocarcionma. Esta elevación de
las hormonas ejerce una influencia inhibitoria en la liberación de TSH por la pituitaria
anterior, por medio de un mecanismo de retroalimentación negau mide el nivel de unión
a proteinas de la hormona tiroidea. Es típicamente elevado en el paciente hipertirioideol,
y disminuido en el paciente hipotiroideo. Si, no obstante, hay una lesión en la glándula
hipofisis, ejemplo una hiperplasia, adenoma entonces la TSH es secretada en exceso.
Devido a que la TSH esta elevada, T3 y T4 estaran elevadas. Esto es hipoertiroidismo
secundario (secundario a enfermedad hipofisiaria). En la mayoría de individuos con
hipotiroidismo, los niveles séricos de TSH resultan estar claramente elevados, pero los
resultados pueden ser inapropiadamente normlaes para el nivle de T4 y T3 en aquellos
con desordenes hipofisiarios o hipotalámicos.
Referencia:
Braverman LE, Utiger RD: Werner and Ingbar’s the Thyroid: A Fundamental and
Clinical
Text. 7th ed. 2005, pp 197-199, 336-341.
Warofsky L, Ingbar SH: Diseases of the thyroid. In: Wilson JD, Brunwald E, et al, eds.
Harrison’s Principles of Internal Medicine. McGraw-Hill; 1991:pp. 1692-1712.
McPherson & Pincus: Henry’s Clinical Diagnosis and Management by Laboratory
Methods, 21st ed., pp. 333-341. 2006 W. B. Saunders Company

51 Un varon de 9 años se presenta con ulceraciones orales y un agrandamiento gingival


eritematoso generalizado. Una revisión de sistemas evidencia una historia de dolor
abdominal y diarrea intermitente. Una evaluación endoscópica muestra ulceraciones en
el ileum y ciego separados por areas de mucosa aparentemente normal. Esta
presentación clínica es muy consistente con :
A. Enfermedad de Crohn.
B. Granulomatosis de Wegener
C. Colitis Ulcerativa
D. Sindrome del intestino irritable.
Respuesta: A
Racional:
La enfermedad de Crohn es una enfermedad inflamatoria intestinal que puede afectar
adultos jonves y se presenta con lesiones oral como ulceraciones aftosas de la mucosa
bucal, labios o gingiva.
Agrandamiento gingival generalizado como se ha descrito. La granulomatosis de
Wegener puede también presentarse con lesiones orales pero es mejor descrita como
una vasculitis granulomatosa afectando principalmente el tracto respiratorio superior,
pulmones, y riñones. Hallazgos endoscópicos de ulceraciones intestinales separadas por
áreas de mucosa normal o lesiones salteadas son comunes en la enfermedad de Crohn y
no caracteristica de una colitis ulcerativa. El sarcoma de Kaposi es un neoplasma
vascular frecuentemente visto en pacientes con SIDA y las lesiones orales aparecen
como maculas rojo azulado que progresan a nódulos azulados.
Referencia:
Shiboski C.H., Winkler J.R., Gingival Kaposi’s sarcoma and periodontitis. A case report
and suggested treatment approach to the combined lesions. Oral Surg Oral Med Oral
Pathol
(1993) 76: pp 49-53.
Goldman: Cecil Textbook of Medicine.23nd. Edition, 2007, p. 1042-1050
Stricker T., Braegger C.P., Images in clinical medicine. Oral manifestations of Crohn’s
disease. N Engl Med (2000) 342: pp 1644-.
Knight J.M., Hayduk M.J., Summerlin D.J., Mirowski G.W., .Strawberry. gingival
hyperplasia: a pathognomonic mucocutaneous finding in Wegener granulomatosis. Arch
Dermatol (2000) 136: pp 171-173.

52 Una mujer de 25 años de edad con prognatismo mandibular progresivo se presenta


con caracteristicas faciales,
Con amplios diastemas y puntas de los dedos bulbosas. Cual de las siguientes y la causa
mas probable?
A. hipertiroidismo
B. Feocromocitoma
C. hipercalcemia crónica
D. adenoma hipofisiario
Respuesta: D
Racional:
Este paciente tiene hallazgos altamente sugestivos de acromegalia, el resultado de
excesivos niveles de hormona del crecimiento (GH). La HC excibe acción anti
insulínica y cerca de 10% de aquellos con acromegalia desarrollan DB. Los niveles de
HC deben ser medidos. Un nivel mayor a 10ng/ml favorece el dignostico de
acromegalia. Si la HC no es siprimida por debajo de 5ng/ml una a dos horas después de
la ingestión de 100g de glucosa, el diagnóstico de acromegalia es mas conclusiva. La
causa de la acromegalia es frecuentemente un adenoma hipofisiario que secreta HC.
Frecuentemente el adenoma es lo suficientemente largo para causar distorción de la silla
turca y puede ser evidente en radiografias laterales. La hipercalcemia no causa facies
descritas. El hipertiroidismo no causa niveles aumentados de HC. El feocromocitoma
raramente ha expresado activación de liberación de factor de hormona del crecimiento
causando acromegalia.
Referencias:
Melmed, Jameson: C.318, Disorders of the anterior pituitary and hypothalamus,
Harrison’s
Principles of Medicine, 16th Edit; p.2070, McGraw-Hill, New York 2005
Larsen: Williams Textbook of Endocrinology, 10th ed, Saunders, 2003, p. 230-243.

53 ¿La neuropatía diabética puede directamente llevar a cual de las siguientes


condiciones?
A. hipotensión postural
B. bradicardia en reposo
C. arritmias
D. cardiomiopatía
Respuesta: A
Racional:
La neuropatía diabética no es una sola entidad si no un número de diferentes sindromes,
desde manifestaciones sublcínicas a clínicas dependiendo de la clase de fibras nerviosas
envueltas.
Puede causar hipotensión postural. Pacientes con DM tipo 2 e hipotensión ortostática
están hipovolémicos y tienen insuficiencia sinpaticoadrenal.
Ambos factores contribuyen a la patogénesis de la hipotensión ortostática. La
hipotensión postural en pacientes con nueropatía autonómica diabética pueden presentar
problemas de manejo dificil. Elevar la presión en la posición de pie debe ser balanceado
contra prevenir la hipertensión en la posición supina.
No causa directamente bradicardia, arritmias o cardiomiopatía a menos que sea
indirectamte por una aterosclerosis coronaria isquémica. La ND puede resultar en
disfagia, gastroparesis, constipación, y sudoración anormal.
Una constelación de desarreglos metabolicos que es frecuentemente vista en pacientes
con resistencia a la insulina y DM tipo 2 son individualmente asociadas con un mayor
riezgo de enfermedad cardiovascular. Estos pacientes han sido variadamente designados
como con sindrome X; sindrome dismetabólico, hipertensión, obesidad, y diabetes no
insulino dependiente(NIDDM), dislipidemia, y enfermedad cardiovascular
aterosclerótica; o el cuarteto de la muerte. El infarto al miocardio, derrame, y
enfermedad cardiovadscular no isquémica son la causa de muerto de hasta 80% de los
individuos con DM tipo 2. Independiente de otros factores de riesgo, la DM tipo 2
aumenta el riesgo de morbilidad y mortalidad cardiovascular pero tambien provee una
interacción sinergista con otros factores de riesgo como fumar, hipertensión, y
dislipidemia.
Referencia:
Powers: Diabetes mellitus, Harrison’s Principles of Internal Medicine, 16th Edit. C.323,
p.2165-2166; McGraw-Hill, New York, 2005
Baker, Maurer, Warman: Perioperative management of diabetes mellitus, Oral and
Maxillofacial Surgery Clinics of North America, 10:3:363-371; W.B.Saunders,
Philadelphia, 1998
Larsen: Williams Textbook of Endocrinology, 10th ed., 2003, Saunders, p. 1440-1441,
1553-1554.

54 La diuresis de dilución intensa después de trauma a la cabeza es causada por :


A. mayor producción de vasopresina.
B. menor producción de vasopresina.
C. mayor producción de reninaincreased renin production.
D. menor producción de renin
Respuesta: B
Racional:
Trauma a el hipotalamo produce una menor síntesis y secreción. Un riñon normal va a
intentar reabsorber o excretar agua libre de soluto para preservar una osmolaridad
plasmática de 275 a 290 mOsm/kg. La principal hormona reguladora de la osmolaridad
del plasma es la vasopresina arginina. Es sintetizada en el hipotálamo y liberada en la
circulación sistémica por medio de la glándula hipofisis posterior. A pesar de amplias
fluctuaciones en la injesta de agua y sodio, el cuerpo normalmente puede mantener una
osmolaridad sérica en un angosto rango (275 a 290 m Osm/kg). Los osmoreceptores
cerca del hipotálamo perciben la osmolaridad del plasma y modulan la liberación de
vasopresina. La vasopresina funciona a nivel del túbulo colector distal del riñon para
aumentar la reabsorción de agua en esta sección relativamente impermeable de la
nefrona. En condiciones hipo osmolares por ejemplo, los niveles de vasopresina caen a
una tasa baja básica para reabsorber menos agua, resultando en una orina más diluida.
La situación descrita es consistente con Diabetes insipidus, donde hay una disminuida
liberación de vasopresina, también conocida como hormona antidiurética. Los pacientes
con DI e inadecuada sed pueden tornarse deshidratados y pueden experimentar severa
hipernatremia con efectos devastadores en el sistema nervioso central. La encefalopatía
hipertónica con obtundación, coma, y convulsiones puede ser producidas por
encogimiento cerebral. Un disminuido volume del cerebro en el craneo puede llevar a
hemorragia subaracnoidea, sangrado intracerebral, o hemorragias petequiales.
Debido a que un paciente de trauma se le deben dar fluidos parenterales algunos clínicos
prefieren usar infusiones de bajas dosis de vasopresina. La vasopresina puede ya sea ser
agregada a la solución cristaloide que esta siendo administrada o infundida por separado
para mantener una antidiuresis constante mientras la injesta de fluido es ajustada
apropiadamente a una poliuria persistente y para cubrir perdidas insensibles de agua.
Dosis de 0.25 a 2.7 mU/kg por hora han sido descritas. Con este método hay potencial
para producir hiponatremia, y los niveles sericos de sodio que deben ser revisados con
regularidad.
La SIADH es producida cuando los niveles de plasma de arginina vasopresina son
elevados por momentos durante los cuales la secreción fisiológica de vasopresina de la
hipófisis posterior puede ser normalmente suprimida. Debido a la anormalidad clínica es
una disminución en la presión osmótica de los fluidos del cuerpo, la marca de la SIADH
es hipo osmolaridad, con euvolemia.
Los criterios clínicos para disgnóstico incluyen: osmolaridad plasmática menor a
275mOsm/kg, osmolaridad de orina mayor a 100mOsm/kg con función renal normal,
euvolemia clínica y elevada excreción de sodio con ingesta normal de sal y agua,
ausencia de otras causas (hipotiroidismo, enfermedad de addison , uso de diuréticos,
insuficiencia hipofisiaria ACTH).
La patofisiología de la polidipsia primaria es esencialmente el reverso de aquella en DI
central:
La excesiva injesta de agua expande y ligeramente diluye la orina. El resultado aumento
en la tasa de excresion de agua expande y ligeramente diluye la orina. El resultado
aumento en el balance de la tasa de excresión de agua expande y ligeramente diluye los
fluidos corporales, suprime la secreción de ADH y diluye la orina. El resultado aumento
en la tasa del balance de aumento de ingesta, y la osmolaridad del agua corporal se
estabiliza, de novo levemente a un menor nivel aproximado del umbral osmótico para la
secreción de ADH. La polidipsia primaria puede ser producida por drogas que causan
boca seca, o por algún desorden periférico causando elevaciones patológicas de renina y
angiotensina. La polidipsia primaria frecuentemente debida a enfermedad mental severa
como la esquizofrenia, mania o un desorden obsesivo compulsivo, en cuyo caso se
conoce como polidipsia psicogénica.
Reference:
Zimmerman: Systemic diseases, Merrit’s Textbook of Neurolgy, 8th Edit., p.825-826;
Lea
& Febiger, Philadelphia, 1989
Brenner & Rector’s The Kidney, 7th ed., 2004, Saunders, p.880-884.

55 ¿Un individuo diagnósticado con enfermedad de cushing puede demostrar cual


anormalidad?
A. hipovolemia
B. hipercalemia
C. hiperglicemia
D. hypotension
Respuesta: C
Racional:
La enfermedad de Cushin envuelve un exceso de glucocorticoides resultando
especificamente de un adenoma pituitario. Anormalidades físicas y bioquímicas
resultan de un estado prolongado de hipercortisolismo. Las anormalidades metabolicas
asociadas con hipercortisolismo incluyen; intolerancia a la glucosa, hiperglicemia,
hipocalemia e hipertensión. Hipovolemia e hipotensión pueden ser visatas en estados de
deficiencia de cortisol ( como enfermedad de Addison o disfunción pituitaria), pero no,
en pacientes diagnósticados con enfermedad de Cushing.
Referencia:
Endocrine Physiology, Mosby, Philadelphia, 2001 pp144-145
Andreoli T: Cecil Essentials of Medicine., W. B. Saunders Company, Philadelphia,
2001
pp570-571

56 Durante la extracción de premolares erupcionados para facilitar la ortodoncia, ¿cual


diente es más probable de tener una fractura radicular durante la extracción?
A. primer premolar maxilar
B.segundo premolar maxilar
C. primer premolar mandibular
D. Segundo premolar mandibular
Respuesta: A
Racional:
El primer premolar maxilar usualmente se bifurcan en raíces palatina y bucal las cuales
son frecuentemente delgadas y levemente curvas. Esto predispone este premolar a
fracturas radiculares con la extracción. El segundo premolar maxilar y primero y
segundo mandibular usualmente son de raíces únicas.
Referencia:
Peterson, LJ, Ellis, E, Hupp, JR, Tucker, MR. Contemporary Oral and Maxillofacial
Surgery, Mosby, St. Louis, 1988, p. 169.
Fonseca, et al. Oral and Maxillofacial Surgery: Anesthesia/Dentoalveolar
Surgery/Office
Management. (Vol 1). WB Saunders Company. Philadelphia. 2000. p. 221

57 ¿Cuál de los siguientes factores podria ser mas importante en decidir remover un
ápice de 2mm de un molar maxilar?
A. alta proximidad del ápice con el seno maxilar
B. edad del paciente menor a 50 años
C. experiencia y habilidad del operadorOperator skill and experience
D. presencia de patología periapical con el ápice
Respuesta: D
Racional:
En general, las raíces con lesiones peri apicales deben ser removidas siempre que sea
possible, incluso si estan cerca del seno maxilar. La edad del paciente menor a 50 años
no debe ser un factor en la decisión. Todos los cirujanos maxilofaciales deben de tener
adeucada experiencia y habilidad para remover ápices.
Referencias:
Berman, SA, Basic Principles of Dentoalveolar Surgery in Principles of Oral and
maxillofacial Surgery, Peterson, LJ, Indresan, AT, Marciani, RD, Roser, SM (eds), Vol
1,
Lippincott, Philadelphia, 1992, p.94.
Fonseca, et al. Oral and Maxillofacial Surgery: Anesthesia/Dentoalveolar
Surgery/Office
Management. (Vol 1). WB Saunders Company. Philadelphia. 2000. p. 432.

58 ¿Cuál de las siguientes condiciones es más probable de ser asociada con un tercer
molar erupcionado asintomático en un adulto joven?
A. reabsorción de la raíz distal de un diente adyacente
B. caries dental
C. perdida de soporte periodontal
D. quiste dentígero
Respuesta: C
Racional:
El más común de los problemas asociados con terceros molares retenidos es la pérdida
de soporte periodontal en el segundo molar adyacente y pericoronitis. Numerosos
estudios han documentado la presencia de patógenos periodontales y la pérdida de
soporte periodontal en distal del segundo y tercer molar. Esta condición se ha
demostrado progresa con la edad. La pericoronitis aguda es relativamente un hallazgo
común y si los terceros molares no son removidos, la condición es prona a recurrir y o
ocurrir con otro tercer molar. 60% de los paicentes con pericoronitis experimentan
síntomas asociados con el tercer molar contralateral dentro de los 12 meses previos.
Otros tipos de patología como una reabsorción de raices adyacentes o quistes
odontogénicos son menos comunes.
Referencias:
Marciani R and White RP, Establishing disease and clinical outcomes in asymptomatic
third molars. Oral and Maxillofacial Surgery Knowledge Update Vol IV. ALV 7, 12,
1996.
Marciani R, Third molar removal: an overview of indications, imaging, evaluation, and
assessment of risk. Oral and Maxillofacial Surgery Clinics of North America Vol 19(1),
1-
3, 2007.
Ash M, Costich ER, Hayward JR: A study of periodontal hazards of third molars. J
Periodontology, 33:209, 1962.

59 ¿Cuál de los siguientes es una indicación para realizar una incisión sulcular en vez de
una incisión festoneada en unión mucogingival cuando se realzia cirugía periapical en
un incisivo maxilar?
A. presencia de una raíz corta
B. preservar la estética gingival anterior
C. evitar incisiones relajantes
D. eliminar la necesidad de sutura
Respuesta: A
Racional:
Tipicamente, en la region anterior donde la estética es una preocupación, una incisión
festoneada submarginal es preferida. No obstante, contraindicaciones a estes abordaje es
desdtrucción periodontal, una lesión periapical grande, o una raíz cort. En estos casos
una incision sulcular de espesor completo con una o dos relajantes es preferida. Una
incisión sulcular puede también ser utilizada para evitar colocar la incisión sobre un
defecto óseo subyacente.
Referencias:
Fonseca, et al. Oral and Maxillofacial Surgery: Anesthesia/Dentoalveolar
Surgery/Office
Management. (Vol 1). WB Saunders Company. Philadelphia. 2000, pp 318-9.
Peterson, LJ, Ellis,E, Hupp,JR, Tucker, MR. Contemporary Oral and Maxillofacial
Surgery 4th edition, Mosby, St. Louis, 1998, pp. 385-394.

60 ¿Cuál de los siguientes factores está asociado con un favorable resultado cuando se
enderezan segundos molares?
A. enderezar involucrando un arco de rotación de mas de noventa grados
B. crecimiento vertical mandibular incompleto
C. la necesidad de corregir la posición bucolingual de la raíz
D. formación radicular del Segundo molar es 2/3 completa
Respuesta: D
Racional:
La formación radicular incomplete es un factor favorable cuando se reposiciona un
diente. Todos los otros factores mencionados pueden incrementar la dificultad del
enderezamiento del segundo molar. El procedimiento es mejor realizado después de que
se ha completado 2/3 partes de desarrollo radicular. En esta etapa el riesgo de fractura
radicular en mínimo. Realizar este procedimiento cuando menos de 2/3 de la raíz se ha
desarrollado puede resultar en un segundo molar flotante en su nueva posición. No
obstante el procedimiento se ha realizdo cuando el desarrollo radicular es complete la
incidencia de subsecuente necrosis pulpar o calcificación se incrementa.
Referencias:
Fonseca, et al. Oral and Maxillofacial Surgery: Anesthesia/Dentoalveolar
Surgery/Office
Management. (Vol 1). W.B. Saunders Company. Philadelphia. 2000. p308-316.
Dentoalveolar Surgery in Peterson’s Principles of Oral and Maxillofacial Surgery,
Miloro,
M. et al., Editors, Second Edition, BC Decker Inc, 2004, pp. 135
61 La tomografía conmputarizada abajo es de un paciente que se present al
departamento de emergencia con una historia de una inflamación instantánea no
dolorosa durante la extracción quirúrgica de un molar maxilar erupcionado por parte de
un dentista general. ¿Cuál es le manejo más apropiado?

A. Observacion
B. Aspiración transcutanea por punciónTranscutaneous puncture aspiration
C. Arteriograma con embolización selectiva
D. exploración quirúrgica
Respuesta: A
Racional:
La TC muestra un enfisema. No obstante una hemorragia puede ser incluida en le
diagnóstico diferencial, el escaneo claramente muestra aire y no sangre. La mayoría de
los casos de enfisema quirúrgico siguendo al tratamiento dental (72%) envuelven el uso
de turbinas de alta velocidad y jeringas de aire. El enfisema quirúrgico es caracterizado
por un aumento de volume tisular, de inicio espontaneo, usualmente desarrollandose en
minutes o segundos. La palpación de los tejidos muestra crepitación, una importante
caracteriztica diagnóstica. El discomformt es un hallazgo variable. Algunos pacientes se
quejan de dolor severo, pero usualmente la molestia es leve y resuelve dentro de unos
días. Muchos casis de enfisema subcutáneo comenzaran a resolver después de 2 a 3 dias
de tratamiento de soporte, y el aumento de volumen resuelve después de 2 a 3 dias de
tratamiento de soporte, la inflamación residual es usualmente mínima después de 7 a 10
días de observación. El tratamiento es usualmente conservativo, y consiste de prevenir
la infección. Bacterias orales pueden posiblemente ser transportadas con el aerosol
dentro de los tejidos blandos y representar un potencial nido de infección.
Adicionalmente un curso de corticosteroides sistémicos pueden promover una rápida
resolución. La descompresión quirúrgica de un enfisema extenso no debe ser usada
rutinariamente, debió a que es posiblemente infectiva y puede incluso empeorar o
diseminar el enfisema.
Referencias:
Heymans SN, Babayof I. Emphysema complications in dentistry. Quint Int 1995; 26:
535-543
Stanton D, Balasanian E, Yepes JF. Subcutaneous cervicofacial emphysema and
pneumomediastinum:
A rare complication after crown preparation. General Dentistry, 53(2), 122-4,
March-April 2005

62El lugar mas frecuente para un supernumerario impactado es:


A .región del incisivo central maxilar
B .región del canino maxilar
C .región del 3er molar maxilar
D .región premolar mandibular

Respuesta: A

Explicación: el sitio mas frecuente de impacción de un supernumerario es la


región central maxilar, seguida por la región del incisivo lateral maxilar y el
canino maxilar.

63 Cual de las orientaciones del tercer molar mandibular impactado es el mas difícil de
remover?
A. mesioangular
B. distoangular
C. horizontal
D. vertical

Respuesta: b

Explicación: la impacción distoangular es la más difícil debido a que la


trayectoria de esta extracción pasa por la rama mandibular. Esto requiere una
cantidad considerable de hueso removido y el seccionamiento múltiple del
órgano dental.

64 Coronectomia es una técnica alternativa para el tratamiento de un 3er molar


mandibular impactado en caso de:
A .infección periapical
B .el diente tiene movilidad
C .la raíz esta íntimamente asociada con el nervio alveolar inferior
D .el diente esta impactado horizontalmente en la trayectoria del nervio alveolar
inferior
Respuesta: c

Explicación: la coronectomia es una técnica viable en casos donde la extracción


del diente puede someter al nervio alveolar inferior en un riesgo de daño
considerable. Las otras tres opciones son contraindicaciones para esta técnica.
65 La mejor técnica para una odontectomia parcial (coronectomia) consiste en remover
el diente:
A .a un nivel de aproximadamente 3 mm sobre el nivel del canal alveolar inferior
y curación por segunda intención.
B .a nivel de que las raíces restantes queden 3mm debajo de el hueso crestal y
curación por segunda intención
C. a nivel de que las raíces restantes queden 3mm debajo del hueso crestal y
curación por primera intención.
D. a un nivel de aproximadamente 3 mm sobre el nivel del canal alveolar inferior
y curación por primera intención.

Respuesta: c

Explicación: el problema de un nervio alveolar inferior dañado en una


coronectomia tiene manifestaciones muy importantes clínicamente y en estos
días consideraciones medico-legales. Debido a que los resultados de este daño
son imprevisibles en que muchos casos llegan a curarse pero otros no, es
preferible llevar a cabo una técnica que reduzca el riesgo de daño al nervio. La
coronectomia u odontectomia parcial o retención deliberada de las raíces es una
de estas técnicas. La mejor técnica para la odontectomia parcial es seccionar el
diente a un ángulo de 45 grados medido en dirección vestíbulo-lingual seguida
por una reducción de los fragmentos restantes 3mm por debajo de el nivel del
hueso. Esta distancia de 3mm ha sido validada en estudios animales y permite la
formación de hueso por encima de los fragmentos restantes. Cierre primario es
muy importante para prevenir infección posoperativa.
La coronectomia parece una técnica segura y directa con pocas complicaciones
potenciales. En las notas de Pogrel se ha registrado un caso leve y transitorio de
parestesia lingual por 5 días, probablemente causado por la retracción lingual,
pero no han sido reportados casos de daño al nervio permanentes. Otros estudios
sugieren un porcentaje mayor de parestesias linguales transitorias debido al uso
de retractores linguales pero también no se reportan casos de daño permanente.
No hay necesidad de tratar la pulpa de las raíces restantes y el tratamiento de las
raíces es contraindicado. Estos remnantes seguiran vitales y con cambios
degenerativos mínimos como indican los estudios animales.
66 El mejor tiempo para administrar un antibiótico pre operativo para remover un 3er
molar impactado asociado a una infección pericoronal es
A .inmediatamente antes de la cirugía
B .0.5 a 2 horas antes de la cirugía
C .3-4 horas antes de la cirugía
D .6 horas antes de la cirugía

Respuesta: b

Explicación: la terapia antibiótica profiláctica en casos de terceros molares


impactados sin síntomas no es indicada en personas sanas. Aquellos pacientes
con infección activa o comprometidos la administración del antibiótico debe ser
medida de forma que la incisión se lleve a cabo en cuando la concentración
sistémica del medicamento este al mas alto nivel.

67 Hemorragia encontrada durante la exploración de este3er molar maxilar desplazado y


previamente impactado seria más probablemente procedente de:

A. plexo venoso pterigoideo


B .arteria sfenopalatina
C .arteria palatina descendiente
D .arteria maseterica

Respuesta: a

Explicación: este escaneo muestra un diente desplazado en la fosa infratemporal.


Terceros molares maxilares que están posicionados superiormente pueden tener
una capa muy delgada de hueso separándolas del espacio infratemporal.
Sangrado venoso del plexo pterigoideo comúnmente hace que el diente sea
difícil de visualizar.

68 La mejor forma de exponer quirúrgicamente un diente impactado para


colocación de brackets ortodonticos es:
A . Exposición completa de la línea cervical
B .exposición parcial de la corona y sin exponer la línea cervical
C .exposición completa de la línea cervical y 1mm del hueso alveolar
D .exposición de la corona hasta revelar el mayor diámetro de la corona sin
importancia a la línea cervical

Respuesta:b

Explicacion:la exposición quirúrgica de un diente impactado debe ser


conservadora de manera que sea solamente removido hueso y tejido blando
suficiente para poner el bracket ortodontico. Efectos dañosos al periodontio
fueron vistos con exposiciones más cercanas a la línea cervical.

69 La extensión posterior de una incisión diseñada para remover un 3er molar


mandibular impactado es extendido lateralmente por causa de:
A . Este diseño pone la incisión sobre tejido keratinizado
B .este diseño previene daño a la arteria bucal
C .este diseño previene daño al nervio lingual
D .este diseño previene trismo posoperativo

Respuesta:c

Explicacion:este diseño es para prevenir daño al nervio lingual.la rama


mandibular se desvía lateralmente y el nervio lingual esta ubicado sobre la cresta
lingual en 10% de los casos.

70 En un atentado de sacar un 3er molar maxilar impactado #16 el diente fue


desplazado y ya no es visible o palpable. El paciente tiene la apertura bucal
limitada. Cual es la ubicación más probable de este diente:
A . Seno maxilar
B .espacio bucal
C .cuerpo de la cigoma
D .espacio infratemporal

Respuesta:d

Explicación: terceros molares superiores pueden desplazarse distalmente a el


espacio infratemporal y en exclusive cuando se aplica fuerza excesiva de
elevación en dirección distal y sin el colocamiento de una retracción distal al
diente. En este caso la limitación de la apertura bucal implica que el diente esta
impugnado a la coronoide y de hecho esta en el espacio infratemporal y no en el
seno maxilar.

71 Cual es la complicación posoperativa mas frecuente en casos de extracción de


terceros molares :
A . Osteítis alveolar localizada
B .absceso subperiosteal
C .daño al nervio alveolar inferior
D .daño al nervio lingual

Respuesta: a

Explicacion:en un estudio que involucro 63 cirujanos 3,760 pacientes y 8,333


terceros molares, fue hallado que la incidencia de osteítis alveolar era 12% en
molares mandibulares. Otras complicaciones fueron infección 1%, daño al
nervio alveolar inferior 1.1-1.7% , y daño al nervio lingual 0.3%

72 Cual es la frecuencia en la cual el nervio lingual se ubica sobre el crest lingual


de la mandíbula:
A . ‹5%
B .10%
C .30%
D .50%

Respuesta:b

Explicacion:en un examen clínico utilizando MRI en mandíbulas de sujetos


humanos normales, fue hallado que en promedio el nervio esta ubicado a 2.8 mm
inferior a el lingual crest, y 2.5 mm medial a la cresta lingual. En 10% del grupo
fue ubicado sobre la cresta lingual y en 25% fue ubicado en contacto directo con
la cresta lingual .

73 Cual hallazgo radiográfico es comúnmente encontrado en casos de exposición


del nervio alveolar inferior durante la extracción quirúrgica de terceros molares
mandibulares.
A . Obscurecimiento de las raíces del tercer molar
B .estrechamiento de las raíces del 3er molar
C .deflexión de las raíces del 3er molar
D .diversión del canal alveolar inferior

Respuesta: a

Explicacion:la exposición del nervio es asociada con el riesgo de daño del


mismo. El hallazgo radiográfico de mas riesgo de daño al nervio es el
obscurecimiento. Todas las demás respuestas también son asociadas a la
exposición del nervio pero a menor grado.

74 En una técnica de separación del hueso alveolar (ridge splitting technique)


para expandir el (edentulous ridge) con el motivo de insertar un injerto óseo
interposicional prior a la inserción de un implante, cual es el espesor mínimo del
hueso alveolar preferible para el procedimiento:
A . 1 mm
B .2mm
C .3mm
D .4mm

Respuesta:c

Explicación: esta técnica (ridge splitting technique) es usada para expandir el


hueso alveolar para la inserción de un implante o un injerto interposicional. El
motiva de la técnica es aumentar el espesor del hueso.la altura del hueso debe ser
adecuada para la posición de un implante y no deben de existir defectos
verticales del hueso. El espesor preferible del hueso es de 3mm y casos de
espesor menor a 3mm son muy sensibles y riesgosos y pueden resultar en
fracturas óseas y resorbcion.

75 Cuando se realiza un desarrollo del hueso alveolar (alveolar rigde


development) usando erupción ortodontica forzada (forced orthodontic eruption)
, cual es la cantidad de movimiento dental recomendada por mes:
A . 1.0-2.0mm
B . 2.1-3.0 mm
C . 3.1-4.0 mm
D . 4.1-5.0 mm

Respuesta: a

Explicacion:el ortodontista tiene que saber los motivos del cirujano en respecto a
los movimientos deseados en el área de tratamiento. Para un alargamiento de
coronas (crown lengthening) los movimientos son rápidos .para un desarrollo
alveolar( alveolar ridge development) las fuerzas serian más lentas,
aproximadamente de 1 a 2 mm por mes.se debe de tener caución de no mover el
diente muy rápido. En adición la longitud de la raíz dentro del hueso determina
la frecuencia de movimiento y la fuerza necesaria . En situaciones de perdida
severa de hueso , la raíz restante dentro del hueso es mínima y la fuerza
requerida es menor.

76 La frecuencia optima de distracción recomendada al realizar una técnica de


distracción alveolar para un implante es de
A . o.5 mm por día
B .1 mm por día
C .2 mm por día
D .2.5 mm por día

Respuesta:b

Explicacion:la frecuencia de distracción es la distancia de hueso elongado al día.


Varias frecuencias han sido estudiadas. Una de 0.5 corre el riesgo de osificación
prematura , en cuando una de 2mm es acompañada por un incremento de tejido
fibroso interconectivo y formación ósea disminuida. Una frecuencia de 1mm por
día ( llevada a cabo en dos movimientos de distracción similares) es optima para
la formación ósea.

77 Al final de una distracción alveolar, el aparato estabilizador es mantenido por


un periodo de:
A . 2-4 semanas
B .5-7 semanas
C .8-12 semanas
D .13-17 semanas

Respuesta:c

Explicacion:la habilidad del distractor a estabilizar el hueso formado es esencial


para la formación de un regenerado sano. Aparatos inestables son asociados con
formación ósea endocondral (endochondral bone formation) y formación ósea
retrasada dentro del espacio distraído. Aparatos estables guían una osteogenesis
adecuada sin formación cartilaginosa.

78 La fase de osteogenesis de distracción alveolar para el desarrollo de un sitio


de implante, que ocurre al final de la distracción y hasta que el aparato sea
removido es conocida como:
A . Periodo de latencia
B .periodo de distracción
C .periodo de consolidación
D .periodo de activación

Respuesta:c

Explicacion:el periodo de consolidación sigue a la distracción activa y continua


hasta remover el aparato. La duración del periodo de consolidación es influida
por la edad del paciente, distancia y tiempo de distracción, y cantidad de trauma
quirúrgica al tiempo de la cirugía. En estos casos de trauma es aconsejado
incrementar el periodo de latencia y consolidación. El periodo de latencia es el
tiempo entre la colocación del aparato (osteotomía) y su activación.
El periodo de distracción es un termino clásico para el tiempo durante el cual el
aparato distractor esta activado, y el espacio entre los segmentos de la
osteotomía es expandido. El periodo de activación también describe el periodo
de distracción pero no es terminología clásica.

79 Osteogenesis ocurre con cuales de los siguientes materiales de injertos:


A . Injertos óseos alogenicos
B .injertos óseos xenograficos
C .injertos óseos aloplasticos
D .injertos óseos autógenos

Respuesta:d

Explicacion:osteogenesis se refiere al crecimiento de hueso del contenido de


células transferidas por el injerto. El hueso autógeno es el único material de
injerto disponible con
Propiedades osteogenicas.
Injertos alogenicos son tratados en bancos de sangre por varios métodos,
resultando en injertos en estados diferentes mineralizados, congelados-
desecados, solvente- deshidratados ,o desmineralizados. Los desmineralizados
seco-congelados( Demineralized freeze dried) y mineralizados solvente-
deshidratados( solvent dehydrated mineralized) son usados en sitios de
extracción, por su osteoconductividad y la característica de que se resorben y
serán reemplazados por hueso en un periodo corto de tiempo.

Aloplasticos incluyen formas del fosfato de calcio-en hidroxiapatatia densa o


porosa,reemplazamiento de tejido duro (hard tissue replacement), y vidrio
bioactivo (bioactive glass). Estos materiales han sido útiles para mantener el
espesor alveolar pero son lentos te absorber por sus características químicas.
Avances recientes en la adicion de materiales y cambios de características han
permitido formas en las cuales se mantiene la forma alveolar y permiten la
formación ósea.

80Comparado a los injertos de tejido conectivo subepiteliales ,injertos gingivales


libres (free gingival grafts) :
A . Resultan en menos cicatrices
B .revascularizan mas rápido
C .son menos adecuados para cubrir raíces
D .otorgan color similar y supremo

Respuesta:c
Explicación: injertos gingivales libres son primariamente usados para
incrementar el área de tejido keratinizado. En áreas anchas de recesión el
cubrimiento de raíces es limitado. Coincidencia del color es inferior y la
cicatrización es evidente en injertos gingivales. Injertos de tejido conectivo
subepiteliales revascularizan mas rápido debido a su irrigación sanguínea dual.

81Cual de las siguientes mejor caracteriza la técnica cerrada de crear un sitio


recipiente para un injerto de tejido conectivo subepitelial.
A . Ayuda a preservar la irrigación sanguínea en el sitio recipiente
B .es preferido cuando la exposición de la raíz es mayor a 4mm apico-coronal.
C .permite un avance coronal significante cuando el aumento del tejido vertical
es deseado
D .el ancho del sitio recipiente debe ser igual a la superficie de la raíz expuesta.

Respuesta: a

Explicación:

Técnica cerrada para la preparación del sitio recipiente.


Closed "pouch" technique for the preparation of a recipient site for a subepitelial
Connective tissue graft. A, split thickness dissection (shaded area). B, Graft
immobilization apically & coronally.

Debido a que no hay una incisión de liberacion,la técnica cerrada preserva mejor
la irrigación sanguínea. Esta técnica es preferida cuando la raíz no esta mas de
4mm expuesta apico-coronalmente.la técnica abierta permite un avance
significante coronal pero la cerrada no. él ancho del sitio recipiente en la técnica
cerrada debe ser 3 veces mayor a la superficie de la raíz expuesta. El sitio
recipiente debe ser diseñado de manera que la irrigación sanguínea periferal
contribuya adecuadamente a sostener el injertp.la disección debe extenderse mas
allá del espesor del tejido blando que esta siendo corregido

82Técnica del rollo palatal (palatal roll technique) para la aumentación de tejidos
blandos alrededor de implantes:
A . es solo útil con defectos gingivales pequeños
B .puede corregir defectos horizontales hasta 5mm
C .puede ser usado para reconstruir la papila
D .debe ser realizada un mes antes de descubrir un implante
Respuesta: a

Explicacion:este procedimiento es realizado al momento de descubrir implantes


que tienen defectos gingivales pequeños que necesitan aumentación. Pueden ser
usados únicamente para ayudar en el aumento de defectos pequeños y defectos
vestibulares.

83Injertos de tejido conectivo son menos previsibles si son realizados al


momento de :
A . extracción
B .injerto cortico-canceloso (cortico-cancellous graft)
C .colocación del implante
D .destapamiento del implante

Respuesta:b

Explicacion:parte de la irrigación sanguínea del injerto de tejido conectivo es


obtenida por el periostio subyacente. Todos los procedimientos quirúrgicos
interrumpen esta irrigación hasta algún nivel. La colocación de un injerto en
bloque, que involucra la elevación del periosteo,tiene el potencial mas alto de
interferir con la irrigación sanguínea, y por esa razón tiene una frecuencia de
complicación mayor comparada a los demás procedimientos listados.

84Después de un desarrollo del sitio para el colocamiento de un implante por


separación ortodontica de los dientes maxilares anteriores, que pasaría al espesor
vestíbulo-lingual del alveolo sin dientes con el tiempo:
A . resorbcion de menos de 1% sobre un periodo de 4 años
B .estrechamiento de 34% sobre un periodo de 5 años
C .resorbcion mayor en adolecentes que en adultos
D .resorbcion sobre un periodo de 5 años sin importancia de la colocación de
implantes

Respuesta: a

Explicacion:un estudio por Spear , Mathews y Kokich evaluó el espesor del


huaso alveolar al largo plazo después de la apertura de un espacio para el
incisivo lateral maxilar en adolecentes. La perdida ósea fue menos de 1%en 4
años. Estudios previos han mostrado que cuando los dientes anteriores de la
maxila son extraídos ocurre un estrechamiento de 34% sobre 5 años

85Cuando la técnica (conventional bone-added/socket lift osteotome) es usada,


que eleva el piso del seno:
A . la fresa usada para el (pilot hole )
B .la presión hidráulica del material del injerto
C .el osteotome mas chico
D .el osteotome mas grande

Respuesta:b

Explicacion:en esta técnica de elevación del seno, taladros u osteotomes son


usados para llegar a nivel del piso del seno. Un osteotome es usado para fracturar
el piso del seno inmediatamente antes de colocar el implante o el injerto. El
material de injerto o el implante eleva el piso del seno como paso final.

86En comparación a áreas injertadas con injertos palatinos autógenos, áreas


injertadas con injertos de matrices alogenicas dermales acelulares demuestran
cual de las siguientes calidades:
A . menor encogimiento del injerto y mas cantidades de tejido keratinizado.
B . mayor encogimiento del injerto y mas cantidades de tejido keratinizado.
C .menor encogimiento del injerto y menos cantidades de tejido keratinizado.
D .mayor encogimiento del injerto y menos cantidades de tejido keratinizado.

Respuesta:d

Explicacion:el injerto de matriz alogenica dermal acelular actúa como un


armazón incompatible en la cual fibroblastos y células epiteliales inmigran y se
adhieren, y de esta manera van incorporando las células al material. Estos tejidos
inmigrantes reemplazan la matriz dermal, causando un incremento en el
encogimiento del injerto y menos cantidades de tejido keratinizado en el sitio
recipiente.
87Donde existe tejido gingival keratinizado, la mejor opción quirúrgica para espesar el
tejido alrededor del implante para minimizar la muestra metálica facial es:
A . colgajo de reposición lateral
B .colgajo semilunar
C .injerto gingival libre
D .injerto de tejido conectivo subepitelial

Respuesta:d

Explicación: hay muchas indicaciones para el uso de un injerto de tejido


conectivo subepitelial. Este caso es una de esas indicaciones. El injerto
subepitelial puede espesar la encía de 1 a 3 mm dependiendo del espesor del
injerto y la contracción del injerto al cicatrizar y curar.

88El espesor biológico ( biologic width) se refiere a la distancia entre:


A . margen gingival y el hueso crestal
B .base del sulco y la cresta alveolar
C .margen gingival y el epitelio junctional.
D .base del sulco y la junción cemento esmalte

Respuesta:b
Explicacion:el espesor biológico es definido como la distancia entre la base del
sulco y la cresta del hueso alveolar. La profundidad exacta del sulco puede ser
problemática de medir con implantes a causa de que no hay una inserción directa
de fibras de tejido conectivo sobre la superficie del implante; en ese caso
teoréticamente, un probe periodontal ( en casos de fuerza excesiva) puede pasar
por encima del ataje del implante con el epitelio( el fondo del sulco) y llegar a la
cima de la cresta del hueso alveolar. El espacio biológico ( de 2mm mínimo ) es
mantenido entre cualquier (microgap) espacio en la interface del implante (
fixture / abutment interface)

89Para una aumentación ósea subantral, HA no resorbible es añadida a hueso


autógeno con motivo de:
A. Añadir espesor al injerto
B .mejorar la estabilidad inicial del implante
C .disminuir la probabilidad de infección
D .mejorar la osteogenesis

Respuesta :a

Explicación :en una mezcla con proporción de 1:1 con un injerto alogenico o
autogenico, la HA no resorbible ayuda a compensar la perdida de volumen óseo
que es observada con el uso de injertos. También es estable en su volumen
independientemente y actúa como una red que da soporte estructural.

Jonathan Escobedo Márquez


De la 110 a 139

90 Las contraindicaciones locales para un implante posterior de diente único de un


diámetro de 4mm incluye:

a. Un diente adyacente que requiere una corona.


b. Un amplio de hueso mesiodistal <7mm
c. Movilidad +1 de un diente adyacente.
d. Amplio de hueso bucolingual <7mm.

Respuesta: B

Razón:
Las contraindicaciones para la colocación de un implante en esta situación incluyen un
volumen inadecuado de hueso de <6mm en el sentido bucolingual y <7mm en sentido
mesiodistal. También más de un solo diente adyacente con una movilidad avanzada es
considerable inaceptable.
Ambos dientes adyacentes que requieren una corona es una indicación relativa para una
dentadura parcial fija (esto es influido por el pronóstico a largo plazo del diente o
dientes adyacentes).

91Una técnica sin la apertura del nicho quirúrgico (flapless) en la colocación de un


implante dental ha demostrado que:

A. Tiene un efecto positivo sobre la preservación de la papila interdentaria.


B. Tiene una taza de fracaso del 10% menor que utilizando una técnica con
colgajo.
C. Proporciona un mayor beneficio en la maxila vertical atrófica.
D. Minimiza la fenestración bucal apicalmente.

Respuesta: A

Razón: La técnica sin la apertura de un nicho quirúrgico (flapless) preserva la papila


interdental aunque no se ha demostrado que tenga un beneficio estético
consistentemente predecible comparado con la técnica con nicho quirúrgico. La
incapacidad de visualizar directamente el alveolo puede ocasionar problemas en
cuanto a una fenestración o dehisencia no detectadas.

92La técnica de expansión de osteotomo para la preparación del sitio de osteotomia


de un implante:
A. Requiere de un reborde alveolar amplio para recibir un implante.
B. No puede ser utilizada en hueso blando.
C. Comprime el hueso inmediatamente adyacente al implante.
D. Requiere una preparación secuencial de fresas antes de utilizarse.

Respuesta: C.

Razón: Los osteotomos son ideales para ser utilizados en el hueso blando maxilar.
Esta técnica puede ser utilizada para dividir o ampliar un borde alveolar angosto
para recibir un implante. Los osteotomos condensan y comprimen lateralmente el
hueso blando en el sitio de osteotomía. Solo necesita ser utilizada una fresa de
1.5mm antes del uso del osteotomo.

93Cual es la cantidad mínima de hueso lingual, bucal para permitir el alojamiento de


un implante de 4mm

A. 5mm
B. 6mm
C. 7mm
D. 8mm

Respuesta: B

Razón:
Por lo menos 1mm de hueso en bucal y lingual es requerido; por lo tanto una
amplitud de 6mm para un implante de 4mm. En cuanto a la amplitud mesial-distal,
son requeridos 7mm. 4mm para el implante y 1mm de hueso en ambas superficies
mesial y distal con el conocimiento adicional de que los ligamentos periodontales de
los dientes adyacentes son de 0.5mm.

94Cual de las siguientes infecciones tiene un virus ADN como factor etiológico:

A. Mononucleosis
B. Escrófula
C. SIDA
D. Herpangina

Respuesta: A

Razón:
Es estimado que el 79% de una infección de Mononucleosis es ocasionada por un
virus Epstein Barr (VEB); y 21% por una infección de citomegalovirus (CMV).
Ambos son virus de ADN.
La escrófula (linfadenitis cervical tuberculosa) es una infección secundaria de la
cadena linfática cervial, asociada con una tuberculosis pulmonar activa (tuberculosis
Mycobacteriana) y tiene como resultado una lesión que drena. El SIDA es
transmitido por un virus de inmunodeficiencia humano (VIH) y es un retrovirus
RNA. Los pacientes presentan infecciones oportunistas. La Herpangima es una
infección por virus Coxsackie y es caracterizado por erupciones vesiculares en las
fauces y el paladar. Los virus Coxsackie son virus de RNA.

95Un paciente que ha tenido esplenectomía dentro de los últimos 6 meses es más
susceptible a cual de los siguientes agentes infecciosos:

A. Treponema pallidum.
B. Haemophilus influenzae.
C. Pnumocystis carinii
D. Mycobacterium tuberculosis

Respuesta: B

Razón:
Los pacientes que han tendio una esplenoctomia son más susceptibles a la infección
por bacterias encapsuladas. El Haemophilus influenzae es el único que se encuentra
en la lista que esta encapsulado. Después de una esplenoctomía es recomendable que
a los pacientes se les administre una vacuna de Pneumnovaz para proveer inmunidad
activa al neumococo encapsulado.

96Abajo se encuentra una tinción de gram de un drenaje de una lesión en el cuello.


Cual de las siguientes es la terapia antibiótica inicial oral más apropiada:
A. Metronidazol
B. Trimethoprim-Sulfametoxasol
C. Penicilina
D. Vancomicina

Respuesta: B

Razón:
La tinción de gram de gram positiva para cocos en este grupo es consistente con el
estafilococo que es aeróbico. El metronidazol tiene una actividad antimicrobiana
contra las bacterias anaerobicas. La penicilina tiene algo de antimicrobiano contra
los estafilococos que no producen penicilinasa. La tinción gram no provee ninguna
prueba de si la bacteria produce penicilinasa, así que esta podría ser una elección
pobre. La vancomicina tiene unas propiedades antibacterianas excelentes contra los
estafilococos, pero no es absorbida cuando es tomada de manera oral. El
Trimetroprim-Sulfametoxasol a pesar de que típicamente es preescrito por su
actividad contra las bacterias gram negativas, también tiene excelentes propiedades
antibacterianas contra los estafilococos, es bien absorbida oralmente, y es el
antibiótico inicial más apropiado de esa lista.

97Un paciente con una infección cervicofacial severa es observado que produce una
orina con coloración de té. La presencia de cual de las siguientes en los análisis de
orina nos sugeriría una fascitis necrozante:

A. Bilirrubina
B. Hemoglobina
C. Mioglobina
D. Haptoglobina

Respuesta: C

Razón:
La fascitis necrozante puede ser asociada con rhabdomyolysis y secreción de
mioglobina en la orina. A pesar de que infecciones severas pueden ser asociadas con
la hemólisis y con la hemoglobinuria consecuente, y una bilirrubina aumentada en la
orina, estos parámetros no sugieren una inflamación muscular y necrosis. La
Haptoglobina une hemoglobina y es una molécula muy grande para que pueda ser
filtrada en la orina.

98La profilaxis Antibiótica antes de una cirugía dentoalveolar es recomendada para


que desorden:

A. Prolapso de la válvula mitral.


B. Prolapso de la válvula mitral con regurgitación.
C. Válvulas de corazón artificiales.
D. Estenosis aórtica calcificada.

Respuesta C:

Razón:
De acuerdo a los lineamientos más nuevos de la AHA (11) del (4-19-2007):
La profilaxis antibacteriana es requerida antes de una cirugía dental solo para las
siguientes condiciones:
1. Válvulas de corazón ratifícales.
2. Un historial de endocarditis bacteriana.
3. Condiciones específicas y congénitas severas (presentes desde el nacimiento) del
corazón incluyendo:
a) Enfermedad cianótica del corazón no reparada o incompletamente reparada,
incluyendo aquellas con medios paliativos.
b) Una reparación completa de un defecto congénito del corazón con material o
dispositivo protésico ya sea colocado por un cirujano o por una intervención
mediante cateter durante los primeros 6 meses después del cateter.
c) Cualquier reparación de defecto de corazón congénito con un defecto residual en
el sitio o adyacente a el sitio de un parche protésico o dispositivo protésico.
d) Un transplante cardiaco que desarrolla un problema en la válvula cardiaca.

99Cual factor disminuye el riesgo post-operatorio de una fractura mandibular


después de la remoción de un tercer molar:

A. Sexo femenino.
B. Edad mayor de 25 años.
C. Dentición completa.
D. Posición distoangular del tercer molar

Respuesta: A

Los principales factores que incrementan el riesgo de una fractura mandibular post-
operatoria después de la extracción de terceros molares son:
1) edad mayor de 25 años
2) sexo masculino
3) dentición completa

100Cual de los siguientes caracteriza mejor la superficie inferior de la membrana


Alloderm (mátriz acelular cutánea), cuando es utilizada como un injerto interposicional
para cubrir una raíz:

A. Retiene la coloración rojiza después del contacto con la sangre del paciente.
B. Facilita la migración de células del epitelio.
C. Debe ser colocada lejos de la superficie expuesta que se intenta cubrir.
D. Promueve la revascularización.

Respuesta: B

Razón:
La superficie de membrana inferior del injerto alloderm facilita la migración celular
epitelial y la unión. El lado de tejido conectivo contiene canales vasculares que
permiten la filtración celular y la revascularización. La superficie de membrana debe
ser colocada en contacto con la superficie de raíz expuesta cuando se pretende lograr
la cobertura de la raíz ya que esta es una superficie de epitelio. La superficie de
tejido conectivo retiene una coloración rojiza después del contacto con la sangre del
paciente; el lado inferior de la membrana permanece blanco.

101Una técnica de colgajo semilunar para la cobertura de una raíz con recesión del
tejido marginal está indicada en cual de las siguientes situaciones:

A. Ausencia de papila interdentaria.


B. El periodonto festoneado
C. Falta de curvatura facial del reborde severa.
D. Zona inadecuada de tejido queratinizado.

Respuesta: C

Razón:
Una técnica de colgajo semilunar no recreará la papila dental y muestra un éxito
pobre en pacientes con un periodonto festoneado. Si no hay una zona queratinizada
para empezar, el procedimiento no puede ser realizado. Una curvatura facial severa
del hueso también prevendrá del éxito al tratamiento.

102Cual de las siguientes es una limitación de la técnica de injerto de tejido


conectivo palatino:

A. Alta incidencia de pobre recuperación.


B. Dependencia de un sitio liso donador en el paladar.
C. La disponibilidad del injerto es dependiente sobre el grosor del sitio donador.
D. Alta incidencia de lesión neurovascular.

Respuesta: C

Razón:
El injerto de tejido conectivo palatino es particularmente útil debido a que no
depende de lo liso del paladar y a que su alivio es muy rápido. La incidencia de
lesión neurovascular es también poco común en la forma en la que se hace de
manera clásica (anterior al primer molar maxilar). Dependiendo del grosor del tejido
del paciente, la cantidad de injerto puede ser mínima y por lo tanto algunos pacientes
pueden requerir un injerto secundario algunos meses después.
103Cual es el mínimo de tiempo para que una stend parótido permanezca en su
lugar después de una reparación de conducto:

A. 2-4 semanas
B. 6-8 semanas
C. 10-12 semanas
D. 16-18 semanas

Respuesta: A

Razón:
Las laceraciones profundas de la mejilla pueden lastimar el conducto parotídeo y las
ramas del nervio facial. El conducto debe ser explorado. Los extremos proximal y
distal son identificados y una stent es suturada en la mucosa intraoral para prevenir
un desplazamiento accidental mientras que el conducto se recupera y es retirado
durante las siguientes 2-4 semanas. Este es tiempo suficiente para permitir la re-
epitelización del conducto dañado. Si la porción proximal del conducto no puede ser
localizada, se aplica presión para disminuir las posibilidades de un sialocele.
Aspiraciones múltiples de la saliva acumulada puede ser una parte necesaria en este
régimen. El uso de antisialogogues (como el glycopyrrolate) siempre es
recomendado como adjunto cuando se repara un conducto salival dañado.

104Cual es el porcentaje de pérdida más grande para un defecto de labio superior


avulsionado que puede ser cerrado de manera primaria sin comprometer la función o
la estética:

A. 20%
B. 30%
C. 40%
D. 50%

Respuesta: B

Razón:
Debido a la gran elasticidad de tejido, un defecto de avulsión de aproximadamente
30% del labio superior o inferior puede ser reconstruido de manera primaria sin
comprometer la función y la estética. Los defectos mayores del 30% requieren
colgajos locales y regionales para prevenir la microstomia.

105Cuanto tiempo después de una reparación de canaliculus inferior e intubación


debe el stent permanecer en su lugar en un paciente adulto:

A. 1-2 semanas
B. 4-6 semanas
C. 7-10 semanas
D. 12-16 semanas

Respuesta: D

Razón:
Las lesiones de canaliculus inferior necesitan reparación dentro de 24-48 horas para
poder prevenir una epífora. La reparación de esta clase de lesiones usualmente es
llevada a cabo por una intubación de curva con la punta siendo inicialmente
canulada con silastic stents. Los stends se extienden de la puncta hacia el ducto
nasolagrimal y emergen en el meato inferior y debe permanecer en su lugar por
aproximadamente 3 meses en el adulto, en caso de procedimientos pediátricos el
mismo procedimiento es realizado; sin embargo, el stent puede ser retirado en un
menor tiempo.

106Un paciente tiene una laceración en el párpado superior con grasa saliendo de la
herida. Herida que en cual de los siguientes puede ser eliminada:

A. Elevador palpebral superior


B. Globo
C. Grasa del Sub-ocular
D. Grasa del retro-orbicular

Respuesta: C
Razón:

La grasa subocular está localizada en la región del parpado inferior entre el periostio y el
músculo orbicularis oculi. No debe estar involucrada con una lesión del parpado
superior . Todas las demás opciones podrían suceder si se tratara de una lesión del
párpado superior.

107Cual es la presión mínima (en onzas por pulgada cuadrada) requerida durante la
irrigación para interrumpir mecánicamente la adhesión bacteriana a la superficie de una
herida:

A. 7lbs (3.18kg).
B. 10lbs (4.55kg)
C. 15lbs (6.82kg)
D. 20lbs (9.09kg)

Respuesta: A

Razón:
Para ser clínicamente efectivo, el irrigante debe ser liberado con chorro a presión
para que impacte en la herida con 7 libras de psi. La presión es la adecuada para
remover bacterias que se adhieren a una herida. Esta cantidad de presión puede ser
generada mediante un aparato de irrigación por pulsos.

108El ligamento de Whitnal se une a cual de las siguientes estructuras:

A. Tubérculo de Whitnal.
B. Ligamento suspensorio de Lockeood
C. Cuerno lateral del elevador de aponeurosis
D. Lóbulo Orbital de las glándulas lagrimales.

Respuesta: D

Razón:
Hay cuatro estructuras que se adhieren al retináculo lateral del tubérculo de Whinall:
el ligamento suspensorio de Lockwood, el cuerno lateral del elevador de
aponeurosis, ligamento check del músculo lateral recto y cuerpo posterior del tendón
lateral canthal. El ligamento de Whitnal, el ligamento suspensor del párpado
superio, se une alrededor del lóbulo orbital de la glándula lagrimal.

109La primera evidencia de hipotensión sistólica es observada en cual tipo de shock


hemorrágico:

A. Clase I
B. Clase II
C. Clase III
D. Clase IV

Razón:
La clase III de Shock hemorrágico es distinguido por un 30-40% de pérdida
sanguínea y la primera indicación es la hipotensión. La clase I y II de shock
hemorrágico no muestra ninguna reducción de presión sanguínea.

110Cual de las siguientes es considerada el transporte menos benéfico en el manejo


de dientes avulsionados permanentes:

A. Leche
B. Saliva
C. Solución de Hank
D. Sangre

Respuesta: D

Razón:
Los transportes aceptables para los dientes permanentes avulsionados incluyen la
solución de sal balanceada de Hank, saliva, y agua (si ninguno de los anteriores está
disponible) La sangre no es un medio aceptable de transporte de acuerdo a las guías
recomendadas por la Asociación Americana de Endodoncistas ya que no repone los
metabolitos celulares PDL. La solución de Hank es una solución salina balanceada
con pH fisiológico.

111Cual de las siguientes es una contraindicación verdadera para la recolocación de


un diente avulsionado:

A. Los dientes primarios pueden ser recolocados dentro de las primeras 1-2 horas
después de la avulsión.
B. La anquilosis después de la recolocación es una complicación rara.
C. Un diente avulsionado debe ser observado por lo menos durante los siguientes 5
años para determinar el resultado del diente.
D. La ferulización rígida es recomendada en la mayoría de los casos después de la
recolocación de un diente avulsionado.

Respuesta: C

Razón:
Los dientes primarios no deben ser reimplantados después de alguna lesión de
avulsión. El mantenedor de espacio es recomendado en estas situaciones. La
reimplantación de dientes primarios avulsionados pueden causar necrosis pulpar e
interferencia con el desarrollo del diente sucedáneo.

112Cual de las siguientes puede ocurrir después de la colocación de un expansor de


tejidos en el cuero cabelludo:

A. Displacia Epidérmica
B. Incremento en el espesor dérmico.
C. Atrofia de grasa.
D. Hiperplasia del músculo esqueletal.

Respuesta: C
Después de la colocación de un expansor de tejido en el cuerpo, los siguientes
cambios histológicos suceden: epidermis adelgazada, disminución en el espesor de
la dermis, no hay cambios en los folículos de cabello o en las glándulas sebáceas,
disminución en el espesor del músculo esquelético, incremento en capilares y atrofia
de grasa.

113La modificación de una cicatriz utilizando una técnica de Z-plastia de 60’


incrementara la longitud de la laceración por cuanto:

A. 35%
B. 45%
C. 75%
D. 95%
Respuesta: C

Razón:
La Z-plastia es utilizada para reacomodar una herida. Una Z-plastia de
30´incrementara la longitud de la herida en un 25% una Z-plastia de 45´ la
incrementara por 50%, y una Z-plastia de 60’ la incrementara en un 75%. El eje
mayor de la herida es rotado 90° y la longitud total de la incisión es alargada en
comparación al largo de la cicatriz extirpada.

114Cual es el organismo más comúnmente aislado encontrado en las mordidas de


animales mamíferos:

A. Escherichia coli
B. Clostridium tetani
C. Pasteurella multocida
D. Fusobacterium nucleatum

Respuesta: C

Razón:
A pesar de que múltiples organismos han sido aislados de mordidas de animales, del
50 al 70% de todas las mordidas contienen Pasteurella multocida. Este organismo
aeróbico es especialmente común en las mordidas de gatos. Antibióticos Beta-
lactámicos son indicados para el tratamiento de dichas heridas. El manejo incivil de
dichas heridas debe incluir debridación e irrigación.

115Cual es el mecanismo para el desarrollo de la coagulopatía seguida de una herida


axonal difusa (HAD):

A. Falta de producción del factor V


B. Liberación de tromboplastina del tejido
C. Liberación de antitrombina III
D. Producción excesiva de protrombina

Respuesta: B

Razón:
La HAD es común después de una herida traumática de desaceleración del cerebro.
Inicialmente, la HAD no muestra hallazgos específicos de CT o MRI. Sin embargo
con el tiempo, se observa un edema difuso en los escaneos de CT. La liberación de
tromboplastina del tejido por un cerebro dañado afecta a la cascada de coagulación
llevando al agotamiento de los factores de coagulación. La coagulopatía es un
hallazgo común después de una HDA.

116Cual arteria suministra la cabeza del cóndilo después de una fractura


subcondilar:

A. Masetérica
B. Pterigoidea media
C. Pterigoidea Lateral
D. Bucal

Respuesta: C

Razón:
El suministro axial al cóndilo mandibular es vía la arteria pterigoidea lateral. Esta
arteria suministra el músculo que está íntimamente asociado y unido a la cabeza y
cuello del cóndilo.

117Cuanto tiempo deben ser tratados los dientes permanentes subluxados con una
férula flexible:

A. 1-2 semanas
B. 3-4 semanas
C. 5-6 semanas
D. 7-8 semanas

Respuesta: A

Razón:
Una férula flexible (grabada por medio de ácido) debe ser utilizada durante 7-10 días
en dientes permanentes subluxados. Un periodo de tiempo corto es preferido en
lugar de un tiempo más largo para prevenir futuras complicaciones como la
anquilosis. La subluxación es definida como el movimiento del diente en cualquier
dirección mientras que una concusión es la secuela fisiológica/patológica de una
subluxación que afecta al tejido pulpar y rodea al ligamento periodontal.

118Cual de las siguientes es verdad en cuanto a los dientes avulsionados:

A. Los dientes permanentes con un foramen apical de menos de 1mm de diámetro


tienen una mejor prognosis que aquellos que tienen más de 1mm de diámetro.
B. Los dientes permanentes avulsionados guardados en solución de Hank tienen un
pronóstico más reservado que aquellos almacenados en solución salina
C. Los dientes permanentes reimplantados deben ser tratados con una férula rígida.
D. Los dientes primarios no pueden ser reimplantados.

Respuesta: D

Razón:
Los dientes permanentes con ápices mayores a un 1mm tienen un pronóstico mejor
que aquellos con ápices cerrados debido a que hay un potencial incrementado para el
restablecimiento de la circulación pulpar. La solución de Hank es considerada el
mejor medio fisiológico para los dientes avulsionados. Contiene cloruro de sodio,
cloruro de calcio, cloruro de potasio y sulfato de magnesio.
Los dientes primarios nunca deben ser reimplantados debido a que tienen un
diagnóstico pobre y puede causar anquilosis del diente permanente.

119 - Cual tipo de daño al diente conlleva al más alto grado de necrosis pulpar?
A. Extrusión.
B. Intrusión.
C. Luxación lateral.
D. Luxación lingual.

Respuesta: B
Justificación:
Los las lesiones de tipo intrusivo a los dientes causaran una mayor compresión e
inflamación de los tejidos periapicales y por tanto un mayor compromiso de la vasculatura
pulpar y el flujo sanguíneo causando una incidencia de 65-90% de necrosis pulpar.

120 - Cuál de las siguientes fracturas radiculares tienen el mejor pronostico?


A. Fractura horizontal en el tercio apical de la raíz.
B. Fractura horizontal en el tercio coronal de la raíz.
C. Fractura horizontal en la porción media de la raíz.
D. Fractura vertical de la raíz.

Respuesta: A
Justificación:
Fracturas en el tercio apical de la raíz tienen el mejor pronosticó de supervivencia
debido a que la porción apical de la raíz está completamente embebida en el hueso alveolar
y rodeado por ligamento periodontal. Están indicadas las valoraciones sucesivas. En
muchos casos, la terapia endodontica puede no estar indicada. Las fracturas radiculares
verticales requieren de extracción debido a que la totalidad de la cámara pulpar esta dañada
en este tipo de fractura.

121 - Cuál es el tratamiento recomendado para los dientes permanentes con fracturas
radiculares verticales?
A. Ferulizar los dientes por 2 semanas.
B. Ferulizar los dinetes por 6 semanas.
C. Ningún tratamiento es necesario.
D. Extracción.

Respuesta: D
Justificación:
Todos los dientes deciduos y permanentes con fracturas radiculares verticales deben
ser extraídos. Estos dientes tienen un mal pronóstico.
122 - El daño a que porción del seno frontal involucrara mas a los ductos naso-frontales?
A. Pared posterior.
B. Porción medial del piso.
C. Pared anterior.
D. Porción lateral del piso.

Respuesta: B
Justificación:
El daño a la porción medial del seno frontal más frecuentemente involucra daño a
los ductos naso-frontales. Esta es un área importante de evaluar intraoperatoriamente para
valorar la permeabilidad de los ductos naso-frontales. Otras áreas mencionadas no
involucran los ductos naso-frontales. Las fracturas de la pared posterior involucran el
cerebro, y las porciones laterales del seno puede involucrar el techo orbitario.

123 - Durante el abordaje de Gillies para acceder al arco zigomático, el plano de disección
está entre cuales dos capas?
A. Capa superficial y profunda de la fascia temporal profunda.
B. Fascia temporoparietal y capa superficial de la fascia temporal profunda.
C. Capa adiposa subcutánea y fascia temporoparietal.
D. Capa profunda de la fascia temporal profunda y musculo temporal.

Respuesta: D
Justificación:
El abordaje de Gillies para reducir las fracturas de arco zigomático utiliza una
disección entre la capa profunda de la fascia temporal profunda y el musculo temporal. El
plano de disección es sub-facial, pero supra-muscular. La disección en la fascia
temporoparietal puede lesionar el nervio facial y no alcanzara el arco zigomático. Debido a
que las dos capas de la fascia temporal profunda de dividen par envolver el arco, la
disección se debe mantener entre el musculo y la capa profunda de la fascia temporal
superficial para así ubicar el elevador profundo en el arco.
124 - El nervio marginal mandibular:
A. Anterior a la arteria facial, pero no la vena, en 100% de los casos.
B. Anterior a la vena facial, pero no la arteria, en 100 % de los casos.
C. Por debajo del borde inferior de la mandibula en el 19% de los casos donde el
nervio es posterior a los vasos faciales.
D. Por debajo del borde inferior de la mandibula en 5% de los casos donde el nervio
esta anterior a los vasos faciales.

Respuesta: C
Justificación:
Un estudio clásico en 1961 realizado por Dingman y Grabb mostro la relación de la
rama marginal mandibular del nervio facial en relación con los vasos faciales. En el 81%
de los casos, el nervio estaba superior al borde inferior de la mandíbula posterior a los
vasos. En el 19% de los casos, el nervio se ubicaba 1 cm inferior al borde inferior de la
mandíbula posterior a los vasos, y en el 100% de los casos, cuando el nervio estaba anterior
a los vasos, la rama marginal mandibular corría por encima del borde inferior de la
mandíbula. Debido a que la rama marginal mandibular corre de posterior a anterior, este
cruza verticalmente la arteria y vena facial y por lo tanto siempre se encuentra ubicado
anterior a ambos vasos faciales.
125 - La técnica de Keen es utilizada para reducir la fractura de cual estructura?
A. Cóndilo mandibular.
B. Tendón cantal medial.
C. Proceso coronoides.
D. Arco zigomático.

Respuesta: D
Justificación:
Los abordajes de Keen y Carmedy-Baxon son 2 métodos clásicos de abordar y reducir un
arco zigomático trans-oralmente. El surco bucal (vestibular) es el abordaje de Keen,
mientras el abordaje coronoide lateral (a lo largo de la rama ascendente) es conocido como
de Carmedy-Baxon. Estos dos abordajes permiten la reducción de un arco fracturado. El
abordaje de Keen permite accesar también al rin infraorbitario y regiones nasomaxilares
mientras que el abordaje de Carmedy-Baxon es algo mas limitado en su exposición.

126 - Cuál es la sutura ósea mas importante para el crecimiento futuro cuando se trata una
fractura naso-septal en la población pediátrica?
A. Nasomaxilar.
B. Nasofrontal.
C. Septovomeriana.
D. Frontoetmoidal.

Respuesta: C
Justificación:
La unión septovomeriana es considerada un centro de crecimiento en la nariz
pediátrica. Por lo tanot, una adecuada reducción de esta región es importante para disminuir
el riesgo de una osificación prematura la cual puede llevar a una futura perturbación del
crecimiento.

127 - Cuál es el factor más importante en el restablecimiento de la altura facial vertical en


el manejo de las fracturas pan-faciales?
A. Reducción de las fracturas de los cóndilos mandibulares.
B. Reduccion de las suturas fronto-cigomatica.
C. Establecer una adecuada proyección zigomática.
D. Establecer una adecuada oclusión dental.

Respuesta: A
Justificación:
Hay muchos hitos cuando se tratan lesiones pan-faciales. Mientras el
restablecimiento de una adecuada oclusión es un componente clave, la altura facial
posterior y la unidad cóndilo-rama deben ser alineadas adecuadamente para así obtener la
altura vertical de la cara. Esto se realiza mediante una adecuada reducción de los cóndilos
mandibulares después que se ha establecido una oclusión adecuada. Si no dientes no están
presentes, entonces la reducción anatómica de la mandibula y/o maxilar debe ser realizada
con el fin de establecer una altura vertical adecuada. Una vez que la oclusión adecuada y la
altura de la rama vertical son establecidas, la alineación de los huesos faciales verticales y
horizontales es realizada.
128 - Entre cuales 2 capas anatómicas es el plano de disección mas seguro durante la
elevación inicial de un colgajo coronal?
A. Grasa subcutánea y galea.
B. Pericraneo y galea.
C. Grasa subcutánea y fascia temporoparietal.
D. Almohadilla grasa galeal y almohadilla grasa temporal.

Respuesta: B
Justificación:
El plano más seguro de disección en la elevación de un colgajo coronal es el plano
subgaleal ubicado entre la galea y el pericráneo. A este nivel, la rama frontal esta por
encima (superior) del nivel de la disección y así es menos susceptible a lesionarse.

129 - Cuál es el área que se fractura con mayor frecuencia en la mandíbula edentula?
A. Cóndilo.
B. Subcondilo.
C. Sínfisis.
D. Cuerpo.

Respuesta: D
43.5 % de todas las fracturas de las mandíbulas edentulas ocurren en la región del
cuerpo. Esto es comparado con solo un 33% en los pacientes dentados. La fractura de
cuerpo es el segmento que se fractura con mayor frecuencia de la mandíbula edentula.

130 - Cual es el endoscopio utilizado más comúnmente (grado de compensación de


angulación y diámetro) para reparar fracturas de cóndilo mandibular?
A. 30 grados, 2 mm.
B. 0 grados, 2 mm.
C. 30 grados, 4 mm.
D. 0 grados, 4 mm.

Respuesta: C
Aunque la reparación de cóndilos endoscópicamente asistida no son realizadas tan
comúnmente como los métodos tradicionales, el endoscopio empleado con mayor
frecuencia es un endoscopio de 30 grados, 4 mm. Esta técnica requiere realizar una
incisión de Rison e insertar un endoscopio mientras se reduce la fractura. La aplicación de
la fijación puede ser realizada directamente o a través de un sistema de trocar.
131 - Cuál de los siguientes diferencia entre síndrome de fisura orbitaria superior y
síndrome de ápice orbitario?
A. Oftalmoplegia.
B. Perdida de la visión.
C. Ptosis del parpado superior.
D. Anestesia de la frente.

Respuesta: B
Justificación:
La perdida de la visión es el signo clínico que define la diferencia entre el síndrome
de fisura orbitaria superior y el síndrome de ápice orbital. Todos los otros signos y
síntomas enumerados son vistos en ambas condiciones. La oftalmoplegia ocurre debido a
que se involucran los nervios craneales II, IV, VI. La anestesia ocurre debido a que se
involucra V1. La Ptosis ocurre debido a la perdida del tono simpático del musculo de
Muller, debido a que las fibras simpáticas terminales viajan con la rama oftálmica del
nervio trigeminal (V1).

132 - Cuál de los siguientes puede causar diplopía binocular?


A. Desprendimiento de la retina.
B. Dislocación del lente.
C. Cicatriz corneal.
D. Alteración en la posición del globo.

Respuesta: D
Justificación:
La diplopía binocular es más común que la diplopía monocular, sin embargo ambas
pueden resultar de un trauma al globo. EL desprendimiento de la retina, la dislocación del
lente, cataratas y cicatrices corneales son causas de diplopía monocular. Los cambios de
posición del globo pueden llevar a una diplopía binocular.

133 - Cuál es la distancia del canal óptico a partir del foramen etmoidal posterior?
A. 1 mm.
B. 5 mm.
C. 10 mm.
D. 12 mm.

Respuesta: B
Justificación:
Cuando se mide a partir la cresta lagrimal anterior la distancia media del foramen
etmoidal anterior y posterior son de 24 y 36 mm respectivamente, el canal óptico esta a una
media de 5 mm posterior al foramen etmoidal posterior (o a una media de 42 mm desde la
cresta lacrimal anterior).
134 - Cuál es la mayor amplitud de un defecto de parpado superior que puede ser reparado
por un cierre primario sin compromiso de la función?
A. 10%.
B. 15%.
C. 20%.
D. 25%.

Respuesta: D.
Justificación:
Las lesiones de los parpados que involucran menos del 25% pueden ser cerradas en
la primera intención, aquellas que son 25-50% pueden ser reparadas con un avance de
tejido local. Aquellas defectos mayores de 50% requerirán un colgajo o injerto de piel lo
cuales reemplazaran las laminillas tanto anterior como posterior.

135 - Un hombre de 30 años de edad tiene deformidad bilateral en las partes bajas de las
piernas tras una colisión en un vehículo de motor. El es ansioso, con los siguientes signos
vitales: PS= 130/100, pulso = 110, rango de respiración = 28. Cuál es su clase de perdida
sanguínea?
A. Clase I.
B. Clase II.
C. Clase III.
D. Clase IV.

Respuesta: B.
Justificación:
La hemorragia clase II representa un volumen de perdida sanguínea de 750 a 1500
ml. Los síntomas incluyen taquicardia, taquipnea y disminución en la presión del pulso.
Esta disminución en la presión del pulso esta relacionada inicialmente a un incremento en
el componente diastólico debido a un incremento en las catecolaminas circulantes lo cual
incrementa el tono vascular y la resistencia. La presión sistólica cambia mínimamente en el
shock hemorrágico temprano.

136 - Un hombre de 25 años de edad presenta inconsciencia posterior a una caída. La


valoración neurológica muestra que se retira al dolor, no hay respuesta verbal, y no hay
apertura del ojo. Cual es su escala de Glasgow?.
A. 4.
B. 5.
C. 6.
D. 7.

Justificación:
La escala de Glasgow está basada en 3 variables: mejor respuesta motora, mejor
respuesta verbal y apertura de los ojos. (GCS= M+V+E) Los rangos de la escala van de 3 a
15. En este caso que se aleja del dolor representa 4, la ausencia de respuesta verbal
representa 1 y la ausencia de apertura ocular representa 1. GCS = (M4+V1+E1) = 6.
137 - Después de un traumatismo severo de cabeza, la presión intracraneal de un hombre de
45 años de edad es de 30 mm Hg. Otros signos vitales inclyen pulso 90, rango respiratorio
20, y presión sanguínea 130/85. Cuál es la presión de perfusión cerebral de este paciente?
A. 50.
B. 60.
C. 70.
D. 80.

Respuesta: C.
Justificación:
El fluido sanguíneo cerebral, mantenido por autorregulación, depende de la presión
de perfusión cerebral y la presión intracraneal. La presión de perfusión cerebral es la media
de la presión sanguínea arterial menos la presión intracraneal. (CPP = MAP – ICP). La
CCP norma en un adulto es >50 mm Hg.
Un método para calcular la media de la presión arterial es la presión diastólica 2 ves
mas la presión sistólica todo dividido entre 3.
{MAP = [(2 x presión diastólica) + presión sistólica] / 3}
= [(2 x 85) + 130] / 3
= 100
CPP = 100 – 30
CPP = 70

138 - Un hombre de 2 años de edad se presenta con quemaduras de segundo grado de la


cabeza entera. Que porcentaje de su superficie corporal está involucrada?
A. 4.5%.
B. 9%.
C. 10%
D. 18%.

Respuesta: D.
Justificación:
La “Regla de los Nueves” es una guía útil y practica para determinar la extensión de
la quemadura y la reanimación de los fluidos. La cabeza de los infantes representa una
gran proporción del área superficial. EL porcentaje de la superficie corporal total en la
cabeza de los infantes es dos veces mas que la de un adulto normal (lo cual es 9%).
139 - Cuál de los siguientes es un componente de la triada de Beck?
A. Presión de pulso extenso.
B. Disminución de la presión venosa central.
C. Aumento de la presión arterial sistólica.
D. Tonos cardiacos sordos.

Respuesta: D
Justificación:
La triada de Beck para el diagnostico del taponamiento cardiaco consiste en un
aumento de la presión venosa, disminución de la presión arterial y tonos cardiacos sordos.
La presión de pulso extenso es vista en el shock.

140 - Cuál es la fractura facial media más común en la población pediátrica?


A. Techo orbitario.
B. Piso orbitario.
C. Zigomatico-maxilar.
D. Nariz.

Respuesta: D.
Justificación:
La nariz es una estructura prominente en los niños y los hueso nasales son frágiles.
Las fracturas nasales son las lesiones esqueletales más comunes en niños. La baja
incidencia de fracturas del tercio medio facial en niños puede también ser explicada por la
elasticidad de los huesos faciales en niños, la posición retrusiva del maxilar, nariz y rines
infraorbitarios y la protección anatómica que ofrece el cráneo.
141 - Cuál es el área más común de fractura mandibulares en pacientes pediátricos?
A. Sínfisis.
B. Parasínfisis.
C. Proceso coronoide.
D. Cóndilo.

Respuesta: D
Justificación:
Las fracturas de cóndilo se han reportado que ocurren más comúnmente en los
pacientes pediátricos (15 - el 60%). Con el aumento de edad, el ángulo está implicado más
comunmente.
Distribución anatómica de las lesiones mandibulares en pacientes pediátricos
Cóndilo 15 - 60 %
Alveolo 8 – 50,6 %
Cuerpo 6 – 44 %
Sínfisis 2 – 40 %
Parasinfisis 23 – 33 %
Angulo 3 – 27 %
Rama 1 – 10 %
Coronoide 0 – 19 %
142 - Qué explica la falta relativa del trauma medio facial pediátrico comparado a los
adultos?

A. Senos paranasales grandes.


B. Huesos faciales rígidos.
C. Estructuras medio facial prominentes.
D. Tamaño del cráneo.

Respuesta: D

Justificación:
La incidencia baja de fracturas medio facial en niños se puede explicar por la
elasticidad de los huesos faciales del niño, la posición retrusiva del maxilar, nariz y los
bordes infraorbitarios y la protección anatómica producida por el cráneo.

143 - Cuál de los siguientes es una consideración para una reducción abierta y fijación
interna en pacientes pediátricos con fracturas condilares?
A. Maloclusión menos en la dentición mixta.
B. Maloclusión menos en la dentición primaria.
C. Maloclusión persistente en la dentición permanente.
D. Para restaurar la altura vertical de la rama a pesar de la edad u oclusión.

Respuesta: C
Justificación:
La reducción abierta y fijación interna esta raramente indicada en la población
pediátrica. La mayoría de las fracturas condilares pueden ser tratadas con una combinación
de fijación intermaxilar o analgésicos y dieta blanda. Las discrepancias oclusales menores
se equilibraran con crecimiento continuo y la erupción dental. Los niños en dentición
permanente quienes tienen maloclusión persistente al momento del tratamiento inicial o
después de un periodo de fijación intermaxilar debe ser considerado para la reducción
abierta y fijación interna para restaurar la altura de la rama y la función oclusal.

144 - Como el shock hemorrágico pediátrico y la descompensación circulatoria difiere de la


del adulto?.
A. Hay mecanismos compensatorios superiores en los pacientes pediátricos.
B. El shock hemorrágico ocurre gradualmente en niños a pesar de la perdida rápida de
sangre.
C. El shock hemorrágico ocurre mas repentinamente, con vasoconstricción,
taquicardia, y contractibilidad miocardica pueden enmascarar los signos y síntomas.
D. El shock hemorrágico raramente ocurre en la población pediátrica debido a una
mayor area de superficie corporal y rango de masa.

Respuesta: C.
Justificación:
El manejo del trauma en pacientes pediátricos difiere de el del adulto. La ausencia de
reservas compensatorias en los pacientes pediátricos puede llevar a un colapso
cardiovascular repentino. Esto ocurre bastante rápido debido a la taquicardia,
vasoconstricción e incremento de la contractibilidad miocardica enmascararan inicialmente
la presencia de signos y síntomas. El radio de superficie corporal no afecta la respuesta del
choque, pero no afecta el calor y la perdida insensible de fluidos en los pacientes
pediátricos.

145. Un paciente de trauma tiene multiples laceraciones fuertemente contaminadas con


tierra. El ha completado una serie de inmunización para el tétano pero no ha recibido el
refuerzo para los últimos 7 años. Como seria manejado este paciente?

A. injección del toxoide tetanico booster


B. Tetanus booster and tetanus immunoglobulin
C inmunoglobulina tetánica
D. Tetanus booster, inmunoglobulina tetánica mas un complete curso de
inmunizacion.tetanus

Respuesta: A

Fundamento:

En una limpia (mínima o no contaminada) podría en un paciente con previa inmunización


quien no ha recibido a booster in los últimos 10 años, una dosis booster de 0.5ml de booster
toxoide estar recomendada. Este tiempo disminuye a 5 años para una herida contaminada.
Si no hay historia de inmunización o no hay datos históricos, la serie de inmunizaciones
esta iniciada; con la adición de inmunoglobulina tetánica (TIG) para heridas contaminadas.

Reference:
Fonseca Oral & Maxillofacial Trauma, 3rd edition; Management of soft tissue injuries, P
751-820, Gomella Clinician’s Pocket Reference, 9th Edition; p 330
146. La remoción de dientes en la linea de fractura mandibular incluye cual de las
siguientes indicaciones?

A. terceros molares impactados completos en hueso en la línea de fractura.


B. dientes con exposición de raíz y ápice en el sitio de fractura.
C. dientes con fractura coronal
D. diente immobile en la línea de fractura

Respuesta. B

Fundamento:

Indicaciones relativas para la remoción de dientes in la linea de fractura mandibular


incluye.
Presencia de patología
Movilidad de los dientes
Dientes que previenen la adecuada reducción de la fractura
Dientes con fractura radicular
Dientes con exposición de ápices

Reference:
Peterson 2nd edition, Principles of management of mandibular fractures. P 401-434,
Fonseca, Oral and Maxillofacial Trauma, Mandibular fractures, P 479-522, 2005,
147.Cuál de las siguientes es una contraindicación relativa para una reducción cerrada de
una fractura mandibular?

A. Poorly controlled seizure disorder


B. enfermedad renal mal controlada
C. Desorden psiquiátrico bien controlado
D. enfermedad pulmonar obstructiva crónica bien controlada

Respuesta: A

Fundamento: Poorly controlled seizure disorders, retraso mental, y desordenes psiquiátricos


son relativamente contraindicaciones para la reducción de fracturas mandibulares en la cual
la fijación maxilomandibular podría ser pobremente tolerada. Si es posible un intento
podría ser hecho con reducción abierta con fijación rigida para esos pacientes.

Referencia

Peterson 2nd edition, Principles of management of mandibular fractures. P 401-434, 2004


Fonseca, Oral and Maxillofacial Trauma, Mandibular fractures, P 479-522, 2005

148. Que tratamiento esta recomendado para pacientes con un fractura de cabeza de cóndilo
intracapsular y oclusión normal?
A. Observación
B. 2 semanas de fijación intermaxilar
C. 6 semanas de fijación intermaxilar
D. 8 semanas de fijación intermaxilar

Respuesta: A
Fundamento
La fractura de cabeza condilar intracapsular con oclusión normal deberías ser observada.
Estos pacientes no necesitan fijación intermaxilar desde que no hay cambios en la oclusión.
El manejo también incluye dieta blanda, control del dolor y un rango de ejercicio para
recobrar la apertura interincisal normal después de iniciar el periodo de cicatrización.
Referencia:
Peterson 2nd edition, Principles of management of mandibular fractures. P 401-434, 2004
Fonseca, Oral and Maxillofacial Trauma, Mandibular fractures, P 479-522, 2005
149 Cuál de los siguientes es una contraindicación para exploración de un fractura de piso
de órbita?

A. Enoftalmos
B. Grado III de hyphema
C. Diplopía persistente
D. Limitación de la función del músculo extraocular

Respuesta: B

Fundamento:

Hyphema es una contraindicación para exploración orbital por la posibilidad de sangrado


dentro de la cámara anterior, la cual puede llevar una mancha corneal o glaucoma.

Referencia:

Peterson 2nd edition, Principles of Oral & Maxillofacial Surgery, Orbital and ocular
trauma, P 463-490, 2004
Brandt MT, Haug RH. Traumatic hyphema: a comprehensive review. J Oral Maxillofac
Surg, 59, 1462, 2001.
150 Cuál de los siguientes es menos probable que cause diplopia?

A. Chemosis
B. Hematoma
C. Extraocular muscle entrapment
D. Abducens nerve injury

Respuesta: A

Fundamento:

Es un edema de la conjunctiva palpebral y bulbar. Esto no permite la diplopia. Otras


condiciones incluyendo hematoma, ENTRAPMENT del músculo y lesi´ón al nervio
podrían causar visión doble.

Referencia:

Peterson 2nd edition, Principles of Oral and Maxillofacial Surgery, Orbital and ocular
trauma, P 463-490, 2004
Fonseca, Oral and Maxillofacial Trauma, ophthalmic consequences of maxillofacial
trauma, p 693-720, 2005l

151 Cuál de las siguientes es la mas clara indicación para la reducción abierta de la
fractura condilar mandibular?
A. Fractura condilar unilateral con comunicación con fractura superior facial
B. Desplazamiento dentro de la fosa craneal media
C. Dislocación de la cabeza condilar medial extracapsular
D. Fractura aislada del cuello condilar
Respuesta: B

Fundamento:
El criterio Zide para reducción abierta para fractura condilar incluye las siguientes
absolutas y relativas indicaciones:

Tabla 22-3 indicaciones para la reducción abierta de fracturas en el cuello mandibular.

1. Indicaciones absolutas
A. Limitación en la función secundaria a lo siguiente:
1. Fractura dentro de la fosa craneal media
2. Objeto extraño dentro de la cápsula articular
3. Dislocación de la cabeza condilar extracapsular lateral
4. OTHER FRACTURE DISLOCATIONS en las cuales el tope mecánico se
presenta en apertura, lo cual es confirmado radiográficamente.
B. inhabilidad para traer los dientes a oclusión para reducción cerrada

2. Indicaciones relativas
A. fractura condilar bilateral con fractura conminuta en tercio medio en la cual la
fijación rígida interna del tercio medio no es posible.
B. Situaciones en las que la fijación intermaxilar no es posible como resultado de
lo siguiente:
1. Restricciones médicas
a. POORLY CONTROLLED SEIZURE DISORDER
b. Desordenes psiquiátricos
c. Retardo mental severo
d. Lesiones concomitantes tales como lesión de cabeza y cuello ( sin
traqueostomía planeada)
2. fractura desplazada donde las dentaduras o tablas no están factibles por la
severa atrofia mandibular.
C. Fractura bilateral en la cual es imposible determinar donde esta la oclusión
apropiada como resultado de la pérdida de dientes posteriores o por la presencia
de una lesión previa en maloclusión esqueletal.
D. IN FRACTURE DISLOCATIONS en adultos a restaurar la posición y la
función de los meniscos.

Referencia:
Zide MF. Open reduction of mandibular condyle fractures. Indications and technique. Clin
Plast Surg. 1989 Jan; 16(1):69-76.
Peterson 2nd edition, , Principles of management of mandibular fractures. P 401-434,
2004
Fonseca, Oral and Maxillofacial Trauma, Mandibular fractures, P 479-522, 2005
152 Un paciente tiene una laceración profunda vertical en su cara extendida de la parte
media baja del párpado a la comisura oral ipsilateral. Al exámen físico revela debilidad en
las ramas del nervio facial. Como sería manejado este paciente?

A. Explorar la lesion e identificar y reparar la lesion nerviosa.


B. Irrigar y cerrarar la lesion sin exploración del nervio.
C. Reparar la laceración y esperar dos semanas para que regrese la function, si no aumenta
la function, luego explorer y reparar.
D. considerer un injerto nervioso del gran nervio auricular.

Respuesta: B

Fundamento:
Lesions nerviosas medias a una línea vertical del canto lateral no necesita ser explorado
porque ahí hay un aumento en la absorción de ramas terminales del nervio facial y existen
multiples pequeñas ramas. La exploración a esas pequeñas ramas no es práctico. Esta lesión
deberá ser manejada como otra laceración facial.

Reference:
Peterson 2nd edition, Principles of Oral & Maxillofacial Surgery, Soft tissue injuires, P
357-370, 2004
Fonseca, Oral and Maxillofacial Trauma, Applied surgical anatomy of the head and neck, p
281-328, 2005,
153 Siguiendo la reparación de fractura panfacial, un paciente desarrollo diplopia, proptosis
orbital y reporta un inusual zumbido. Cuál es el diagnóstico mas probable?

A. trombosis de seno cavernoso


B. sindrome del apice orbital
C. Fístula carotida cavernosa
D. arteritis temporal

Respuesta. C

Fundamento:

La fistula carótida cavernosa es una complicación potencial del trauma craniofacial. Los
signos y síntomas estan relacionados al anormal fluido de la sangre entre un alto fluido
(carótida interna) a un bajo sistema (seno cavernoso), los pacientes frecuentemente tienen
ojos hinchados y rojos, dolor orbital y un zumbido y un silbido, hay proptosis y cambios
visuales. También trombosis del seno cavernoso, síndrome del ápice orbital y arteritis
temporal puede tener algunas alteraciones visuales, ninguno dara un sumbido inusual. La
fístula carótida cavernosa puede ocurrir dentro de los días o semanas seguidos al trauma.
Referencia:

Origitano TC, Al-Meffy O. Aneurysms of the cavernous sinus: treatment options and
consideration. In: Youmans RJ. Neurological Surgery. Philadelphia: WB Saunders,
1995, 1320-1335.
Fattahi T, Brandt MT, Jenkins WS, et al. Traumatic carotid-cavernous fistula:
pathophysiology and treatment. J Craniofacial Surg, 14:1, 2003.

154 En cual zona del cuello es mas susceptible la examinación física?

A. Zona 1
B. Zona 2
C. Zona 3
D. Zona 4

Respuesta: B

Fundamento:

Lesiones penetrantes en el cuello pueden tener una significativa morbilidad y mortalidad.


La zona 1 se extiende de la clavícula al cricoides. La zona 2 es del cricoides al borde
inferior de la mandíbula. La zona 3 es del ángulo mandibular a la base del cráneo. La
examinación física de la zona 2 es mas fácil de lograr por su localización y fácil acceso.
mientras estudios extras sobre el exámen clínico de la zona 2 están indicados, la zona 1 y 3
mas frecuentemente requieren mas herramientas de disgnóstico tales como un arteriograma
o esofagoscopia.

Referencia:

Advanced Trauma Life Support Student Manual. Initial assessment and management, P
21-46, American College of Surgeons. Sixth Edition. 1997.
Miloro, M. et al, Peterson’s Principles of Oral and Maxillofacial Surgery, Second Edition,
Initial management of the trauma patient, p 327-356, BC Decker 2004, chapter 18
155 Cual es la posición mas probable en una fractura de cóndilo mandibular?

A. Superior y lateral
B. Superior y medial
C. Anterior y medial
D. Anterior y lateral

Respuesta: C

Fundamento:

El desplazamiento condilar usualmente ocurre en una dirección medial y anterior


secundaria a una tracción del músculo pterigoideo lateral. El paciente tendrá una mordida
prematura en el lado ipsilateral y sobre la apertura podrá desviarse al lado ipsilateral
también.

Reference:
Peterson 2nd edition, Principles of Oral and Maxillofacial Surgery, Principles of
management of mandibular fracture, p 401-434, 2004
Fonseca, Oral and Maxillofacial Trauma, Mandibular fractures, p 479-522, 2005

156 Formación de un callo cartilaginoso siguiendo en una formación inicial con una marca
de hematoma.
A. inicial
B. Primario
C. Secundario
D. terciario

Respuesta: C

Fundamento:

La cicatrización ósea ha sido dividida en 2 categorias: primaria y secundaria. La primaria


se caracteriza por menos de 1mm de espacio entre el borde óseo, sin movilidad ósea y sin
formación de callo. Este tipo de cicatrización ocurre siguiendo la colocación de placas load
bearing plates. la cicatrización ósea secundaria ocurre con la formación de un hematoma,
infiltración fibroblástica, cartilaginosa y formación de un callo óseo, y la remodelación.
Este tipo de cicatrización ocurre siguiendo la reducción cerrada de las fracturas.

Referencias:

Peterson 2nd edition, Principles of Oral and Maxillofacial Surgery, Principles of


management of mandibular fracture, p 401-434, 2004
Fonseca, Oral and Maxillofacial Trauma, Mandibular fractures, p 479-522, 2005
Abubaker OA. Oral and Maxillofacial Surgery Secrets. Maxillofacial trauma, 197-206
2001

157. La aperture maxima interincisal de 5mm inmediatamente después de 3 semanas de una


fijación maxilo-mandibular se explica mejor por?

A. anquilosis ósea
B. anquilosis fibrosa
C. Muscle splinting
D. hipertroofia de los músculos de la masticación

Respuesta C:

Fundamento:

En la mandibula , 3 semanas de IMF para una fractura de cóndilo para un adulto es justo
SLIGHTLY mas largo de lo usual. Sin embargo, no es lo suficientemente largo para causar
fibrosis o anquilosis ósea. La hipomovilidad seguida de reducción cerrada es due to muscle
splinting and guarding. Esto responde muy bien a fisioterápia. La anquilosis es la mayor
complicación de una prolongada IMF especialmente en niños.

Referencias:
Peterson 2nd edition, Principles of Oral and Maxillofacial Surgery, Principles of
management of mandibular fracture, p 401-434, 2004
Fonseca, Oral and Maxillofacial Trauma, Mandibular fractures, p 479-522, 2005

158. Cual de los siguientes pacientes es major tratado con descamación química superficial.

A. 45 años de edad con queratosis actinica y lentigines.


b. 70 años de edad con severa photoaging y wrinkles.
C. 28 años de edad con localizadas cicatrices hipertróficas en la piel.
D 16 años de edad con cicatrices de acne y herpes labial activo

Respuesta: A

Fundamento:

La descamación química esta bien aceptada como forma de tratamiento para larejuvenecer
la piel. Este tiene la ventaja de ser seguro y eficaz, hecha con facilidad y bajo costo y
relativamente rápido. Varios componentes ácidos y básicos son usados para producer una
lesión controlada en la piel y puede ser clasificada como superficial, media y descamación
profunda deacuedo al nivel de penetración, destrucción del tejido e inflamación. la
descamación superficial involucra penetración de solo la eídermis y la dermis papilar y esta
indicada en el tratamiento del acné ligero y su eritema post inflamatorio, leve photoanging
(glogau i,ii), y crecimiento epidermal como la queratosis actínica y lentigines como un
melanoma y otras discromías pigmentadas. multiples sesiones son usualmente requeridas
para óptimos resultados. un paciente de 70 años de edad con severa photoagingy dynamic
wrinkles es un paciente glogau clase iv , y un tratamiento de descamación superficial no
tendrá ningún efecto benéfico. las cicatrices hipertróficas son usualmente tratadas con
inyecciones de esteroides en la lesión, terapia lase, demoabrasión o revisión quirúrgica de
la cicatriz. El herpes labial activo es una contraindicación para el tratamiento rejuvenecedor
químico y con laser.

FIGURA
Referencias:

Glogau classification of photoaging: Reproduced from: Monheit GD: Skin rejuvenation


procedures. Principles of Oral and Maxillofacial Surgery. 2nd edition, Volume 2, BC
Decker Inc, London, P 1419:
Demas PN, Braun TW: Chemical skin resurfacing. Esthetic surgery of the aging face.
Oral and Maxillofacial Surgery Clinics of North America. Vol 6(8), 1998. P 2
Monheit GD: Skin rejuvenation procedures. Principles of Oral and Maxillofacial Surgery.
2nd edition, Volume 2, BC Decker Inc, London, P 1419

159 Una técnica de descamación química de la piel facial combinada con 30% de ácido
tricloroacético y solución Jessner, que nivel de penetración se espera?

A. Epidermis
B. dermis papilar
C. dermis reticular
D. tejido subcutaneo

Respuesta C:

Fundamento:

a) Epidermis- la capa superficial de piel y es penetrada por todos los agentes. Bajas
concentraciones de agentes de descamación superficial pueden ser confinados a esta
capa.
b) Dermis papilar- este nivel de penetración es ATTAINED siguiendo la aplicación de
agentes descamantes superficiales como TCA (10-30%), acido glicólico (10-30%) y
solución Jessner.
c) Dermis reticular. La dermis reticular superior es penetrada por agentes descamantes
por mediana profundidad (TCA 30-50%, Jessner mas 30% TCA, 88% Phenol) y la
dermis reticular media es penetrada por un profundo agente descamativo como
Baker phenol y Littons phenol
d) Tejido subcutáneo, esta profundidad dugeriria una completa THICKNESS lesión en
la piel que pudiera resultar en una significativa cicatrización. No hay agentes
descamativos formulados designados para un nivel de penetración predecible.

FIGURA

Referencias:
Cross section of skin with superficial, medium and deep wound areas: Reproduced from
Monheit GD: Skin rejuvenation procedures. Principles of Oral and Maxillofacial Surgery.
2nd edition, Volume 2, BC Decker Inc, London, P 1420
Nahai F, Baker TJ, Stuzin JM. Chemical Peel. The Art of Aesthetic Surgery. St Louis:
Quality Medical Publishing, 2005, p 386.
Demas PN, Braun TW: Chemical Skin Resurfacing. Esthetic Surgery of the Aging face.
Oral and Maxillofacial Surgery Clinics of North America, Vol6 (2), 1998. P 1

160 Paciente femenina de 38 años de edad presenta eritema y leve induración de la región
perioral y a lo largo del borde inferior de la mandibula, tres semanas después de una
descamación química profunda. Ella menciono en su historia el uso de tretinoin tópico e
isotretinoin por cuatro meses antes de la descamación. Cual de los siguientes es el
diagnóstico mas probable y el tratamiento recomendado,

A. infección virfal, prescriber un agente antiviral


B. infección bacteriana, prescribir antibiotico
C. infección por hongo. Prescriber un agente antifungico
D. cicatriz hipertrofica. Corticoids topicos y o inyectados

Respuesta: D

Fundamento:

esta presentación representa cicatriz hipertrofica. infecciones virales presentan dentro de 2


semanas clusters de ulceraciones dolorosas. similarmente, las infecciones bacterianas son
mas sintomáticas y se presentan con exudado, y erupciones pustular; infecciones fúngicas
son frecuentemente vistas como areas difusas eritematosas y con prurito, y koh staining es
mas usual para diagnosis. pacientes quienes reciben tratamiento con isotretinoin (accutane)
y tretinoin (retin-a) son predispuestas a complicaciones como cicatriz hipertroficad due to
thinness de la piel y reducción de las estructuras anexas. las cicatrices típicas se presentan
como placas eritematosas que están induradas y con apariencia papular que clásicamente
involucra las áreas perioral y borde inferior de la mandíbula. occlusive silicone skin
dressings pueden también ayudar.

Referencias:
Nahai F, Baker TJ, Stuzin JM. Chemical Peel. The Art of Aesthetic Surgery. St Louis:
Quality Medical Publishing, 2005, p 421.
Demas PN, Braun TW: Chemical Skin Resurfacing. Esthetic Surgery of the Aging face.
Oral and Maxillofacial Surgery Clinics of North America, Vol6 (2), 1998. P 21

161. Cual de los siguientes agentes descamativos esta directamente asociado con eventos
cardiacos como multiples contracciones ventriculares prematuros (PVC.s) durante la
reparación de la piel quimicamente?

A. 50% de ácido glicólico


B. Phenol Bakers
C. 35 % de ácido tricloracético
D. solución Jessner

Respuesta: B

Fundamento:
30% -705 de ácido glicolico es usado en una descamación química superficial y no se
conoce que produzca toxicidad cardiáca. El ácido glicolico disminuye la cohesión de
corneocytes en el estrato córneo en bajas concentraciones y causa completa epidermólisis
en altas concentraciones. Phenol Bakers es hepatotóxico, nefrotóxico, cardiotóxico y
depresivo del sistema respiratorio. La descamación con Phenol debe ser manejada de 1 a 2
horas con administración continua de fluidos y monitoreo con EKG. El administrador
continuamente mezcla la solución para asegurar la emulsión y mantener concentraciones
estándares. Aproximadamente 15 minutos son permitidos entre cada unidad aplicada facial,
y los pacientes son ususlmente monitorizados pro dos horas posoperatorias. 35% de ácido
tricloroacetico es usado para una descamación de profundidad media y no es cardiotóxica.
TCA es frecuentemente mezclada con CO2 solido o solución Jessnersy esas terapias
combinadas forman un efectivo tratamiento de photoaging. La solución Jessner contiene
resorcinol, acido salicílico, acido láctico y etanol y no es cardiotóxico. Proporciona una
descamación superficial o de mediana profundidar por si solo o en conbinación con otros
agentes.

Referencias:
Nahai F, Baker TJ, Stuzin JM. Chemical Peel. The Art of Aesthetic Surgery. St Louis:
Quality Medical Publishing, 2005, p 427.
Demas PN, Braun TW: Chemical Skin Resurfacing. Esthetic Surgery of the Aging face.
Oral and Maxillofacial Surgery Clinics of North America, Vol 6(2), 1998. P 8-12
Monheit GD: Skin rejuvenation procedures. Principles of Oral and Maxillofacial Surgery.
2nd edition, Volume 2, BC Decker Inc, London, P 141

162. Cuál de los siguientes es un régimen de tratamiento preoperatorio adecuado en


pacientes bajo reparación de la cara con laser en piel?

A. tretinoin topico para incrementar la actividad fibroblastica y producción de colagena.


B. terapia con acido Alpha hydroxy para inhibir melanocitos e hiperpigmentación.
C. terapia con hidroquinona para aumentrar la penetración de los agents tópicos. D.
Acyclovir para prevenir la formación milia

Respuesta: A

Fundamento:

El pretratamiento con medicamentos específicos es esencial antes de usar la terapia con


cirugía laser. Estos multiples pasos en el régimen están dirigidos especialmente para
preparar la piel y las estructuras asociadas para optimizar la respuesta a la cirugía laser. El
tretinon es usado para normalizar la queratina y aumentar la producción de colágeno
fibroblástico para permitir una mas rápida cicatrización de la herida. Los ácidos alpha
hydroxy son usados para alcanzar queratinolisis para aumentar la penetración de otros
agentes tópicos. La hidroquinona es usada para inhibir la actividad melanocitica y
minimizar la hiperpigmentación inflamatoria. El aciclovir es usado para disminuir el
potencial postoperatorio de infecciones herpéticas. Milia son quistes de retención de
queratina resultado de folículos pilosos cerrados, y no son causados por infecciones virales.

Referencias:
S. Obagi Pre and postlaser skin care. Oral and Maxillofacial Surgery Clinics North
America 16, 2004 181-187.
Chisholm BB: CO2 Laser skin resurfacing for the aging face. Oral and Maxillofacial
Surgery Clinics of North America. Vol 6(8), 1998. P 23-38

163 Diez días siguiendo procedimiento con CO2 para reparación de piel con laser, el
paciente presenta dolor, fiebre, malestar y seguida de erosiones eritematosas no purulentas:

A. milia.
B. infección viral.
C. hiperpigmentación post inflamatoria.
D. infección fúngica.

Respuesta: B

Fundamento:

Rationale:
Milia son quistes de queratina formados por poros obstruidos. No están asociados con
fiebre, malestar u son también generalmente distribuidos por toda la cara. Las lesiones
virales después de la cirugía laser son generalmente erosiones herpéticas. Y el tratamiento
con antivirales hasta que la reepitelización se complete esta indicada para reducir la
incidencia de cicatriz. La hipermigmentacion pos inflamatoria no es dolorosa, y no presenta
fiebre, malestar o erosiones. Una examinación normal en el decimo dia postoperatorio no
debería verse como esto y los pacientes no deberían tener síntomas como fiebre, dolor y
malestar.

Referencias:

P. Demas and J. Bridenstine. Diagnosis and Treatment of Postoperative Complications


after Skin Resurfacing. J Oral Maxillofac Surg 57 (1999) 837- 841.
Chisholm BB: CO2 Laser skin resurfacing for the aging face. Oral and Maxillofacial
Surgery Clinics of North America. Vol 6(8), 1998. P 23-38
164. La siguiente fotografia muestra un paciente con cambios de pigmentación dos semanas
después de terapia laser con CO2. Cual de las siguientes describe el manejo inicial de esta
condición?

A. terapia topica con esteroidesTopical steroid therapy


B. evitar los bloqueadores solares
C. maquillage para camuflagear
D. evitar retinoides tópicos

Respuesta: C

Fundamento:

Cambios temporales de hiperpigmentación después de cirugía laser son mas frecuentemente


encontradas en Fitzpatrick III (35%) y IV (75%) tipos de piel. El tratamiento incluye
aplicación tópica de hidroquinona, retinoides tópicos (tretonoin) y acido alpha hydroxy
(acido glicolico) la hidroquinona actua inhibiendo los melanocitos, mientras el acido
glicolica promueve la queratinolisis. Los retinoides tópicos aumentan la actividad
fibroblástica y la producción de colágeno. La terapia con esteroide no esta defendida. Los
bloqueadores solares y la disminución a la exposición de la luz ultravioleta son
aconsejables. El maquillaje para camuflajear puede ser usado y puede ser requerido en un
largo periodo in los casos de una prolongada o permanente hiperpigmentación.

Reference:
P. Demas and J. Bridenstine. Diagnosis and Treatment of Postoperative Complications
after Skin Resurfacing. J Oral Maxillofac Surg 57 (1999) 837- 841.
Chisholm BB: CO2 Laser skin resurfacing for the aging face. Oral and Maxillofacial
Surgery Clinics of North America. Vol 6(8), 1998. P 23-38

165 Un paciente reporta que cuando su piel es expuesta al sol, ella usualmente se quema, y
se broncea con dificultad. Esta paciente es mejor clasificads dentro del tipo de piel
Fitzpatrick como:

A. I.
B. II.
C. III.
D. IV.

Respuesta: B

Fundamento:

La clasificación de Fitzpatrick es usada en diagnósticos y tratamientos planeados antes de


procedimientos con terapia laser. La clasificación esta basada en factores incluyendo el
color y la complexion de la piel, y su respuesta a la exposición solar. Existen cinco distintas
clases basadas en la respuesta a la exposición solar. Tipo I: la piel siempre se quema y
nunca se broncea, Tipo II: la piel usualmente se quema y broncea con dificultad, tipo III: la
piel exhibe un quemado medio y un bronceado promedio. Tipo IV. La piel levemente
quemada y con facil bronceado. Tipo V: piel muy poco quemada y extremadamente facil
bronceado.

Los pacientes para terapia con laser CO2 incluye Fitzpatrick tipo I y II. Los pacientes tipo
III y IV son mas propensos a complicaciones como hiperpigmentacion después de terapia
laser CO2.

Referencias:
Nahai F, Baker TJ, Stuzin JM. Skin Treatments. The Art of Aesthetic Surgery. St Louis:
Quality Medical Publishing, 2005, P 391.
Chisholm BB. CO2 laser skin resurfacing of the aging face. Esthetic surgery of the aging
face. Oral and Maxillofacial Surgery Clinics of North America. Vol 6(8), 1998. P 32

166 Cual de los siguientes procedimientos es mas apropiado para tratamiento de medio a
moderado rhytids y lleno de manchas generalizado decoloración en una piel Fitzpatrick tipo
II.

A. CO2 laser skin resurfacing


B. Dermoabrasión
C. Microdermoabración
D. Trichloroacetic acid-Jessner.s solution peel

Respuesta: D

Fundamento:
Pacientes Fitzpatrick clase I y II son Buenos candidates para terapia laser CO2, sin
embargo la cirugía laser usualmente indicada para rhytids facial mas severa y mas severa
elastosis solar, y no muy efectiva tratando dyspigmentación de la piel.

La demoabrasión es generalmente hecha con instrumentos rotatorios como un cepillo de


alambre y diamond fraise. La principal indicación de dermoabrasión es cicatriz de acné y
otras cicatrices faciales que involucren areas focales y no generalizadas de la cara. La
técnica de microabrasión es considerada un procedimiento muy superficial desdes que
remueve el estrato corneo y epidermis externa. Esto tiene limitadas indicaciones como
tratamiento de muy finas rhytids y extremadamente leves hiperpigmentaciones. Este es un
procedimiento que tiene que ser repetido cada dos semanas o en combinación con otros
agentes. 35%-50% de acido tricloroacético. Solución Jessner combinada es ideal para tratar
las rhytids faciales de leves a moderadas, daño actínico y decoloración. El daño de
penetración es mejor controlado con esta régimen combinado comparado con TCA solo.

Referencias:
Nahia F, Baker TJ, Stuzin JM. Chemical Peel. The Art of Aesthetic Surgery. St Louis:
Quality Medical Publishing, 2005, p367, 447.
Demas PN, Braun TW: Chemical Skin Resurfacing. Esthetic Surgery of the Aging face.
Oral and Maxillofacial Surgery Clinics of North America, Vol 6 (2), 1998. P 21
Chisholm BB. CO2 laser skin resurfacing of the aging face. Esthetic surgery of the aging
face. Oral and Maxillofacial Surgery Clinics of North America. Vol 6(8), 1998. P 32

167 Cual de los siguientes afirmaciones describe correctamente la técnica Mustarde para
otoplastia?

A. Cartilage excision technique in a stepwise fashion to reduce a hypertrophic conchal wall


B. acercamiento postauricular para remover vestigios del musculo auricular posterior y su
ligament debajo de la fascia mastoidea.
C. creación de un dobles del antihelix por cartilage weakening and mattress sutures
D. Creation of a new antihelical fold at the expense of sharp cartilaginous ridges seen
through
the thin anterior auricular skin

Respuesta: C

Fundamento:

La técnica Mustarde involucra debilitar el cartilage y colocarluna serie de suturas


horizontals para crear un dobles del antihelix, la tecnica Davis corrige la hipertrofia conchal
por una excición de cartílago. La Técnica Converse-Wood Smith involucra varios cortes de
espesor completo a través del cartílago y suturando al dobles del antihelix. Esta afilada
cresta puede frecuentemente ser sentirse y verse atraven de la delgada piel anterior. La
técnica Furnas tiene alta incidencia de recidiva y su acercamiento posauricular, donde el
musculo es removido y suturado a la fascia mastoides al cartílago del oído.
Referencias:
Owsley TG, Tejera TJ: Otoplastic surgery for the protruding ear. In: Fonseca R, Baker, S,
Wolford LM (eds). Oral and Maxillofacial Surgery. Vol 6, WB Saunders, Philadelphia
2000, p 408
Bauer BS: Correction of the constricted ear. Plast Surg Tech 1:2, 153-160, 1995

168 Pericondritis después de otoplastia es mas comunmente causada por cual de los
siguientes organismos:

A. streptococo piogenes, Escherichia coli, Hemophylus influenza.


B. streptococcus pyogenes, Neisseria gonorrhea, Bacteroides species.
C. staphylococcus aureus, Hemophylus influenza, Bacteroides species.
D. staphylococcus aureus, Escherichia coli, Pseudomonas aerugionsa.

Respuesta: D

Fundamento:

Pericondritis ocurre en el period temprano postoperatorio, y es usualmente relacionado en


un hematoma inadecuado y sin detectar. Síntomas incluyen dolor, eritema, fiebre. Heridas
en cartilage pueden ser lentas para cicatrizer y podran demander una terapia mas agresiva
incluyendo un debridamiento y drenaje quirúrgico. Mientras el cultivo es recomendado,
terapia empirica de condritis auricular podría incluir cobertura para pseudomonas, la cual
puede ser la causa mas común. Dando opciones limitadas para terapia, ciprofloxacina oral
ha sido recomendada como el medicamento de elección. Algunos recomiendan la
dicloxacilina o cefalexina como tratamiento inicial. Infecciones significativas pueden
justificar el tratamiento de antibióticos intravenosos. Destrucción masiva de cartílago y
severas deformidades del oído pueden resultar como resultado de tratamiento agresivo.

Referencias:
Owsley TG, Tejera TJ: Otoplastic surgery for the protruding ear. In: Fonseca R, Baker, S,
Wolford LM (eds). Oral and Maxillofacial Surgery. Vol 6, WB Saunders, Philadelphia
2000, P 408
Tanzer, RC: Congenital deformities, Deformities of the auricle. In: Reconstructive Plastic
Surgery, 2nd edition, W.B. Saunders, Philadelphia, 1997, P 1671
169 La principal inervación sensitive de la auricular es vía el:

A. nervio auriculotemporal
B. plexo cervical
C. lesser occipital nerve.
D. greater auricular nerve.

Respuesta: D

Fundamento:

La sensibilidad nerviosa la da principalmente de las ramas anteriores y posteriors del gran


nervio auricular. Este nervio viaja 8mm posterior al pliegue postauricular. El gran nervio
auricular originada del segundo y tercer nervio cervical, va alrededor del borde posterior
del esternomastoideo, y después perfora la fascia profunda, asciende hacia ese músculo
bajo el platisma dentro de la glandula parótida, donde se divide en rama anterior y
posterior.la rama posterior (mastoidea) inerva la piel sobre el proceso mastoideo y en la
parte posterior de la auricula, excepto en su parte superior, un filamento atraviesa la
auricula y alcanza la superficie lateral donde se distribuye al lóbulo y la parte inferior de la
concha. Estos proveen la inervación sensitiva para la piel sobre la glándula parorida y el
proceso mastoideo, y ambas superficies de la parte externa del oído. El nervio
auriculotemporal y en menor grado el occipital suministran a la cavidad de la concha y el
meato auditivo externo y ramas auriculares del vago puede suministrar a la pared posterior
del meato auditivo externo.

Referencias:
Oswley, TG, Tejera TJ: Otoplastic surgery for the protruding ear. In: Fonseca R, Baker S,
Wolford LM (eds). Oral & Maxillofacial surgery Vol 6, WB Saunders, Philadelphia 2000.
P 408
Tanzer, RC: Congenital deformities, Deformities of the auricle. In: Reconstructive Plastic
Surgery, 2nd edition, W.B. Saunders, Philadelphia, 1997, P 1671

170 Cual de los siguientes es un mecanismo mas importante para dar soporte nasal?

A. tamaño y forma de los huesos nasales


B. Medial crural feet attachment to the nasal septum
C. espina nasal anterior
D. grososr del tejido blando del ala de la nariz

Respuesta: B
Fundamento:

Los mecanismos de soporte nasal pueden ser divididos dentro de primaries y secundarios:
Primario/ mayor o principal
1. Accesorio al septum caudal y medial crura(interrumpido por una completa incisión
transfixión)
2. Accesorios fibrosos entre el cartílago nasal superior e inferior (interrumpido por las
incisiones intercartilaginosas)
3. Tamaño, forma de los cartílagos laterales inferiores
Secundario/menor
1. Ligamento interdomal
2. Complejo sesamoideo, el cual in effect attaches, el cartílago lateral inferior del
borde piriforme
3. Dorso septal cartilaginoso
4. Espina nasal anterior
5. Membrana septal
6. Cartílago alar con su overlying skin

Referencias:
Kennedy BD: Indications and Techniques for Rhinoplasty. Principles of Oral and
Maxillofacial Surgery. Peterson LJ (ed). JB Lippincott Co, 1992. P 1724-5
Koehler J, Waite PD: Basic principles of rhinoplasty. Principles of Oral and Maxillofacial
Surgery, 2nd ed, BC Decker Inc., London 2004, p. 1350

171) cual de las siguientes afirmaciones describe correctamente las técnica de Mustarde
para otoplastía?
A. la técnica de escisión del cartílago en un amplio paso estético para reducir una
hipertrofia de la pared de la concha.
B. un abordaje postauricular para remover el vestigio del musculo auricular posterior y su
ligamento por debajo de la fascia mastoidea
C. creación de un nuevo dobles antielicoidal del cartílago debilidato y sutura de colchonero
D. creación de un nuevo dobles antihelicoidal a expensas del puente cartilaginoso agudo
vistos a través de la piel auricular anterior adelgazada.
RESPUESTA: C
Análisis : la técnica de Mustarde involucra el adelgazamiento del cartílago y reposición por
una serie de suturas de colchoneros horizontales para crear un dobles antihelicoidal. La
técnica de Davis corrige hipertrofia de la concha por medio de una amplia incisión del
cartílago. La técnica de Converse - Wood - Smith involucra un corte de espesor total en el
cartílago y suturarlo formando un dobles antihelicoidal. Este puente agudo puede a veces
verse o sentirse atreves de la delgada piel anterior. La técnica de Furnas tiene un alto
índices de recidiva y es un abordaje post auricular, donde el musculo es removido y
suturado de la fascia mastoide al cartilago de la oreja.
172 )La pericondritis luego de una otoplastía es mas común causada por cual de los
siguientes microorganismos:
A. Estreptococos piógenos, Echericha coli, Hemofilius influenzae
B. Estreptococos piógeno, neisseria gonorreae, Bacteroides
C. Estafilococos aureus, Echericha coli, Pseudomonas aeruginosa
Respuesta : D
Análisis: la pericondritis ocorre en el periodo postoperatorio temprano, y es usualmente
relacionado un hematoma sin detectar o mal tratado. Los síntomas incluyen dolor, eritema,
fiebre y descarga. La herida del cartílago puede curarse lento y demandar una terapia
agresiva, incluyendo debridamiento quirúrgico y drenaje. Mientras se recomiendan en las
culturas, la terapia empírica de la condrosis auricular puede incluir la cobertura por
Pseudomonas, que pueden ser la causa mas común. La opciones dadas son limitadas para
terapia de pacientes ambulatorios, ciprofloxacin oral, es recomendado como la droga a usar.
Otros recomiendan dicloxacilina o cefalexin oral como tratamiento inicial. Las infecciones
significativas pueden necesitar el tratamiento antibiótico intravenoso. La destrucción
masiva del cartílago y las deformidades severas del oído pueden resultar, a pesar de un
tratamiento agresivo
173) La inervación sensorial principal de la aurícula es por vía:
A. nervio auriculotemporal.
B. plexo cervical.
C. nervio occipital menor.
D. nervio auricular mayor.

Respuesta ; D
Analisis: La fuente sensorial del nervio es sobre todo de las ramas anteriores y posteriores
del nervio auricularmayor. Este nervio viaja 8 milímetros por detrás al pliegue
postauricular. El nervio auricular mayor se origina del segundo y los terceros nervios
cervicales, viaja alrededor de la frontera posterior del esternomastoideo, y, después de
perforar la fascia profunda, ascienden sobre ese músculo debajo del platisma en la glándula
parotida, donde se divide en ramas anteriores y posteriores. La rama posterior (mastoidea)
inerva a la piel sobre el proceso mastoide y en la parte posterior de la aurícula, excepto en
su parte superior; un filamento perfora la auricula para alcanzar su superficie lateral, donde
se distribuye al lóbulo y a la parte más inferior del concha. Así, proporciona la inervación
sensorial para la piel sobre la glándula parotida y el proceso del mastoide, y ambas
superficies del oído externo. Los Nervios auriculotemporal y los nervios occipitales
menores proveen a la cavidad conchal y el meato auditivo externo, y las ramas auriculares
del nervio vago pueden inervar a la pared posterior del meato auditivo externo.
174) cual de los siguientes es el mayor mecanismo de soporte de la punta nasal?
A. El tamaño y agudeza de los huesos nasales
B. El piso de la crura medial insertada al septum nasal
C. Espina nasal anterior
D. El grosor del tejido blando del ala
Respuesta: B
Análisis: Los mecanismos de soporte de la punta nasal se pueden dividir en mecanismos
primarios y secundarios:
Primario/mayor:
1. la inserción de la parte caudal del tabique y la crura medial (interrumpidos por
una completa incisión)
2. inserciones fibrosas entre los cartílagos nasales superiores e inferiores laterales
(interrumpidos por las incisiones intercartilaginosas)
3. Tamaño, forma, y fuerza de los cartílagos inferiores laterales
Secundario/menor:
1. Ligamento interdomal
2. Complejo Sesamoideo, que, en efecto une los cartílagos inferiores laterales a la
escotadura piriforme.
3. Dorso septal cartilaginoso
4. Espina nasal anterior
5. Septum membranoso.
6. Cartílagos alares con su piel sobrepuesta

175) Esta deformidad del techo nasal abierto es corregida mejor por:

A. Septoplastia nasal
B. Reposisionamiento del injerto protector
C. Osteotomias nasals laterals
D. Afeitar los cartilagos laterales superiores

Respuestas: C
Analisis: Una deformidad abierta del techo se crea durante una cirugía de reducción de la
jiva dorsal. Da el aspecto de un puente nasal amplio, aplanado por la visita frontal. Ésta
condición puede ser corregida con las osteotomías nasales. Las osteotomías se realizan
bilateral y el puente/dorso nasal es adelgazado sin fracturar de los huesos nasales. Las
osteotomías laterales bilaterales son de uso comun, y las osteotomías de vez en cuando
intermedias pueden también ser realizadas. El cuidado se debe tomar durante el tratamiento
es el sobre-adelgazamiento que puede causar obstrucción nasal. Colocar el tabique nasal de
nuevo o una septoplastia no tiene ningún efecto en la eliminación de esta condición. Los
injertos protectores se utilizan para la definición de la punta nasal. El afeitar los cartílagos
laterales superiores puede empeorar la deformidad abierta del techo.

176)este paciente require adelgazamiento y definicion de su punta nasal. Este resultado


puede ser mejor obtenido por:

A. Rinolastia externa con injerto protector


B. Septoplastia con Weir excision procedure
C. Rinoplastia interna con osteotomias nasals laterals
D. Incisión de transfijacion y bajar el ángulo septal
Respuesta: A
Análisis: La punta nasal es una estructura suave del tejido fino. Las alteraciones y los
refinamientos de la punta nasal son logrados lo mejor posible por cirugía de tejidos blandos
y con los injertos de cartílago cuidadosamente puestos. Una técnica abierta da mayor
acceso quirúrgico y visibilidad directa, y se indica especialmente para los casos complejos.
Implica incisiones marginales bilaterales y una incisión transcolumnelar seguida por la
esqueletinización y la exposición. Las quejas de este paciente se tratan mejor con una
rinoplastia externa, adorno del cartílago, y la colocación de los injertos modificados
particulares del cartílago para definir la punta nasal (véase la figura abajo)
Un procedimiento cerrado es usado para corregir el ancho de la base alar, especialmente
reducción de la anchura de la base alar. Esta insicion involucra un abordaje pequeño en la
mucosa vestibular y la piel. La insicion es usualmente conservadora y rara vez mayor a 3
mm de ancho. La osteotomía nasal lateral puede adelgazar la nariz y corregir los defectos
oseos pero no las simetrías de las deformidades de la punta nasal. Ellos son indicados para
adelgazar la nariz cuando el ancho del dorso y el puente es mas de 80% del ancho de la
base alar de la nariz. Disminuir el angulo septal con una incisión transfijacion completa
puede reducir la proyección de la punta y produce efectos indeseables.

177)la coneccion fibrosa entre el cartílago lateral superior y el cartílago lateral inferiro de la
nariz es llamado:

A. scroll area. Area de insercion entre ambos cartilagos


B. rinion.
C.valvula nasal interna
D. quiebre supratip

Respuesta : A
Analisis: el scroll es el área de insercion de los cartílagos laterales inferiores y laterales
superiores. Cuatro configuraciones son comunes: enclavijado (el 52%), traslapado (el
20%), extremo-extremo (el 17%), y opuesto (el 11%). El scroll proporciona una ayuda
importante a la punta nasal y una incisión intercartilaginosa durante una rhinoplastia
endonasal el procedimiento viola esta área, de tal modo teniendo efectos en la posición de
la punta nasal. El rinion es la ensambladura entre los huesos nasales y el cartílago lateral
superior, es decir, osea y cartilaginosas del puente nasal. La cubierta del tejido blando y la
piel de la nariz es las más finas a este punto. La válvula nasal interna es formada por la
ensambladura del borde caudal de los cartílagos laterales superiores con el tabique dorsal.
El valor normal de este ángulo es aproximadamente 10 grados, y adelgazar la válvula
puede tener efectos nocivos en la cirugía nasal funcional de la vía aérea causando
obstrucción nasal. La rotura de la supratip es una depresión dorsal sutil apenas cefálica a la
punta nasal en la ensambladura del dorso nasal y de la extremidad nasal.

178)la esqueletinizacion de la nariz con un abordaje externo de rinoplastia es conseguida


por el uso de cual insicion bilateral?
A. Marginal
B. Transfijacion
C. Intercartilaginoso
D. Killian
Respuesta : A
Análisis: Las incisiones marginales bilaterales son conectadas por una incisión
transcolumnelar para formar las incisiones básicas para un abordaje externo de rinoplastia.
Esta conexión permite la exposición a los cartílagos laterales inferiores y el dorso nasal.
Una incisión marginal es paralela a los bordes caudales de los cartílagos laterales inferiores.
Implica la colocación de una incisión curva a pocos milímetros de los márgenes de la
abertura nasal. Una incisión de transfijacion se realiza en la ensambladura de la parte
inferior del tabique nasal y de la parte superior de los muslos intermedios, y se puede hacer
parcialmente o totalmente o solamente en un lado (hemitransfijacion). Una incisión
intercartiliginosa forma la base de un abordaje interno (endonasal) de rinoplastia y se hace
entre el margen cefálico del cartílago lateral inferior y el margen caudal del cartílago lateral
superior. La incisión de Killian no se utiliza en rinoplastia, sino es un diseño tradicional de
la incisión para septoplastia nasal. Refiere a una incisión hecha varios milímetros cefálicos
al borde caudal del tabique, y puede ser extendido al piso nasal, si procede.

179 ) el recorte cefálico del cartílago lateral inferior de la nariz durante la rinoplastia tiene
cual de los siguientes efectos en la punta nasal:
A. Rotación superior
B. Rotación inferior
C. Ensanchamiento
D. Sin efecto

Respuesta : A
Análisis: Aunque se acentúa la preservación del cartílago, la resección del cartílago se
requiere en ciertas circunstancias. Hay 3 principios de base de la supresión del cartílago en
la región nasal de la extremidad: técnica de complete strip (figura a), una técnica de
complete strip debilitada, y una técnica strip interrumpida. (Figura b) . luego el retiro del
strip, por lo menos 6 milímetros ancho al cartílago lateral inferior se debe conservar para
asegurar una adecuada posicion de la punta.
El recorte cefálico de los cartílagos laterales inferiores proporciona para alguna
rotación ascendente de la punta asi como del borde cefálico y su conexión a los cartílagos
laterales superiores es retirada. La rotación es grandes con técnica de stripinterrumpida.
Además, el recorte cefálico también adelgaza la punta nasal, de tal modo le da más
definición y refinamiento. Dependiendo de la extensión medial de la escisión en el área del
domo, el ajuste cefálico puede causar realmente una proyección suave de la punta nasal.
180)cual de los siguientes pares de deformidades nasales y tratamiento indicado ( de
injerto de cartílago) de las opciones es correcta?
A. Pobre soporte de la punta nasal. Injerto extendido
B. Colapso de la valvula nasal interna. Injerto strut de columnela
C. Deformidad de nariz en silla de montar. Injerto en paraguas
D. Punta nasal amorfa. Injerto protector

Respuesta: D
Análisis: el soporte de la punta nasal es realzada por los injertos columnelares, que se
pueden poner entre las cruras intermedias derecha e izquierda de los cartílagos laterales
inferiores, y lindar por la espina nnasal. La válvula nasal interna es esencial para una buena
vía aérea funcional, y si está angosta, se puede abrir/mejorar con los injertos esparciadores,
que se ponen entre el tabique nasal y los cartílagos laterales superiores. Un mínimo de
ángulo de 10-15 grados se debe mantener en el área de la válvula para asegurar la evidencia
de la vía aérea. Un injerto de la punta tal conasal asi como un injerto protector se utiliza
para dar más definición a la punta nasal amorfa. Los injertos protectores consisten en un
injerto formado trapezoidal del cartílago que se sutura en el área de la punta. En los
márgenes superiores se hacen muescas haciendo un abordaje cerrado de rinoplastia utiliza
incisiones marginales y el desarrollo de un bolsillo de tejido blando anterior e inferior a los
muslos intermedios. Con una técnica abierta, el injerto se sutura directamente al área
domal. Los injertos en paraguas refieren a una combinación de un injerto columenelar y de
un injerto de la punta. Se indican estos injertos en caso de que haya carencia de la
definición de la punta y soporte de la punta debido a la debilidad en la región de los muslos
intermedios. Una deformidad de la nariz en silla de montar es corregida por el aumento con
cartílago o materiales aloplasticos, pero no con los injertos en paraguas
181) paciente que se presenta con obstruccion nasal un año después de sufrir un trauma
nasal. Al examen intranasal revela desviación septal y dislocación de la espina nasal .
colapso bilateral de las válvulas internas y externas y se nota hipertrofia de la turbina
inferiror derecha. Durante la septoplastia, el cirujano debe:
A. Remover topdas las porciones de cartilage desviado y del septum oseo para mejorar
la via aérea
B. No tomar el cartílago septal y oseo para injertos por la etiología postraumática
C. Dejar un adecuado dorso y base de cartílago para el soporte del dorso y la punta
nasal
D. Insidir ambos lados de la mucosa para crear una comunicación entre las fosas
izquierda y derecha para un adecuado enderezamiento septal

Respuesta: C
Analisis: El cirujano debe mantener una L-puntal del cartílago para el soporte nasal.
Generalmente, aproximadamente 1 centímetro de cartílago se deja en los aspectos dorsales
y caudales para mantener el soporte. Todas las porciones del tabique desviado no necesitan
ser quitadas; en lugar, uno puede tomar o quitar selectivamente el cartílago para enderezar
las porciones restantes. En este paciente, el cartílago y el hueso obtenido del tabique
tendrán valores para que el injerto mejore la vía aérea funcional con los injertos del
cartílago. Es siempre ventajoso dejar al menos un lado de la mucosa septal intacto para
reducir al mínimo la incidencia de perforaciones septal.
182) que injerto mejora la función nasal externa de la valvula
A. Dorso nasal
B. protector
C. Alar batten
D. Columenelar strut

Respuestas: C
Análisis: El aumento del dorso de la nariz no tiene efectos beneficiosos directos sobre la
función de la vía aérea nasal. Semejantemente, un injerto de camuflaje sobre la pirámide
ósea nasal izquierda o un injerto protector de la punta puede ayudar a mejorar asimetría
residual pero no mejorará la función de la vía aérea. Los injertos esparcidores pueden
mejorar la válvula nasal interna, y los injertos alar en los pliegues del supra-alar mejorarán
la función nasal externa de la válvula. Además, un injerto columnelar del puntal puede
proporcionar cierta elevación y soporte a la punta nasal y debe mejorar posteriormente la
función a largo plazo de la vía aérea en este paciente con pobres soporte de puna.
183) este paciente se presenta con una deformidad nasal post-traumatica. Cual de las
siguientes es correcta con respecto a osteotomías de hueso nasal en estos pacientes?
A. Comenzar osteotomías laterales debajo de la turbina inferior para prevenir un
colapso de la via aérea
B. Evitar extender las osteotomías laterales en la raíz para prevenir una deformidad
“rocker”
C. Evitar hacer osteotomías laterales y mediales por la historia del trauma
D. Hacer una osteotomía intermedia en el lado izquierdo para ayudar a obtener simetría
E.
Respuesta: B

Análisis: Es recomendable evitar extender la osteotomía lateral en el hueso nasal grueso en


la raíz porque éste puede causar una deformidad “rocker”. Comenzar la osteotomía lateral
sobre la turbina inferior preserva el hueso triangular en el borde piriforme y previene el
derrumbamiento de la turbina inferior en la vía aérea. Las osteotomías intermedias
permitirían la movilización libre y completa de cada lado de la pirámide osea y mejorarían
la asimetría. Una osteotomía intermedia en el lado derecho de este paciente (el lado más
largo) y no el lado (más corto) izquierdo permitiría el acortamiento de la pirámide nasal y
haría la nariz más simétrica

184) el concepto de trípode en rinopkastia se refiere a:


A. Crura medial y cartílagos laterales superiores
B. Crura medial y septum (caudal)
C. Crura media y crura lateral bilateral
D. Crura lateral bilateral y scroll
Respuesta: C
Analisis:el concepto es muy importante en rinolastia. La crura medial y la crura lateral de
los cartílagos laterales inferiores forman el trípode nasal. La posicion natural de la nariz es
inclinada como el miembro formado por la crura media unida es siempre mas corta que los
otros dos miembros, que son formados por las cruras laterales. La modificación de uno de
estos miembros causa efectos significativos en la posición y el perfil de la punta nasal. El
conociomiento de este concepto permite precisar las alteraciones en la proyección de la
punta nasal y rot ación de la punta nasal. Por ejemplo, acortar uno de los miembros largos
(crura lateral) puede acortar la nariz, y alargar el miembro más corto (crura medial) puede
aumentar la proyección de la punta.
185) idealmente, el ancho de la base alar en la población blanca podría estar entre cuantos
mm de dsitancia inter cantal:
A. 1 – 2
B. 3 – 4
C. 5 – 6
D. 7 – 8
Respuesta: A

Análisis: el ancho de la base alar es relacionado con la distancia intercantal, y debe ser
entra 1 – 2 mm de distancia intercantal para un balance facial optimo del tercio medio de la
cara el aumento del ancho de la base alar puede significar elongación maxilar o posision
posterior de la cara media. La distancia normal intercantal es 34 +/- 4 mm en caucásicos, y
puede variar con la raza. Generalmente individuos de piel osacura (NEGROS) tienen
válvulas largas (telecantismo). El ancho del cuerpo de la nariz y la punta son aprox. 80 %
del ancho de la base alar.
186) la distorsión y el adelgazamiento de la abertura palpebral horizontal normal ocurre
debido a:
A. Debilidad del septum orbital
B. Laxidad de los tendones cantales media y lateral
C. Prolapso de la glándula lagrimal
D. Déficit de tonicidad de la piel inferior de ala
Respuesta B

Análisis: La debilidad del septum orbital y el prolapso graso causará el aspecto de una ala
más baja “bolsa” sin efecto sobre abertura horizontal. En cambio, la laxicidad o la
dehiscencia de los tendones cantales conduce a una disminución de la distancia entre las
comisuras intermedios y laterales, de tal modo causando no sólo adelgazamiento de la
abertura palpebral horizontal pero también de la dislocación inferior del margen más bajo
del ala y el redondeo de los ángulos cantales. El prolapso de glándulas lacrimales conduce a
la distorsión en el aspecto lateral del ala en la parte superior sin efecto sobre abertura
horizontal. La pérdida de tonicidad de la piel más baja del ala causará una tapa ala más baja
“bolsa” sin efecto sobre abertura palpebral.

187) esta condición es llamada:


A. dermatocalasis.
B. blefarocalasis.
C. pseudoptosis.
D. esteatoblefaron.

Respuesta: A
Análisis: la dermatocalasis se refiere al exceso de la piel del parpado superior. Esta se ve
con el envejecimiento debido al colágeno y interrupción de la fibra elastica. La
Blefarocalasis se refiere a una condición inflamatoria rara, recurrente de los párpados
superiores en los pacientes jóvenes que dan lugar a la redundancia de la piel del parpado
superior debido a la relajación. Pseudoptosis es una condición adonde la ceja cae y
consecuentemente, el párpado superior se puede desplazar inferiormente; esta condición se
asemeja a una “real” ptotis. Sin embargo, las dos condiciones se deben distinguir como el
párpado ptotico generalmente necesita cirugía de elevación de la aponeurosis, pero el
parpado pseudoptotico primero necesita elevación de la frente. La Esteatoblefaron se
refiere a bombear el tejido postseptal de la grasa orbital debido a un tabique orbital
debilitado, dando por resultado un”párpado en bolosa” defecto cosmético.

188) la distancia del margen reflejo 1 (MRD-1) se mide entre:


A. Del margen del parpado al reflejo luminoso corneo en mirada fija primaria
B. Del margen de la ceja al reflejo luminoso corneo en mirada fija
C. Del margen superior de la frente al margen de la frente inferior pasando por reflejo
luminoso corneo en mirada fija
D. Del reflejo luminoso corneo en mirada fija a la comisura cantal lateral

Respuesta: A
Analisis: El MRD se considera uno de los indicadores más sensibles de la ptosis del
parpado superior, y es de importancia suprema en la evaluación preoperatoria. Puede
ayudar a distinguir la ptosis del parpado de blefarocalasis, pues estas condiciones requieren
diversos tratamientos para la corrección. El MRD se puede dividir en MRD-1 (parpado
superior al reflejo córneo) y MRD-2 (parpado inferior al reflejo córneo). MRD-1 normal es
2-5 milímetros (véase la figura) y MRD-2 normal es 5-6 milímetros. Si MRD-1 > 5
milímetros, puede ser indicativo de tirotoxicosis; valores < 2 milímetros se consideran en
ptosis del parpado.

189) paciente de 56 años se presenta para evaluación del parpado inferior inchado. Que
pruebas se pueden hacer para diferenciar entre prolapso de grasa y edema?

A. Prueba de distracción de parpado


B. Prueba de parpadeo
C. Aspiración por aguja
D. Ballottement
Respuesta:D
Analisis: La diferenciación fácil de la grasa orbital del edema es alcanzada por el
ballottement apacible del párpado inferior; La técnica del ballottement implica el uso de la
presión digital apacible al globo, de tal modo desplazando el globo posteriormente. Si la
plenitud es debido al prolapso graso, el ballottement del globo dará lugar al movimiento
anterior distinto en plenitud; al contrario, en caso de que la etiología sea por el edema, el
ballottement del globo no conduce a la dislocación anterior enplenitud. Además, se le
puede pedir al paciente mirar en una dirección ascendente; si el prolapso graso a través del
tabique orbital es la etiología de la inchason, el que mire superiormente acentuará la
condición. Incluso en casos de edema intersticial severo, la posibilidad de aspirar el líquido
con una aguja es casi mínima, así que esto no es recomendable. La prueba rápida se utiliza
para determinar el riesgo de ectropion después de cirugía del párpado; el parpado inferior se
tira lejos del ojo y se permite un “snap-back”, y el retraso indica un riesgo más alto de
ectropion. Un parpado normal vuelve a su posición en el plazo de aproximadamente 1-3
segundos. La prueba de la distracción del parpado se utiliza para determinar la laxicidad de
los ligamentos del párpado inferior. Aquí el parpado se tira lejos del globo; en un párpado
normal, la tapa se puede tirar lejos solamente menos de 6 milímetros.

190) cuantos bolsas grasas están presentes en el parpado superior?


A. 1
B. 2
C. 3
D. 4

Respuesta: B
Análisis:el parpado superior consta de 2 bolsas grasas: la medial y la central que pueden ser
removidas durantela blefaroplatia del parpado superior. El parpado inferior consta de 3
bolsas grasas; medial, central y lateral. En el parpado superior el compartimiento lateral es
ocupado por la glandula lagrimal, que puede no ser maltratada durante la blefaroplastia del
parpado superior.

191. Las líneas verticales glabelares del ceño son causadas por la acción de los músculos?

A. Frontal
B. Procer
C. Corrugador
D. Orbicular

Justificación: C

El músculo frontal subgaleal se origina en el plano del cuero cabelludo y se inserta en el


orbicular del ojo. A diferencia de los múltiples depresores de la ceja, el músculo frontal es
el único y verdadero elevador de ceja. Esto mantiene hacia arriba la posicion de la frente
(ceja), pero puede causar arrugas horizontales con el paso del tiempo. Líneas verticales
glabeles ceño normalmente son causados por
la onduladora músculo. Este músculo se origina hacia el hueso frontal justo encima de los
huesos nasales y las inserciones en la dermis de la parte superior del frente. Tiene 2 caras
oblicua y transversal, la cual tira de la mitad de las cejas hacia la línea media con una ligera
depresión causando vertical y oblicuos surcos, respectivamente. Rhytids horizontal de la
región media son causadas por el
músculo procer, que se orgina más de la parte inferior y superior de los huesos nasales y
arriba de los cartílagos alares laterales, y se inserta dentro de la dermis glabellar. Juntos, el
corrugador y el procer son los principales músculos depresores de la ceja y son los
músculos más comunes tratados con toxina botulínica A para aliviar las líneas frontales de
la región glabelar. Estos son los mismos músculos que también son operados en ceja y
levantar la frente. La parte orbital del origen del orbicular del ojo se origina en el tendón
cantal medio y alrededordel hueso, y se inserta en las porciones adyacentes de los
depresores, el frontal, y la dermis por debajo de la frente.

Reference:
Wieder JM, Moy RL. Understanding Botulinum Toxin: Surgical Anatomy of the Frown,
Forehead, and Periocular Region. Dermatologic Surgery, 1998, 24, 1172-74.
Cuzalina A: Forehead and Brow procedures. Principles of Oral and Maxillofacial Surgery.
BC Decker, London. 2004. P 1387

192.- Esta condición se ha producido después deltratamiento con la toxina botulínica A


(Botox) de la región periorbital.
La etiología más probable es que el tratamiento inadecuado de que músculos
A. Elevador
B. Muller 's
C. Orbicular
D. Frontal

Respuesta: A

Justificación:
El paciente tiene lid ptosis. El elevator palpebrae superior es el principal retractor de lid.
Surge desde el ala menor del esfenoides, en la profundidad de la órbita ósea, y en los cursos
superiores músculo recto y forma con una aponeurosis con un importante hueso, un
conjuntotarsal cutáneo. La verdadera lip ptosisseguida de tratamiento de Botox se debe a
un tratamiento inadecuado del músculoelevador palpebral superior. Esta complicación suele
ser auto-corregible en 3-6 meses una vez que los efectos del Botox desparecen. El músculo
Muller tiene inervación simpática, se implanta por debajo de la superficie músculo
elevador, y atribuye al tarso. Proporciona aproximadamente 2 mm de elevación del párpado
superior, sobre todo cuando el sistema simpático es estimulado. El músculo orbicular tiene
tres partes: pretarsal, preseptal y orbitaria. El pretarsal y preseptal son los responsables de
reflejo del cierre del parpado. La toxina botulínica ocasiona la parálisis incompleta del
músculo frontal es a menudo elegido para lograr un efecto natural, por lo que algunas
expresiones de ánimo son mantenidas sobre todo de la frente eliminando las líneas de la
tristeza.
Sin embargo, la acción de los activos residuales del musculo frontal puede causar un
excesivo abultamiento de cejas.

Referencia:
Coleman Moriarty K. Botulinum Toxin in Facial Rejuvenation. Mosby, 2004, 47-48.
Cuzalina A: Forehead and Brow procedures. Principles of Oral and Maxillofacial Surgery.
BC Decker, London. 2004. P 1387

193.- En los hombres, durante el levantamiento quirúrgico de frente, el borde inferior de


todo el frente es el mejor posicionado ____mm por encima del borde inferior de la parte
superior del reborde.
A. 2
B. 4
C. 6
D. 8
Respuesta: A

Justificación:
Las cejas deben formar una elegante curvatura por encima del borde supraorbitario. En las
mujeres, la terminal es aproximadamente 1 cm por encima del borde supraorbitario, con la
mayor altura de curvatura de la ceja en la línea lateral del limbo. Normalmente, la ceja se
divide en 3 regiones diferentes en las mujeres: media 1 / 3, el ápice, y la cola. La porción
medial es de 1-2 mm por encima de la frontera inferior del reborde supraorbitario. El
vértice está situado 8-10 mm por encima de la frontera inferior supraorbitaria lateral, y la
cola se encuentra 10-15 mm desde el borde anterior del reborde supraorbitario. En los
hombres, la lip supraorbitario es más pronunciada, con grandes cejas, más orientada
horizontalmente con menos curvatura en comparación con las mujeres.

Figures reproduced from: Evans TW: Browlift. In: Esthetic Surgery of the Aging Face.
Hupp JR (ed). Atlas of Oral and Maxillofacial Surgery Clinics of North America. Vol 6(8),
1998.

Reference:
Alexander RW: Cosmetic Surgery of the Periorbital Area: Blepharoplasty, Brow Lifting
and Forehead Lift. Oral Maxillofac Surg Clin of N Am 2(2): 413-423, 1990.
Cuzalina A: Forehead and Brow procedures. Principles of Oral and Maxillofacial Surgery.
BC Decker, London. 2004. P 1387
Evans TW: Browlift. In: Esthetic Surgery of the Aging Face. Hupp JR (ed). Atlas of Oral
and Maxillofacial Surgery Clinics of North America. Vol 6(8), 1998. P 23-38

194.- La incisión coronal utilizada en un levantamiento de cara es mejor utilizada en:

A. paciente varon con la línea del pelo en media frente.


B. paciente femenina con línea del cabello media frente.
C. paciente varón con un alto cabello.(implantación)
D. paciente femenina con implantación alta

Respuesta: B

Justificación:
levantamiento de ceja coronal es uno de los primeros procedimientos de levantamiento de
cejas y frente. Un incisión se hace en el cabello (la incisión se realiza en paralelo al bisel de
folículos pilosos, pero que podían dar lugar a la pérdida del cabello) 1-3 cm detrás del
cabello. La disección se realiza en un plano subgaleal subperióstico o que luego se conecta
a un plano lateral subtemporoparietal disección. Cuero cabelludo, las exisiones de tejido se
realiza para elevar las cejas.
La incisión coronal es el mejor indicado para las mujeres u hombres no calvicie con un
medio a baja implantación de cabello. Trichophytic o pretrichial incisiones son mejor
utilizados en pacientes con alta implantación de cabello, donde no es deseable alargar la
frente / cabello.
la incisión Pretrichial se realiza en frente de los cabellos y deja una cicatriz en la frente
delante de los cabellos. En cambio, la incisión se hace trichophytic justo detrás de los
cabellos, que es biselada de modo que la parte delantera de los folículos en los cabellos
sobrevive, y el cabello crece de escondiendo la cicatriz de la incisión. Generalmente, es
aconsejable evitar incisiones visibles en el cuero cabelludo en hombres calvos (líneas media
y alta frente), y procedimientos endoscópicos son generalmente la mejor opción en estos
casos.

Reference:
Larrabee WF: Browlifts. In Atlas of Head & Neck Surgery-Otolaryngology. BJ Bailey
(ed) Lippincott-Raven, Philadelphia, 1996, pp. 502-505.
Cuzalina A: Forehead and Brow procedures. Principles of Oral and Maxillofacial Surgery.
BC Decker, London. 2004.
Evans TW: Browlift. Oral and Maxillofacial Surgery Clinics of North America.
Vol 6(8), 1998. P 23-38
195.- ¿Cuál de las siguientes maniobras es realizada de forma rutinaria en la cirugía
endoscópica de elevación de frente?

A. Divulgación de periostio y músculos depresores


B. Completar la eliminación de la inervación motora glabellar
C. seccionamiento de nervios supraorbitario y supratrochelar
D. Overcorrection medial de la frente de contorno

Respuesta: A

Justificación:
Esta pregunta pone a prueba la comprensión de los resultados de la manipulación
quirúrgica y la causa y el efecto de las manipulaciones. Debido a que el "sling" de la parte
posterior del cuero cabelludo ejerce una fuerza hacia atrás sobre la frente, hay completa
liberación perióstica y muscular
la conexión debe ser realizada para permitir la plena elevación de la ceja y la relajación del
cuero cabelludo posterior a la fijación de puntos. La remoción completa de la inervación
motora no es un objetivo, pero una complicación de la cirugía endoscópica de frente;
temporal disminución en la inervación del nervio motor glabellar de la región es un efecto
deseado de la terapia con toxina botulínica. El nervio supratroclear supraorbitario y la
inervación sensorial de suministro, y no deben ser afectados durante cirugía. Lateral (y no
medial) frental contorno es más importante y lateral frental ptosis la principal manifestación
de frente ptosis. El exceso de corrección medialmente que perturban la normal arquitectura
de frente.

Referencia:
Cosmetic Occuloplastic Surgery, 3rd edition, Putterman, Allen M. and Warren, Linda A.
Saunders 199, pg 315
A Retrospective Comparison of Open and Endoscopic Brow-lifts, Puig, C., LaFerriere, K.,
Arch Facial Plastic Surgery 2002; 4: P 221,225
Cuzalina A: Forehead and Brow procedures. Principles of Oral and Maxillofacial Surgery.
BC Decker, London. 2004. P 1396

196.- El indicador más fiable de la frontal ptosis de las mujeres es la medición de la


distancia entre el
punto más alto de la ceja y:

A. Trichion.
B. mitad de la pupila.
C. mediados de frente.
D. Nasion.

Respuesta: B

Justificación:
El trichion se refiere a la línea de cabellos, y que no es una señal estable, no debe
utilizados para el diagnóstico y la planificación del tratamiento. El diagnóstico de la ptosis
frente es el mejor realizado pora medir la distancia entre el punto más alto de la frente hasta
la mitad de la pupila. Esta
distancia se mide delante de la mirada en una línea diagonal y es normalmente de 25 mm
en un no-ptotic cejas femeninas, medida por un Colón medidor de ceja. El punto más alto
de la ceja (APEX) en las mujeres se encuentra aproximadamente a medio camino entre el
limbo lateral y canto lateral. La mitad de la frente y Nasion son puntos de referencia más
subjetivas e inconsistentes y no son fiables.

Figure reproduced from: Evans TW: Browlift. In: Esthetic Surgery for the Aging Face.
Hupp JR (ed). Atlas of Oral and Maxillofacial Surgery Clinics of North America. Vol 6 (2),
1998. P 118

Reference:
Zukowski ML: Endoscopic Brow Surgery. Oral Maxillofac Surg Clin N Am 12(4):703-
703, 2000
Evans TW: Browlift. In: Esthetic Surgery for the Aging Face. Hupp JR (ed). Atlas of Oral
and Maxillofacial Surgery Clinics of North America. Vol 6 (2), 1998. P 118

197.- ¿Cuál de los siguientes es el candidato ideal para liposucción cérvicofacial?


A. Paciente de 50 años con la piel laxa
B. Paciente de 35 años, con moderada jowling
C. Paciente de 30 años hioides anterior
D. Paciente de 45 años de edad, con bandas platysmal

Respuesta: B

Justificación:
El candidato ideal para liposucción es generalmente inferior a 40 años, tiene una piel con
buena elasticidad, una posición favorable de hioides, y depósitos de grasa localizada. Un
paciente con exceso de piel laxa puede ser más adecuado para rhytidectomy. Anterior o de
bajo el hioides presenta una limitación de posición para el resultado deseado, y puede ser
mejor tratados con abierto y lipectoia genioplástica. Bandas del Platysmal puede verse
agravado por la liposucción cerrada, y debe ser abordarda con abierta lipectomía y
procedimientos de aplicación del músculo. Un paciente joven con aislado, moderada
jowling puede lograr el resultado deseado con sola liposucción , como los depósitos grasos
son probablemente supraplatismales.

Reference:
Cuzalina, LA; Koehler, J. Submentoplasy and Facial Liposuction. Oral Maxillofacial Surg
Clin N Am 17:85-98, 2005. Ziccardi, VB. Adjunctive Cervicofacial Liposuction. Atlas Oral
Maxillofacial Surgery
Clinics of North America 8:2, 81-97, 2000.Watts J: Cervicofacial Liposuction. Oral and
Maxillofacial Surgery Clinics of North America. Vol 12, 623-629, November 2000 Facial
Suction Lipectomy: Fonseca’s Oral and Maxillofacial Surgery Volume 6: 491-511
198.- La liposucción de la región de papada es mejor que abordada desde la incisión:

A. submentoniana.
B. submandibular.
C. infra-auricular.
D. nasolabial.

Respuesta: C

Justificación:
Una incisión en el pliegue submentoniano se utiliza sistemáticamente para llevar a cabo
lipoescultura de las regiones submentoniana y submandibular. Alternativamente,incisiones
del pliegue submandibulares también puede hacerse para eliminación de grasa en la región
submandibular y submentoniana. La región de papada se encuentra posterior al cuello
alrededor de la frontera inferior de la mandíbula. Con el fin de reducir al mínimo ñas
posibilidades de daños a las estructuras anatómicas incluyendo nervio facial, se recomienda
que esta región debe enfocarse desde un acceso posterior, como la infraestructura de la
región auricular.
Además, este enfoque de infra-auricular permite extensión subdermal y la aspiración más
del borde inferior de la mandíbula y parte superior del cuello, que puede conducir a una
mejora de contornos faciales, haciendo hincapié en la importancia de la línea de la
mandíbula. Una pequeña incisión en el área perinasal es raramente necesaria, pero puede
ser usada para acceder a un área de adiposidad facial que no se puede acceder con las
mencionadas incisiones.

Figures reproducidas por: Alexander, RW. Liposculpture of the Cervicofacial Region. In:
Esthetic Surgery of the Aging Face. Hupp JR (ed). Atlas Oral Maxillofacial Surg Clin N
Am 6:2, 1998. P 79, 81

Reference:
Alexander, RW. Liposculpture of the Cervicofacial Region. In: Esthetic Surgery of the
Aging Face. Hupp JR (ed). Atlas Oral Maxillofacial Surg Clin N Am 6:2, 73-85, 1998.
Cuzalina, LA; Koehler, J. Submentoplasy and Facial Liposuction. Oral Maxillofacial Surg
Clin N Am 17:85-98, 2005.
Jugan,MJ. Liposculpting Procedures. Peterson’s Principles of Oral and Maxillofacial
Surgery. BC Decker, Hamilton, 2004.

199.- Las señales anatómicas más relevantes que se identificaron durante la liposucción
cervical -son los siguientes:

A. vientre anterior del músculo digástrico, vientre posterior del músculo digástrico, borde
inferiore de la mandíbula.
B. cartílago tiroides, hueso hioides, submentonianos depósitos de grasa.
C. borde inferior de la mandíbula, borde anterior del músculo esternocleidomastoideo,
cartílago tiroides.
D. borde inferior de la mandíbula, hueso hioides, el vientre posterior del músculo
digástrico.

Respuesta: C

Justificación:
Estéticamente subdermal importantes depósitos de grasa se producen en el area de zona
submental central y en la región media de la mandíbula. Estos depósitos son a menudo de
naturaleza hereditarios, y pueden ser resistentes a la dieta y el ejercicio. Esta grasa es
generalmente bien distribuida en la zona del cuello, pero puede ser relativamente más
aumentada en el centro del cuello, se extiende desde el pliegue a la zona submandibular de
cartílago tiroides entre la parte anterior de los bordes bilaterales del músculo
esternomastoideo. Riesgo para las estructuras vasculares y neurológicos se minimiza si la
liposucción se limita a la zona del borde inferior de la mandíbula anterior al
esternocleidomastoideo, y superior al cartílago tiroides. Liposucción Submentoniana no
debería extenderse más allá de estos bordes.
Figure reproduced from: Alexander, RW. Liposculpture of the Cervicofacial Region. In:
Esthetic Surgery of the Aging Face. Hupp JR (ed). Atlas Oral Maxillofacial Surg Clin N
Am 6:2, 1998. P 74

Reference:
Fattahi, TT. Management of Isolated Neck Deformity. Atlas Oral Maxillofacial Surg Clin
N Am 12:261-270, 2004
Ziccardi, VB. Adjunctive Cervicofacial Liposuction. Atlas Oral Maxillofacial Surg Clin N
Am 8:2, 81-97, 2000

200.- Sangrado durante la liposucción cervicofacil si se realizan en el plano correcto es más


probable debido a
daños a la:

A. vena yugular anterior.


B. plexo plexo.
C. vena facial.
D. vena retromandibular.

Respuesta: B

Justificación:
Si la liposucción se realiza adecuadamente en la capa supraplatismal, debe ser más
profundo en los vasoso para mínimo riesgo. Si la cánula de aspiración se coloca
inadvertidamente al músculo platisma, que puede provocar daños a las venas yugular
anterior, faciales, o retromandibular. Mientras el sangrado es mínimo con la técnica
tumescente, puede haber daños al plexo subdermal

Reference:
Abraham MT, Romo TR. Liposuction of the Face and Neck. Emedicine.com, 2005.
Jugan MJ. Liposculpting Procedures. In: Peterson’s Principles of Oral and Maxillofacial
Surgery. BC Decker, Hamilton, 2004. P 1410-11.

201.- Aumento de labios utilizando materiales dérmicos aloplásticos es realizado


principalmente a:

A. aumento de la exposición bermellón.


B. disminución de rollo blanco definido.
C. alrededor de la comisura aaoral.
D. eliminar cupids proa.
Respuesta: A

Justificación:
Labios estéticos son el resultado de una agradable combinación de volumen, definición
anatómica, faneca, y la simetría. El esquema básico de una estética del labio superior es la
forma de una M y el labio inferior es curvilíneo o parabólico. El labio superior contiene un
tercio del total de volumen de labios y el labio inferior añade dos tercios del volumen de los
labios. El uso de materiales autógena o aloplásticos en los labios sobre todo logra un
aumento de volumen de labios y exposición bermellón. Bien definidos Cuspide Bow,
mayor despliegue de color blanco, filtro visble columnas, y comisuras fuerte contribuye a
la estética y los labios son los objetivos al relleno inyectables

Reference:
Sykes JM, Emery BE. Upper and lower lip augmentation with dermal autografts. Oper tech
otolaryngol head & neck surg. 1995; 6:307.
Niamtu, J. Advanta facial implants. Oral Maxillofacial Surg Clin N Am. 2005; 17:29.

202.- Aumento de labios con colágeno bovino es el mejor realizado por inyección de
materiales en que plano?

A. Epiteliales
B. Dérmico
C. Subdermico
D. intramuscular

Respuesta: B

Justificación:
Desde la FDA decisión de retirar la autorización de materiales de silicona inyectables,
bovinos colágeno se hizo popular como un material de relleno de labios. Duración de la
mejoría clínica después de la inyección varía con la selección del sitio, el grado de
actividad muscular en el sitio de inyección, y la técnica de inyección. Los resultados
pueden durar hasta 2-3 años, aunque la mayoría de los pacientes a cambio de un "retoque"
de seis meses a un año después de la cirugía inicial. Un "layering" técnica ha demostrado
dar mejores resultados. Correcta ubicación secuencial implica inyecciones de colágeno en
el plano intradermotuberculinización. Cuanto menos enlaces antigénicos, cruzados,
glutaraldehído tratados colágeno (Zyplast) se inyecta en la dermis profunda, seguido por
colocación de Zyderm colágeno en el plano dérmico superficial.

Reference:
Kamer FM, Churukian MM: Clinical use of injectable collagen. Arch Otolaryngol 1984:
110: P 93-98 Sykes JM, Emery BE. Upper and lower lip augmentation with dermal
autografts. Oper tech otolaryngol head & neck surg. 1995; 6:307.
203.- El nervio más frecuentemente lesionado en la cara en procedimientos de
levantamiento es:

A. bucal.
B. espinal accesorio.
C. auricular mayor.
D. marginal mandibular.

Respuesta: C

Justificación:
Perjuicio a servir la causa de parálisis facial es rara, y sólo informó de que se produzca en
0,53 a 2,6% de los pacientes. Ochenta y cinco por ciento de las lesiones del nervio motor
resolver espontáneamente, y resultados de la reparación quirúrgica son impredecibles y no
muy alentadores. Daño a la mayor nervio auricular es la más común y ocurre en hasta un
7% de los pacientes. Neuropraxia temporal por lo general se resuelve en 2-4 meses, y causa
entumecimiento / parestesia alrededor de la inferior parte de la oreja y la piel circundante.
Transección del nervio gran auricular es la mejor tratados con microanastomosis inmediata.

Reference:
Waite PD, Cuzalina LA: Rhytidectomy (Face lift). In: Fonseca R, Baker S, Wolford LM
(eds). Oral & Maxillofacial Surgery, Vol 6, WB Saunders, Philadelphia, 2000. P378 Rees
TD, Aston SJ, Thorne CH. Postoperative considerations and complications in aesthetic
plastic surgery. Rees TD and Latrenta GS. Second edition Vol II. WB Saunders Company
1994. p 273

204.- Mujer de 50 años de edad, desea mejorar su apariencia cérvico-faciales con lift
cirugía.

¿Cuál de las siguientes técnicas quirúrgicas es la mejor opción?


A. Subcutánea
B. Composite
C. Mini-levantamiento
D. Endoscópica

Respuesta: B

Justificación:
Los diversos procedimientos disponibles para rhytidectomy se pueden clasificar
anatómicamente según a la profundidad de la disección de la piel a periostio. Un "piel only"
lifting es el procedimiento más simple, pero generalmente no se indica. El SMAS face-lift
implica cirugía tanto para el SMAS y la piel. Esto puede lograrse por reconstrucción de
SMAS ya sea después de eliminación de una tira de piel preauricular o por una aplicaión
sola de SMAS. El grado de la elevación de colgajo de SMAS es variable from none, a una
pequeña cantidad, la extención sub SMAS, la elevación al borde lateral del músculo
zygomaticus el más importante de la cara. En el mini-lift, incisión de la piel es limitada y el
SMAS es aplicado con una serie de suturas en la parte inferior de cara y cuello.levantar la
cara en un plano profundo, la disección es en un plano por debajo de la grasa malar
almohadilla. Un lifting compuesto agrega la disección del colgajo de SMAS de la porción
inferior del orbicular del ojo a la disección del plano profundo. El lifting subperióstico
eleva el periostio del arco cigomático y la cara anterior del maxilar superior con el fin de
reposicionar toda la unidad superior.
Face lit SMAS el procedimiento necesita ser individualizado; En este paciente, un
subcutáneo lifting no abordará las deformidades esqueléticas, ptotic profundos estructuras
de los tejidos blandos o cambiar la textura de la piel. Una técnica mini-lifting no en
dirección de la zona cervical y región malar grasa almohadilla predecible. Un lifting
endoscópico opción no va a mejorar la textura de la piel,
perióstica y dará a conocer las conexiones con los tejidos blandos superficiales sólo mejorar
las áreas
superior facial y mediofacial. Se espera obtener los mejores resultados con el uso de un
compuesto rhytidectomy incorporar multiples planos de disecciones SMAS y
procedimientos, lo que dará mejor control de los tejidos blandos y proporcionar resultados
más duraderos.

Reference:
Ghali GE, Evans TE. Rhytidectomy. Peterson’s principles of Oral and Maxillofacial
Surgery. Second edition. 2004. Vol 2: 1371
Waite PD, Cuzalina LA. Rhytidectomy (face-lift). Fonseca Oral and Maxillofacial Surgery.
Cleft/ Craniofacial, Cosmetic Surgery. 2000. Vol 6:379

205.- El procedimiento que ajusta y asegura el borde posterior del sistema


musculoaponeurotico superficial (SMAS)-platysma capa muscular en un procedimiento de
lifting es el mejor calificado:
A. Imbricación.
B. Involución.
C. Placation.
D. Transposición.

Respuesta: A

Justificación:
Con la actual tendencia limitada hacia la cirugía de cara, la identificación y uso eficaz de la
capa SMAS es fundamental para el éxito y la longevidad de rhytidectomy. Esta capa,
inicialmente descrita por Mitz y Peryonie en 1976, incluye la fascia temporal superficial
superior, y se extiende hasta la superficie superficial del músculo platisma inferior.
Se encuentra ajena a la cápsula de parótida, y contiene cantidades variables de grasa.
Después de desarrollo de los colgajos de piel y SMAS, el SMAS capa puede ser
manipulado por cualquier plication (sutura y sin plegar la excisión) o imbricacion (la
excisión y reposicionamiento) o una combinación de ambos procedimientos.

Reference:
Chisholm BB: Surgical facial rhytidectomy. Oral Maxillofac Surg Clin North Am 12,
2000: 719-728
Ghali GE, Evans TE. Rhytidectomy. Peterson’s principles of Oral and Maxillofacial
Surgery. Second edition. 2004. Vol 2: 1371

206.- Relativo la cirugía de estirar la cara, la incidencia de necrosis de colgajo de piel


encontramos que se encuentra mucho más alto en fumadores en comparación con los no
fumadores?

A. 5
B. 8
C. 12
D. 15
Respuesta: C

Justificación:
Algunos cirujanos consideran el tabaquismo como una contraindicación absoluta: sin
embargo, la mayoría
Recomendamos que se deje de fumar por lo menos 2 a 6 semanas antes de la cirugía, y por
lo menos 2 semanas después de la cirugía. La tasa global de complicaciones para los
pacientes que siguen fumando es más del doble en comparación con los pacientes que dejan
de fumar antes de la cirugía. Bupropión
(Wellbutrin) se usa a menudo para ayudar en la cesación del hábito de fumar. Parches de
nicotina y goma de mascar
aún pueden plantear los mismos riesgos como el consumo de tabaco en términos de entrega
de nicotina y sus
efectos vasoconstrictores.

Reference:
Waite PD, Cuzalina LA: Rhytidectomy (face lift). In: Fonseca R, Baker S, Wolford LM
(eds). Oral & Maxillofacial Surgery, Vol 6, WB Saunders, Philadelphia 2000. P368
August Meredith: Complications in oral and maxillofacial surgery. Oral and Maxillofacial
Surgery clinics of North America, May 2003 pages 278-280.
Schaner P. Evaluation of the esthetic surgery patient. Atlas of Oral and Maxillofacial
Surgery Clinics of North America. P121-30, 2004.

207.- Macgregor la revisión se refiere a una zona adyacente de la:


A. esternocleidomastoideo y lóbulo oreja, donde se encuentran importantes nervios.
B. Arco cigomático y la prominencia donsde un plexo de vasos se encuentra.
C. muesca antegonial y parte inferior de la mandíbula donde se encuentra la arteria facial.
D. Considerando que en el área preauricular cuando el nervio facial cruza el arco
cigomático.

Respuesta: B

Justificación:
McGregor´s sutura también es conocida como el "gulch sangrienta". Su nombre se debe al
fuerte conjunto fibroso
zygomatico-cutánea, que a menudo presenta en la dimpling piel o retracción. Esto también
es importante porque un plexo de vasos suministrado por la arteria facial transversa
se convierte en la arteria cervical superficial en esta área. El daño a estos vasos puede crear
sangrado durante el desarrollo del colgajo de piel en el procedimiento lifting. Además, el
nervio bucal se encuentra justo en la zona profunda de peligro, y la rama facial del
cigomático se hace más superficial en esta área.

Reference:
Waite PD, Cuzalina LA: Rytidectomy (Face Lift). In: Fonseca R, Baker S, Wolford LM
(eds). Oral & Maxillofacial Surgery, Vol 6, WB Saunders, Philadelphia, 2000. P 366-7
Goldwyn RM. Late bleeding after rhytidectomy from injury to the superficial temporal
vessels. Plastic Reconst Surg 88(3): 443-445, 19

208.- McGregor la revisión se refiere a una zona adyacente a la:


A. Esternocleidomastoideo y lóbulo oreja, donde se encuentran importantes nervios.
B. Arco cigomático y la prominencia que un plexo de los buques que se encuentra.
C. Antegonial muesca e inferior de la mandíbula donde se encuentra la arteria facial.
D. Considerando que en el área preauricular cuando el nervio facial cruza el arco
cigomático.

Respuesta: B
Justificación:
McGregor parche también es conocido como el "gulch sangriento". Su nombre se debe el
fuerte zygomatico-cutánea fibroso archivos adjuntos que, a menudo presente en la piel o
retracción dimpling. Ello también es importante porque un plexo de vasos suministrada por
la arteria facial y transversal se convierte en la arteria cervical superficial en este ámbito. El
daño a estos vasos puede crear sangrado durante el desarrollo del levantamiento del colgajo
procedimientos. Además, el nervio bucal se encuentra justo en esta zona profunda de
peligro, y la rama facial del cigomático se hace más superficial en esta área.

Reference:
Bloomquist DS, Lee JJ. Principles of mandibular orthognathic surgery. Peterson’s
principles of Oral and Maxillofacial Surgery. Second edition. 2004. Vol 2: 1168-1173
Nishioka GJ, Mason M, Van Sickels JE. Neurosensory disturbance associated with the
anterior mandibular horizontal osteotomy. JOMS 1988: 46: 107
Hwang K, Lee WJ, Song YB, et al: Vulnerability of the inferior alveolar & mental nerve
during genioplasty: An anatomic study. J Craniofac Surg 2005: 16: 10

209.- El tamaño mínimo de poro de los implantes faciales que permite el acúmulo de la
resistencia bacteriana a
infección:

A. <1 micra.
B. 10 a 25 micrones.
C. 30 a 50 micrones.
D. > 50 micrones.

Respuesta: D

Justificación:
Implantes porosos tienen el potencial de crecimiento de bacterias que pueden introducirse
en el momento de la cirugía o después de la intervención, debido a la ruptura del tejido.
Esto ocurre cuando los poros de tamaño son> 1 micra. Defensas del huésped humano
incluyendo macrófagos requieren un tamaño de poro de> 50 micras para entrar y afectar las
bacterias que han infectado el implante. Por lo tanto, el implante ideal
tendría poros inferior a 1 micrón para evitar la inoculación de bacterias o > 50 micrones
para permitir a los macrófagos con destruir a las bacterias.

Reference:
Cohen MS, Constantino PO, Friedman CD. Biology of implants used in head and neck
surgery. Fac Plast Surg Clin North Am. 1999; 9: 17
Eppley BL. Alloplastic implantation. Plast Reconstr Surg. 1999; 104:1671.

210.- Que dato cefalométrico muestra aposición ósea después de un adelanto de


deslizamiento horizontal genioplastico óseo?
A. Pogonion (Po)
B. Gnathion
C. punto B (B)
D. Menton (Me)

Respuesta: C

Justificación:
Varios investigadores han demostrado que mínima disección de los tejidos blandos hizo
más previsible de tejidos blandos y duros, porque la respuesta es menos dentro de la
resorción ósea avanzadas del segmento. Con tan mínima disección de tejidos blandos, no se
observó la remodelación ósea gnathion o menton. Aposición ósea se produce en el punto B
y pogonion muestra resorción ósea.

Reference:
Betts NJ, Edwards SP. Soft tissue changes associated with orthognathic surgery. Peterson’s
principles of Oral and Maxillofacial Surgery. Second edition. 2004. Vol 2: 1229-1233
Van Sickels JE, Smith CV, Tiner BD, Jones DL: Hard and soft tissue predictability with
advancement genioplasties. Oral Surg Oral Med Oral Pathol 1994 Mar; 77(3): 218-21
Polido WD, Bell WH. Long-term osseous and soft tissue changes after chin advancements.
J. Craniomaxillofac Surg. 1993, 21: 54-9

211.- ¿Cuál de las siguientes declaraciones en relación con la evaluación de la estética


tercio medio facial es cierto?

A. La prominencia cigomática deberían estar situados a 2 cm inferior y de 1,5 a 2 cm lateral


al canto lateral del ojo.
B. El borde infraorbitario debería ser 0 a 2 mm posterior a la córnea
C. Más de 3-4 mm de esclera debe ser visible inferior entre el limbo y el párpado inferior.
D. La importancia mejilla deberían estar situados a 3 mm superior al plano de Frankfort
horizontal.

Respuesta: A

Justificación:
La región de tercio medio facial es el mejor evaluado en cuatro visitas - frontal en reposo-y
sonriendo, perfil, de tres cuartos oblicua y basal. Anchura bicigomática normal debería ser
88,5% + / - 4,6% de la altura facial (medido a partir de Nasion gnathion) en milímetros. La
prominencia cigomática se encuentra inferior 2cm y 1,5 - 2 cm lateral al lateral. El borse
Infraorbitario debe ser 0-2mm anterior a la córnea. Esclera Normal mustra es inferior a 4
mm. Evaluación Cefalométrica evalua de la región cigomática debe incluir el uso de
McNamara Nasion perpendicular, y la distancia del borde infraorbitario a esta línea
perpendicular es de aproximadamente 5 mm. El cigomáticos (mejilla) conviene
situado por debajo del plano horizontal Frankfort, y no encima de ella.

Reference:
Zide and Epker. Systematic Aesthetic Evaluation of the Cheeks for Cosmetic Surgery.
Cosmetic Oral & Maxillofacial Surgery, Oral & Maxillofacial Surgery Clinics of North
America. Vol. 2, No. 2, May 1990.
Eppley BL. Alloplastic implantation. Plast Reconstr Surg. 1999; 104:1671.

212.- La etiología más común de pérdida de cabello en los hombres es:


A. alopecia areata.
B. alopecia androgenética.
C. alopecia de tracción.
D. estrés psicológico.

Respuesta: B

Justificación:
Alopecia androgentic es la causa más común de patrón de calvicie masculina. Se estima
que afecta aproximadamente al 60-80% de los hombres, y una hormona
(Dihidrotestosterona) se asocia con esta causa de la calvicie masculina. La predisposición
genética contribuye a testosterona el número de receptores de las células foliculares y la
actividad de la 5-alfa reductasa actividad en diferentes zonas del cuero cabelludo. Esta
enzima reduce la síntesis de testosterona y la síntesis de proteínas, lo que la producción de
finos y pelo más fino, hasta que el cabello se pierde. Alopecia areata es una enfermedad
poco frecuente que los parches redondos de pérdida de cabello aparecen de repente. El
cabello, cada vez es atacado por el tejido propio del paciente células inmunitarias por
razones desconocidas.
Alopecia de tracción se asocia con tensión sostenida en el cuero cabelludo el pelo, y la
tracción causas para aflojar el cabello de sus raíz folicular. También se produce la pérdida
del cabello a la secundaria local folicular la inflamación y atrofia. Los pacientes sometidos
graves fisiológicos (por ejemplo, con aguda y enfermedad crónica sistémica) los cambios
significativos o el estrés psicológico pueden sufrir de algunos grados de pérdida de cabello,
que suele ser temporal.

Reference:
Cummings: Otolaryngology: Head & Neck Surgery, 4th ed. Page 673-680
Unger WP, Nordstrom REA: Hair transplantation. Marcel Dekker, New York 1998.
Hendler BH: Hair restoration surgery. Esthetic surgery of the aging face. Oral and
Maxllofacial Surgery Clinics of North America. Vol 6(8), 1998. P 40-1

También podría gustarte